Download as pdf or txt
Download as pdf or txt
You are on page 1of 229

LUMINAIRE-2015

LUMINAIRE
2015
PART A
• PATHOLOGY
• PHARMACOLOGY
• MICROBIOLOGY
• FORENSIC MEDICINE

1
GOVT MEDICAL COLLEGE, TRIVANDRUM
LUMINAIRE-2015

GOVT. MEDICAL COLLEGE , THIRIVANANTHAPURAM


LUMINAIRE 2015

Published by
College Union Medical College Thiruvananthapuram

Cover Design
Vishnu Dev A M

Type setting
2011 MBBS Batch
Govt Medical College TVM

Contact
Bhagyaraj S
8907464900

2
GOVT MEDICAL COLLEGE, TRIVANDRUM
LUMINAIRE-2015

Editors Note.........

As all of you know, over a period of time, LUMINAIRE has established its niche in the
dissemination of information related to the syllabus for II and III MBBS examinations.
We earnestly tried to update the contents and make it more affable for the students
taking into consideration the changes in the examination pattern. Such changes would
make it a valuable handbook as well as compendium which are sure to be received
well by the fraternity of medical students. To be different, an attempt has been made
to make it more PG entrance oriented as well with a view to ease the pressure on part
of the aspirants. The completion of work within the stipulated time was a tall order
and we would be failing in our duty if we don’t gratefully acknowledge the
contributions of Maqbal Muhasin Nasar and Anandhavishnu S without whom the
venture would not have been a success. We all know that perfection is something
unattainable, but we take solace in the words of Antoine de Saint Exupery: “A designer
knows he has achieved perfection not when there is nothing left to add, but when there
is nothing left to take away”. We can claim success in our mission if this book helps
you to come out with flying colours in the exams.

AKHIL UNNIKRISHNAN ARJUN GOPI


Co Convener CONVENER

3
GOVT MEDICAL COLLEGE, TRIVANDRUM
LUMINAIRE-2015

MESSAGE FROM THE STUDENTS’ FEDERATION OF INDIA

The Students’ Federation of India has always stood with the marginalized
sections of society. Our organization has a significant presence in
campuses around the world. We have been at the forefront of informing
students about various issues of public interest and generate reaction and
creative discussions with them.
Our organization has always fought against social and religious barriers to
prevent the spread of communalism and create the humanism among the
students. We have also strived to promote literary, cultural and educational
movements which have helped change the face of society.
We utilize this occasion to pay our deep respect to great visionaries of
past, like Dr. Ernesto Che Guevara, the doctor- poet -revolutionary who is the
thrilling epitome of the leftist revolutionary movement.
Us together,

PRESIDENT SECERETARY
MIDHUN MOHAN ABIN A J

Revolutionary Greetings SFI Medical College Unit

4
GOVT MEDICAL COLLEGE, TRIVANDRUM
LUMINAIRE-2015

PATHOLOGY
PAPER- I:- General Pathology and Clinical Pathology
PAPER-II:- Hematology and Systemic Pathology

QUESTION PAPER PATTERN


• Essay- 8 marks
• Clinical essay- 6 marks
• Short notes- 4 x 4 marks
• Short notes- 4 x 2 marks
• Short answers- 4 x ½ marks

EXAM TIPS:-
• Learn your CLIP record thoroughly for paper I.
• Learn related hematology portions for paper I.
• Draw histopathological diagrams wherever necessary.
• In case if any tumor is asked, write it under following headings- benign/malignant,
etiology, pathogenesis, gross and microscopy, clinical features, lab findings, staging (if
possible) and prognosis.

“ALL THE BEST”

5
GOVT MEDICAL COLLEGE, TRIVANDRUM
LUMINAIRE-2015

CLINICAL PATHOLOGY
1. FNAC.
2. CSF findings in Meningitis.
3. Sperm Analysis.
4. Urine microscopy- casts & crystals
5. ESR
6. PCV
7. Coomb’s test
8. Barr body
9. Steps in tissue processing
10.Fixatives
11.Bone marrow aspiration & biopsy
12.Proteinuria- causes
13.Ketonuria - causes
14.Leishman’s stain
15.Anti coagulants
16.Pap smear
17.Lipid special stains
18.Immunohistochemicalstains
19.Flow cytometry

CELL INJURY (IMP)


1. Cell injury – causes, mechanism, reversible and irreversible types
2. Reperfusion injury, Free radical injury
3. Necrosis.
4. Apoptosis
5. Gangrene
6. Pathological calcification
7. Metaplasia
8. Fatty change
9. Intra cellular accumulations.
10.Endogenous pigments.
11.Dysplasia
12.Lipofuscin
13.Define- hyperplasia, hypertrophy, atrophy with examples.
6
GOVT MEDICAL COLLEGE, TRIVANDRUM
LUMINAIRE-2015

INFLAMMATION AND HEALING (IMP)


1. Inflammation- definition, signs, types, vascular & cellular events, mediators,and
outcome in inflammation(8 marks ESSAY)
2. Wound healing- types, mechanisms, factors and complications. (8 marks essay)
3. Chemical mediators of inflammation
4. Morphological patterns & outcome of a/c inflammation
5. Granulomatous inflammation
6. Granuloma- definition, Evolution & fate of tubercle
7. Fracture healing
8. Cell adhesion molecules
9. Granulation tissue
10.Acute phase reactants

HEAMODYNAMIC DISORDERS (IMP)


1. Shock – Types, pathogenesis, stages, manifestations (8 marks essay)
2. Embolism- definition, Pulmonary embolism,Fat embolism, Air embolism, Amniotic
fluid embolism (8 marks essay)
3. Thrombosis – defn, pathogenesis (virchows triad), types, fate (8 marks essay)
4. Edema-defn, types, mechanisms (8 marks essay)
5. CVC – Lung, Liver, Spleen.
6. D.I.C
7. Infarction-defn, morphology, clinical outcome
8. Gamma gandy bodies, heart failure cells
9. Decompression sickness
10.Differences between an exudate and transudate

GENETICS& PEDIATRIC DISORDERS


1. Karyotyping
2. Down’s syndrome
3. Turner’s syndrome
4. Klinefelters syndrome
5. Gauchers diseases
6. Marfan’s syndrome
7. Common pediatric malignant tumors
7
GOVT MEDICAL COLLEGE, TRIVANDRUM
LUMINAIRE-2015

IMMUNOLOGY
1. SLE - Etiology and pathogenesis, serological test, Renal lesions, cardiac lesions,
morphology of other organs, hematological presentation (8 marks essay)
2. Amyloidosis –defn, types, classification, Staining, Morphology in differrent organs(8
marks essay)
3. HIV-Pathogenesis in HIV, neoplasms in AIDS, opportunistic reactions in AIDS
4. Hypersensitivity reactions- all types with examples.
5. Transplant rejection
6. GVH reaction
7. Immune tolerance
8. Sago spleen, Lardaceous spleen
9. MHC

INFECTIOUS AND PARASITIC DISEASE


1. Tuberculosis- primary and secondary TB, primary sites of TB, pathogenesis,
diagnosis, Mantoux test(8 marks essay)
2. Leprosy - reactions in leprosy, Lepromin test, difference between lepromatous and
tuberculoid leprosy
3. Infectious Mono Nucleosis(IMN)
4. TORCH complex
5. Syphilis
6. Malaria- Black water fever
7. Mycetoma
8. Candidiasis
9. Amoebiasis
10.Aspergillosis
11.Rhinosporidiosis

NEOPLASIA (IMP)
1. Neoplasia- defn, difference between benign and malignant tumors
2. Carcinogenesis- types, mechanisms, and features (8 marks essay)
3. Metastasis- pathways of spread.
4. Anaplasia
5. Oncogenes- defn, types, mechanisms.
8
GOVT MEDICAL COLLEGE, TRIVANDRUM
LUMINAIRE-2015
6. Tumor suppressor genes- RB gene and p53
7. Paraneoplastic syndromes
8. Immune surveillance
9. Tumour markers
10.Tumour antigens
11.Premalignant lesions- carcinoma in situ
12.Grading & staging of cancer

RBC DISORDERS
1. Anaemia- defn, types and classification.
2. Megaloblastic anemia- Biochemical basis, Classification, Blood and BM findings
Pernicious anemia
3. Iron Deficiency Anemia - etiology, peripheral smear, bone marrow findings,
biochemical test, RDW.
4. Sickle Cell Anemia - pathogenesis, investigation, peripheral smear, Sickling test.
5. Thalassemia - pathogenesis, Peripheral smear investigation
6. Hemolytic anemia - Classification, Investigations***
7. Immunohemolytic anemia
8. Hereditary spherocytosis
9. Sideroblastic anemia
10.Blood cell indices
11.Aplastic anaemia
12.Reticulocyte count
13.MAHA
14.Coomb’s test
15.PNH

PLATELET DISORDERS
1. ITP and TTP
2. Thrombocytopenia
3. Hemophilias - A & B
4. Von Willebrand’s d/s
5. DIC
6. Blood transfusion reactions
7. Hemolytic diseases of new born

9
GOVT MEDICAL COLLEGE, TRIVANDRUM
LUMINAIRE-2015

WBC DISORDERS
1. AML - FAB & WHO classification of acute leukemia, peripheral smear findings,special
stains(essay)
2. ALL - Classifications, Blood picture, Special stains(essay)
3. CML - Molecular abnormality, natural history, Blood & Bone marrow picture, DD-
leukemoid reaction(essay)
4. Leukemoid reactions
5. IMN
6. Agranulocytosis
7. Myelodysplastic syndrome
8. Polycythemia vera
9. Myeloblast, megaloblast, lymphoblast

LYMPHOID SYSTEM
1. Hodgkins lymphoma - Classification, Reed - Sternberg cells - types, Ann Arbor
staging
2. Multiple myeloma - investigations, urinary findings, Bence - Jones protein, plasma
cell, Russelbodies and Dutcher bodies in MM

THEHEART
1. MI-Gross &light microscopic features,consequences,complications,markers for
diagnosis (8 marks essay)
2. RHD-Aetiopathogenesis,Jonescriteria,Gross and microscopy with aschoff body
(essay)

3. Vegetations-types(in RHD,SLE,NBTE,IE)
4. Pericarditis-types
5. Cardiomyopathy-Types
6. Infective endocarditis
7. Aschoff bodies
8. Mac callum patch
9. Bread & butter pericarditis
10.Libman sacks endocarditis
11.Myocarditis

10
GOVT MEDICAL COLLEGE, TRIVANDRUM
LUMINAIRE-2015
12.Loefflers endocarditis

BLOOD VESSELS AND LYMPHATICS


1. Atherosclerosis- Aetiopathogenesis, riskfactors, AHA classification, foam cells, fatty
steak, atheromatous plaques, sequalae.
2. Aneurysms-classsification,dissecting aneurysm.
3. Arteritis - Classification of arteritis,giant cell arteritis
4. Capillary and cavernous hemangioma.
5. Wegenersgranolomatosis
6. Vascular changes in hypertension

THE RESPIRATORY SYSTEM


1. Bronchogenic carcinoma – etiopathology, histologic types,gross,microscopy
2. Emphysema-aetiopatho,morphology-gross and microscopy
3. Bronchiectasis
4. Pneumoconiosis
5. Pneumonia- stages of pneumonia
6. ARDS
7. Atelectasis
8. Chronic bronchitis-reid index
9. Bronchialasthma-Curschmannspiral,charcotleden crystals
10.Lung abscess

HEAD AND NECK


1. Pleomorphic adenoma
2. Warthins tumor
3. Leukoplakia
4. Hairy leukoplakia
5. Erythroplakia
6. Adenoid cystic Carcinoma
7. Mucoepidermoid Carcinoma

THE GASTROINTESTINAL SYSTEM


1. Gastric Ca- classification(WHO &Lauren), risk factors, morphology,
spread,investigations

11
GOVT MEDICAL COLLEGE, TRIVANDRUM
LUMINAIRE-2015
2. Colorectal Ca- aetiopathogenisis,clinical features, morphology, TNM
classification,associated tumor markers, investigations
3. Peptic ulcer
4. Crohns disease and ulcerative colitis
5. Colorectal polyps- definition, types
6. Barrett’s oesophagus
7. Typhoid ulcer, Tuberculosis ulcer and Amoebic ulcer
8. Carcinoid
9. Meckel diverticulum
10.Achalasia cardia
11.Pseudomembranous enterocolitis
12.Curling ulcer
13.Cushing’s ulcer
14.Intussusception
15.Appendicitis

LIVER AND PANCREAS


1. Cirrhosis- classification,aetiopathogenesis,morphology,complication (essay)
2. Hepatocellular carcinoma - essay
3. Hepatic failure-hepatorenalsyndrome,hepaticencephalopathy
4. Viral hepatitis-hepatitis B,Cmorphology
5. Alcoholic liver d/s
6. Primary biliary cirrhosis
7. Jaundice
8. Gallstones and types
9. Pancreatitis - a/c &c/c
10.Mallory body
11.Councilman body
12.Haemochromatosis(pigment cirrhosis,bronze diabetes)
13.Wilson’s d/s
14.Liver abcess
15.Hydatid d/s
16.Cholecystitis

KIDNEY AND LOWER URINARY TRACT

12
GOVT MEDICAL COLLEGE, TRIVANDRUM
LUMINAIRE-2015
1. Post streptococcal glomerulonephritis-etiology,urinaryfindings,microscopic
findings in renal biopsy,investigation
2. Renal cell carcinoma-histological types,morphology,paraneoplastic syndromes.
3. Nephrotic syndrome-causes,test to confirm the diagnosis,causes of proteinuria.
4. IgA nephropathy
5. C/c glomerulonephritis
6. Diabetic nephropathy
7. Pyelonephritis
8. Hypertensive Changes In Kidney
9. Hydronephrosis
10.Renal Calculi
11.Wilm’s Tumor
12.Renal Lesion In SLE - Lupus Nephritis
13.PKD
14.Glomerulonephritis- RPGN , FSGN , MPGN
15.A/c tubular necrosis
16.Difference between nephritic and nephrotic syndromes

MALE GENITAL SYSTEM


1. Testicular tumour - Classification
2. Seminoma
3. BPH
4. CA prostate
5. Bowen’s d/s&Bowenoidpapulosis
6. Schiller duval body

FEMALE GENITAL TRACT


1. Ovarian tumors-WHO classification ( Essay )
2. Cervical ca –risk factors,pathogenesis,morphology,staging,investigation( Essay )
3. CIN
4. Endometriosis,Adenomyosis
5. CA endometrium
6. Leiomyoma uterus
7. Teratoma
8. Dysgerminoma

13
GOVT MEDICAL COLLEGE, TRIVANDRUM
LUMINAIRE-2015
9. Choriocarcinoma
10.Hydatidiform mole
11.Brenners tumor
12.Krukenberg tumor
13.Struma ovary
14.Chocolate cyst of ovary

THE BREAST
1. Ca breast – classification,riskfactors,staging, Prognostic Factors ( Essay )
2. Fibroadenoma
3. Paget’s D/s of nipple

SKIN
1. Squamous Cell Carcinoma
2. Basal Cell Carcinoma
3. Malignant Melanoma
4. Distinguish b/w benign mole and malignant melanoma

ENDOCRINOLOGY
1. Diabetes Mellitus- Including Pathogenesis Of Type 1 &2 DM
2. Pheochromocytoma
3. Hashimoto’s Thyroiditis
4. Grave’s D/S
5. Ca Thyroid- Classification and features of each type
6. Multiple Endocrine Neoplasia
7. Cushing’s Syndrome
8. Addison’s D/S
9. Hyper&Hypothyroidism
10.Goitre-Simple &MNG
11.Follicular adenoma Thyroid
12.Sheehan’s Syndrome
13.Psammoma Bodies

THE MUSCULOSKELETAL SYSTEM


1. Classification of Bone tumor

14
GOVT MEDICAL COLLEGE, TRIVANDRUM
LUMINAIRE-2015
2. Osteosarcoma- Etiology,Gross &Microscopy, Radiological Appearance
3. Ewing’s Sarcoma –Etiology , Gross & Microscopy , Radiological Appearance
4. Osteochondroma
5. Osteoclastoma (Giant Cell Tumor)
6. Osteomyelitis
7. Paget’s Disease Of Bone
8. PNET
9. Rheumatoid Arthritis
10.Osteoporosis
11.Osteogenesis Imperfecta
12.Osteoarthritis
13.Pott’s D/S
14.Psoas Abcess
15.Gout

SOFT TISSUE TUMORS


1. Lipoma,Liposarcoma
2. Rhabdomyoma,Rhabdomyosarcoma
3. Myositis Ossificans

THE CENTRAL NERVOUS SYSTEM


1. CSF findings in TB , Pyogenic and Viral Meningitis
2. Meningioma
3. Glioma , Astrocytoma
1. Brain Abscess
2. Medulloblastoma
3. Schwannoma,Neurilemmoma

EYES&ENT
1. Retinoblastoma
2. Rhinosporidiosis

MODEL ESSAYS

15
GOVT MEDICAL COLLEGE, TRIVANDRUM
LUMINAIRE-2015
1. 24 year old female presents with fever, joint pain, puffiness of face since 2
weeks. Irregular discolouration of facial skin and alopecia was noted.
a) What is the provisional diagnosis?
b) Describe etiology and pathogenesis
c) Mention two serological tests by which you can confirm the diagnosis
d) Describe the renal lesions in this condition and what is the most
common pattern of renal lesion?
e) What are the cardiac lesions that can occur in this patient?
f) Mention the pulmonary manifestations of this condition.
g) What is the usual hematological presentation in this condition?
h) What is the usual cause of death?(renal disease)

2. 25 year old man was involved in a major road traffic accident. He had multiple
injuries and was bleeding profusely. When brought to the casualty, he was in stupor.
His BP was 80/40 mm Hg and pulse 145/min. He was tachypnoeic with cold
extremities and died the following day. An autopsy was done.
a) What type of shock did he develop?
b) What are the other types of shock?
c) Describe the pathogenesis of septic shock
d) What are the stages of shock?
e) What are the morphological changes in the organs at autopsy?

3. 5 year old child was admitted with fever, vomiting, unconsciousness in the
hospital. O/E he showed neck rigidity, petechial spots on the skin. TC- 18000 DC P78
L20 E2
a) What is your diagnosis?
b) What is the ideal specimen to be collected for diagnosis?
c) What is the special stain to be used?
d) Mention the etiological agents of this condition.
e) What are the likely CSF findings?
f) What are the complications of lumbar puncture?

4. 60 year old female sustained fracture neck of femur,. On the 2nd day she
developed sudden severe dyspnoea, tachycardia, tachypnoea and died.
a) What is the provisional diagnosis?
b) Mention other causes of the diagnosis
16
GOVT MEDICAL COLLEGE, TRIVANDRUM
LUMINAIRE-2015
c) What is the pathogenesis of this condition?
d) Describe the morphology of the lesion in the lung.
e) What are the special stains to be used to reach the diagnosis.

5. 58 year old male came with the h/o c/c bleeding PR since 1 year. O/E he was
very pale. Nails showed koilonychia.
a) What is the diagnosis?
b) Mention the investigations needed to confirm the diagnosis.***
c) What are the etiological factors responsible?
d) What is the blood picture and bone marrow findings in this condition.***

6. 60 year old male presents with gradual darkening and shrivelling of right toe.
a) What is the diagnosis?
b) Name two basic investigations you will do for this patient.
c) Describe the morphological appearance of the lesion.
d) How will you differentiate it from other types of similar lesions.

7. 8 year old boy from a tribal village at Wayanad came with the h/o severe pain in
fingers and toes. There is h /o intermittent jaundice. O/E the child had pallor, jaundice
and hepatomegaly. Investigations Hb-6g/dl, Reticulocyte count-17%, Urobilinogen +++
a) What is the provisional diagnosis?
b) List other investigations indicated in this patient.
c) How will you confirm your diagnosis?
d) Describe the peripheral smear findings inthe patient.
e) What is the basic abnormality which caused the disease.

8. 24 year old male presented with fatigue, fever, gum hypertrophy and
spontaneous mucosal and cutaneous bleeding. There is also recent h/o recurrent
infections of resp tract and oral cavity. O/E mild lymphadenopathy and
hepatosplenomegaly.
a) What is the diagnosis?
b) Mention two investigations.
c) Mention the peripheral smear findings and draw the diagram.
d) What are the special stains used?myeloperoxidase,[immune
stain-CD13,33,117]
e) What is the WHO classification of this condition?
17
GOVT MEDICAL COLLEGE, TRIVANDRUM
LUMINAIRE-2015
f) What is the revised FAB classification of this condition?

9. 5 year old child presents with fatigue, fever, epistaxis, gum bleeding of 1 week
duration. Clinical examination reveals generalised lymphadenopathy and
hepatosplenomegaly. The child also complains of episodes of headache and vomiting
during this 1 week period.
a) What is the diagnosis?
b) Name two investigations
c) Describe the blood picture
d) Which special stain will you use?PAS
e) Which immunostain can be used?Blymphocyte-CD10,19..T lymph-CD2,3
f) Classify this condition.
g) What is the pathogenesis?

10.52 year old male presented with fatigue, weight loss anorexia and left upper quadrant
pain. O/E massive splenomegaly, hepatomegaly and mild lymphadenopathy.
a) What is the diagnosis?
b) What are the investigations?
c) Describe the natural history of the condition.
d) Describe the blood picture with a diagram.
e) What is the bone marrow picture?
f) What is the characteristic molecular abnormality associated with this
condition?
g) What are the differential diagnoses?

11.50 year old male is admitted with weakness and lower back ache. His ESR value is 200
mm/1st hr.
a) What is the probable clinical diagnosis?
b) Mention three lab investigations and their findings which will help in
diagnosis
c) Name the organ that is commonly involved as a complication of this
condition.
d) How will you confirm the diagnosis?
e) Mention two urinary findings(BJ protein detection,increase uric acid)

12.15 year old male presented with painful swelling of bony consistency above the knee.
18
GOVT MEDICAL COLLEGE, TRIVANDRUM
LUMINAIRE-2015
Xray revealed a metaphyseal lesion with characteristic sun-ray spicule appearance.
a) What is the probable diagnosis?
b) What are the common sites affected and the common age group involved?
c) Describe the morphology and histology of the lesion.
d) What is the radiological appearance?
e) Common sites of metastasis.(lung,bones,pleura,heart)

13.45 year old female came with h/o bleeding and foul smelling discharge PV. O/E
patient is emaciated, cervix bleeds to touch.
a) What is the provisional diagnosis?
b) Describe the pathogenesis.
c) How will you stage this lesion?
d) What investigation would have been helpful in detecting this lesion early?
e) What is the morphology of this lesion?
f) What are the risk factors associated with this condition?

14.47 year old lady presents with a hard lump in the breast,fixed to the skin and axillary
lymphadenopathy
a) What is your diagnosis?
b) Which pre operative investigation will prove the diagnosis?
c) What is the aetiopathogenesis of this condition?
d) What are the hisological types of this lesion.Name the histological type
with
i) lymphoplasmacytic infiltrate
ii) peculiar pattern of metastasis
e) What is the likely gross and microscopic picture?
f) What are the prognostic factors of this condition?
g) What is the staging of this condition?

15.5 year old boy presented with painful swelling in the diaphysial region of femur. Xray
revealed a “onion peel appearance” at shaft of femur.
a) What is the diagnosis?
b) What is the reason for the typical radiological appearance of the lesion?
c) What is the chromosomal abnormality commonly associated with this
condition?
d) Name another condition with similar histology.
19
GOVT MEDICAL COLLEGE, TRIVANDRUM
LUMINAIRE-2015
e) Describe the gross and microscopy of the lesion.

16.65 year old male, c/c smoker, complains of cough, hemoptysis. Xray chest shows
opacity in left lower lobe. CT shows solid mass lesion.
a) What is the diagnosis?
b) What are the 2 additional investigations to confirm your diagnosis?
c) What are the four histological types?
d) Describe the pathological features of each type.

17. 12 year old female child presented with puffiness of face and oliguria. Urine
examination showed massive proteinuria.
a) What is the provisional diagnosis?
b) Mention the test for confirming the diagnosis.
c) Mention the principle and procedure of the test for proteinuria.
d) Mention the causes of proteinuria

18.52 year old businessman was admitted in the hospital with severe hematemesis. He
was a known alcoholic for the past 20 years. O/E he was anemic, ascites present, liver
enlarged.
a) What is the probable diagnosis?
b) Describe the pathology of the condition.
c) What are the complications of this condition?
d) What is the cause of ascites and hepatic enlargement in this case?
e) Mention 4 pathognomonic microscopic changes in this condition.
f) Mention 1 condition occuring in children below 3 years where similar
morphological changes are seen (Wilson’s disease)

19.48 year old male was admitted to the hospital because of hematemesis. O/E he had
jaundice, ascites, splenomegaly and nodular hepatomegaly. One nodule was large.
a) What is the most likely diagnosis?
b) Mention 4 investigations you will do for this patient.
c) Describe the pathogenesis of this condition.
d) FNAC was unsuccessful. Hence a liver biopsy was done.
i)Mention why this was done
ii)What is the histopatholgy likely to be in this patient.
e) Mention 1 late sequelae that often occurs in such patients.
20
GOVT MEDICAL COLLEGE, TRIVANDRUM
LUMINAIRE-2015

20.6 year old female child presented with fever, puffiness of face and decreased urine
output for two days. The child gives a h/o sore throat 3 weeks back.
a) What is the provisional diagnosis?
b) What is the etiology?
c) What investigations will you do?
d) Mention 3 urinary findings.
e) Mention 3 tests to be done in this patient to find out the urinary
abnormality.
f) Mention imp light microscopic features in renal biopsy of this case.

21.52 year old hypertensive male complains of sudden onset of severe retrosternal chest
pain and sweating. O/E rapid and feeble pulse with cold extremities.
a) What is your diagnosis?
b) Describe the gross morphological changes in this condition
c) What are the microscopic changes in this condition.
d) What are the markers for diagnosis?
e) What are the complications?

22.10 year old boy complained of fever, joint pains and swelling in several large joints.
a) What is your provisional diagnosis?
b) How will you confirm the diagnosis?
c) Describe the etiopathogenesis of this condition.
d) Describe the gross and microscopic features of the heart in this condition.

23.80 year old male presented with pathological fractures. Xray showed osteoblastic and
metastatic deposits in the bone.
a) What is the most likely primary malignancy?
b) Which clinical investigations will guide to confirm your diagnosis?
c) Name the tumor markers associated with the malignancy and its
importance in clinical evaluation.
d) What is the staging system for this malignancy?
e) Describe the gross and microscopy.
f) What are the hormonal, genetic and environmental factors for this
condition

21
GOVT MEDICAL COLLEGE, TRIVANDRUM
LUMINAIRE-2015
24.65 year old male presents with fatigue, anorexia, weight loss and passing fresh blood
from rectum and crampy left lower quadrant discomfort. O/E he was anemic.
a) What is your provisional diagnosis?
b) What are the risk factors for this lesion?
c) What relevant investigations will you do?
d) What is the pathogenesis of this condition?
e) Describe the morphology and histopathology of this condition.
f) What are the tumor markers associated with this condition?
g) What is the TNM classification of this condition?

25.48 year old male presented with dyspepsia, hard palpable mass in the epigastrium and
hard fixed palpable supraclavicular node.
a) What is the diagnosis?
b) How will you confirm your diagnosis?
c) What is the WHO histological classification of this condition?
d) Mention the risk factors of this condition.
e) Describe in detail the morphology of this lesion.

26.20 year old male presented with fever, jaundice, vomiting of 10 days duration. There is
tender liver on palpation.
a) What is the diagnosis?
b) What are the probable values of LFT?
c) Describe the morphology of liver in this condition.

27.50 year old aniline dye worker attends the OPD with complaints of painless hematuria
of 1 month duration. He was emaciated.
a) What is your provisional diagnosis?
b) What relevant investigations will you do?
c) What is the histological type of this lesion?
d) What are the morphological patterns seen in this condition.
e) Describe the grading of this condition.

28.A 3 year old boy presented with severe anemia and failure to thrive. His frontal bones
were prominent, Hb- 4gm%, TC-10,000/cu.mm
a) What is the clinical diagnosis?
b) Give two investigations to confirm the diagnosis?
22
GOVT MEDICAL COLLEGE, TRIVANDRUM
LUMINAIRE-2015
c) Describe the blood picture.
d) Pathogenesis

29.A 43 yr old man presents with c/c cough 2 months duration ,evening rise of
temp,weight loss and occasional blood staining of sputum. x-ray revealed a patchy
opacity at the apex of the right lung.
a) What is the clinical diagnosis?
b) How will you classify this condition?
c) What are the primary sites of this lesion? (Lungs, intestine, tonsil, skin,
conjunctiva)
d) Name the best specimen for culture.
e) Describe the pathogenesis of this condition.

30.65 year old female came to the medical OPD with complaints of macroglosia of 2 years
duration and slurring of speech. O/E Pallor (+). No organomegaly or lymphadenopathy.
ESR – 100 mm in 1st hour.
Biopsy from tongue- Microscopy showed irregular, homogenous pink material
deposited in the sub epithelial region and vessel wall. Bone marrow trephine biopsy –
Plasma cells 5%
a) What is the provisional diagnosis?
b) How will you confirm the diagnosis?
c) Classify the diseases
d) Describe the etiopathogenesis of the commonest type seen in developing
countries.

GROSS SPECIMENS
1) GANGRENE FOOT
• Amputated specimen of foot showing dry shrunken, shrivelled and dark appearance of
big toe with marked demarcation between gangrenous and normal areas.
• Gangrene- massive necrosis with super added putrefaction.
• Types, causes,diff b/w wet and dry gangrene

2) CALCINOSIS CUTIS
• C/S showing skin with underlying patches of chalky white deposits.
• 2 types of pathological calcification: dystrophic and metastatic
• Dystrophic - dead and dying tissues. No hypercalcaemia. E.g.; in atherosclerotic
23
GOVT MEDICAL COLLEGE, TRIVANDRUM
LUMINAIRE-2015
patches
• Special stains- vonkossa, Alizarin red, osmic acid stain
• Psammoma bodies - spherules of calcification in some tumors like meningoma,
papillary serous cystadenocarcionma of ovary and papillary serous carcinoma of
thyroid.
• Metastatic; occur in association with hypercalcaemia. Calcium gets deposited in living
and healthy tissue, sites: kidney, stomach, lungs, BV,cornea

3) ANEURYSM AORTA With THROMBUS


• Specimen heart with dialated aorta. Cut section showing brownish thrombus filling
lumen.
• Aneurysm is defined as any permanent abnormal localized dilation of a blood
vessel or heart
• Sites: large arteries (esp. abdominal aorta below renal artery and above bifurcation)
• Complications: rupture & fatal hemorrhage, compression of adjacent structures
(ureter, vertebral bodies), thrombo embolism, distal ischaemia
• Disorders associated with aneurysm – Atherosclerosis > Hypertension
• Syndromes associated with aneurysm- Marfans syndrome, EhlerDanlos syndrome

4) ATHEROMA AORTA.
• Cut opened specimen of a portion of aorta, internal surface showing raised yellowish
white plaques and ulcerated areas.
• Common sites – Abdominal aorta> thoracic aorta> coronary arteries> popliteal artery>
internal carotid artery> vessels of circle of Willis
• Fatty dots &streaks – Earliest change
• Atherosclerotic plaque components – Fibrous cap, Cellular area, Soft core
• Risk Factors of atherosclerosis- Hypertension, Dyslipidemia, Increasing age, male
sex, Lipoprotein A levels
• Complications: Erosion,Ulceration, Rupture, Hemorrhage into plaque ,Aneurysmal
dilation,Thrombosis
• Consequences: MI, cerebral infarction, Aortic aneurysm, peripheral vascular disease

5) MURAL THROMBUS - HEART:


• Cut open heart showing solid mass of thrombus attached to endocardial surface near
apex of ventricle.
• Thrombosis is defined as the formation of clotted mass of blood within the intact CVS
24
GOVT MEDICAL COLLEGE, TRIVANDRUM
LUMINAIRE-2015
during life.
• Pathogenesis: Virchow’s triad, Lines of Zahn
• Embolus: An intravascular solid, liquid or gaseous mass
• Types of thrombi: mural, arterial &venous (Diff. b/w arterial & venous thrombi)
• Fate: Propagation, Embolisation, Dissolution, Organization, Recanalisation.

6) LYMPHOMA
• 2 types - Hodgkin’s disease &non Hodgkin’s lymphoma
• Hodgkin’s –localized, extra nodal involvement rare. Ann Arbor staging
• RS cell-types (Popcorn – Lymphocyte predominant, Lacunar – Nodular Sclerosis,
• NHL- involves multiple nodes, extra nodal involvement –common

7) FIBRINOUS PERICARDITIS (White heart)


• Specimen of heart shows white flaky appearance.
• Fibrinous exudate is formed on the surface of the 2 layers of pericardium and gives a “
bread and butter appearance” when the two layers are seperated.
• Causes - a/c MI, Rheumatic fever, Dressler’s syndrome, uraemia, chest radiation, SLE,

8) INFARCTION SPLEEN
• Specimen showing cut section of spleen with wedge shaped pale infarct.
• Causesof splenic infarct- thrombo embolism arising in the heart, obstruction of
microcirculation (e.g. in myeloproliferative diseases, sickle cell anaemia, arteritis,
Hodgkin’s disease)
• Hereditary disease in which splenic infarction is seen - sickle cell anemia
(auto-splenectomy)
• Infarction - ischaemic tissue necrosis caused by occlusion of either arterial supply or
venous drainage in a particular tissue.
• Types: according to colour—
-white/anemic- spleen, kidney, heart
-red/h’ gic-intestine, ovarian torsion, lung
-according to presence or absence of infection - bland and septic.
• Type of necrosis seen in infarct- coagulative (but in brain - liquefactive)
• Causes of infarction: interrupted blood supply(ischemic );venous(stagnant hypoxia);
thrombosis; embolism; non occlusive circulatory insufficiency.

9) AMYLOID SPLEEN
25
GOVT MEDICAL COLLEGE, TRIVANDRUM
LUMINAIRE-2015
• Types of amyloid spleen:
• 1. sago - amyloid deposition limited to splenic follicles resembling sago grains.
• 2. Lardaceous: deposition in splenic sinuses and connective tissue in red pulp, fusion
of deposits gives rise to map like areas of amyloid.
• Amyloid liver – Specimen of liver showing pale waxy grey appearance (deposition
starts in the Space of Disse)
• Special stains (microscopy) – Congo red, Thioflavin T;
• Gross – Lugol Iodine -mahagony brown colour,Sulphuric acid - Blue

10)CVC SPLEEN
• Specimen showing C/S of enlarged, tense and cyanotic spleen.
• Etiology: RHF, portal hypertension, from liver cirrhosis.
• Histological features: sinuses are dilated and congested, foci of recent h’age
• Gamnagandy bodies (Siderofibrotic nodules) - scar tissue laden with Ca,Fe.
• Special stains: Prussian and Vonkossa

11)LOBAR PNEUMONIA
• Specimen of lung showing widespread fibrinosuppurative consolidation, affected part
pale with greyish homogenous dry surface
• Etiology: staphylococcus, pneumococcus, klebsiella.
• Stages:
1. stage of congestion
2. Stage of red hepatisation
3. Stage of grey hepatisation
4. Stage of resolution
• Complications: lung abscess, partial healing, organization, empyema, bacteraemic
dissemination to pericardium, brain, kidneys, heart valves.
• Carnification: Post pneumonic fibrotic reorganisation

12)PULMONARY EMBOLISM
• Occlusion of pulmonary tree by thromboembolism
• Causes:
1. thrombi originating from large veins from lower legs,
2. thrombi in varicosities in superficial veins of leg and pelvic veins
• Consequences: sudden death, corpulmonale, pulmonary infarction, pulmonary h’ge,
resolution, pulmonary hypertension, pulmonary atherosclerosis.
26
GOVT MEDICAL COLLEGE, TRIVANDRUM
LUMINAIRE-2015
• Paradoxical embolism; may occur by passage of an embolus from right heart to left
heart through atrial or ventricular septal defect. In this way pulmonary emboli may
reach systemic circulation.

13)FIBROCASEOUS TUBERCULOSIS:
• Pneumonectomy specimen with destruction of lung parenchyma producing multiple
irregular cavities, lined by caseous material.
• 5 primary sites of TB – lung, intestine, tonsil, conjunctiva, skin.
• Consequences of progressive pulmonary TB – fibrocaseous cavity, miliary TB,
tuberculous bronchopneumonia.
• Complications: dissemination, hemorrhage, Amyloidosis.
• Lung parenchyma destroyed, dystrophic calcification maybe seen. Right lung
commonly affected. Cavity formed as a result of caseous necrosis.
• Cavitary lesions of the lung: pulmonary TB, lung abscess, bronchiectasis,
bronchogenic carcinoma, congenital cysts of lung. Hydatid cyst.
• How to distinguish between lung abscess and other cavitary lesions? -Lung abscess is
a localized area of lung tissue with suppuration. There will be air fluid level also within
the abscess cavity due to pus collection.

14)CASEATING TB LYMPHADENITIS:
• Cut section of enlarged lymph node, capsule is thickened, c/s showing multiple grey
white areas with necrosis at the centre.
• Diff from Hodgkin’s lymphoma: TB-matted, firm nodules with skin involvement. HL
-discrete, knotty nodules with no skin involvement
• Matting of nodules due to PERIADENITIS
• Complications – cold abscess, c/c non healing sinus, disseminated TB.
• Investigations - CXR, FNAC lymphnode, Mantoux test, TB PCR
• Common lymph node groups affected- Hilar, Cervical, Mesentric

15)LUNG ABSCESS:
• Specimen of lung showing an abscess cavity with shaggy wall and fibrosis
• It is a local suppurative process with necrosis of lung tissue.
• Organisms causing- Streptococci, staphylococci, other gram -ve organisms.
• Complication: Amyloidosis, extension to pleura, dissemination.

16)BRONCHOGENIC CARCINOMA LUNG:


27
GOVT MEDICAL COLLEGE, TRIVANDRUM
LUMINAIRE-2015
• Pneumonectomy specimen- greyish white, poorly circumscribed firm to hard lesion
,with focal areas of h’ge and necrosis.
• Etiology.- smoking, atmospheric pollution, asbestosis, dietary and genetic factors,
chronic scarring
• Age group- 40- 70
• Metastasis: direct, lymphatic and hematogenic spread, prognosis generally poor,
• Histological types: squamous cell carcinoma(25-40% in smokers),
adenocarcinoma(25-40% most common in women and non smokers), small cell
carcinoma(20-25%), large cell carcinoma(10-15%)
• Paraneoplastic syndromes:
1. ectopic hormone production. (Cushing’s syndrome, hyponatremia,
hypercalcaemia, gynaecomastia, carcinoid syndrome)
2. other systemic manifestations (polymyositis, myopathy, clubbing,
acanthosisnigricans, trousseau’s syndrome)
• Sites of metastasis- adrenals(>50%), liver, brain, bone
• Worst prognosis - Small cell carcinoma
• Best response to chemo & radiotherapy – Small cell ca

17)HYDATID CYST:
• Cystic space seen in lung, inner surface is smooth and contains a whitish translucent
membranous material. - Tender coconut appearance.
• Etiology- Echinococcusgranulosus (dog tape worm)
• Other sites: liver, kidney, eye, brain, bone, spleen
• Complications: anaphylaxis on cyst rupture.
• Diagnosis by Ultrasound, IgM ELISA test, Casoni’s intradermal test

18)LIPOMA
• Well encapsulated, round to oval mass, C/S shows smooth, yellowish, greasy
appearance
• Most common soft tissue tumor of adulthood
• Classified based on particular morphological features like conventional lipoma,
fibrolipoma, angiolipoma, myelolipoma, spindle-cell lipoma etc.
• Special Stains - Sudan III, Sudan IV, Sudan black, oil red O, Osmic acid.

19)RETINOBLASTOMA
• C/S showing eye showing grayish white mass within the retina, lesion could be partly
28
GOVT MEDICAL COLLEGE, TRIVANDRUM
LUMINAIRE-2015
necrotic, partly solid (Tumor may be endophytic or exophytic)
• Most common intra-ocular malignancy of children.
• 60% sporadic, 40% familial
• Knudson’s two hit hypothesis
• Cell of origin: neuroepithelial cells in posterior retina.
• Types of rosettes: Flexner- Wintersteiner, Homer Wright
• Other rosettes showing tumors: neuroblastoma, medulloblastoma
• Types of RB: autosomal dominant and autosomal recessive
• Increased risk of developing osteosarcoma
• Trilateral retinoblastoma – Bilateral retinoblastoma + Pinealoblastoma
• Spread – locally (vitreous, retina, optic nerve, brain), blood (bone marrow)

20) TUBERCULOUS ULCER-INTESTINE


• Section of intestine showing narrowing and stricture formation in the intestine with a
dilated proximal segment. Mucosa shaggy and ulcerated.Lossof normal mucosal
folds.
• Lesion begin in the Peyer’s patches or lymphoid follicles with formation of small ulcers
that spread through the lymphatics to form large ulcers which are transverse to the
long axis of the bowel. The ulcers may be coated with caseous materials. Serosa may
be studded with visible tubercles.
• Complication: transverse fibrous strictures and intestinal obstruction.
• Causes of ulcer intestine-TB, typhoid, yersinia, peptic ulcer, malignancy, carcinoid,
amoebiasis

21) TYPHOID ULCER-INTESTINE


• Segment of resected small bowel showing thinned out mucosa with surrounding
mucosal folds.
• Ulcer-longitudinal
• Complication: perforation of ulcer, haemorrhage.

22) AMOEBIC ULCER


• Segment of colon showing multiple ulcers on the mucosal surface. The ulcers have
overhanging ragged edges and are covered with necrotic tissue. Mucosa between the
ulcers is not involved.
• Early intestinal lesions appear as small area of elevation on the mucosal surface. In
advanced cases typical flask shaped ulcers having narrow neck and broad base are
29
GOVT MEDICAL COLLEGE, TRIVANDRUM
LUMINAIRE-2015
seen.
• Most common site- caecum
• Complication: amoebic liver absecess, perforation, haemorrhage, and formation of
amoeboma.
• Diagnosis – demonstration of trophozoite/cyst in stool.
• Special Stain – Gomorri-Trichrome, Fe- hematoxylin stain to demonstrate amoeba.

23) GANGRENE INTESTINE


• Coiled loop of intestine with black discolouration, wall oedematous with loss of normal
mucosal folds, surface is shaggy.
• Causes- occlusion of superior mesenteric artery, other causes: volvulus,
intussusception, inferior mesenteric vein thrombus.
• Why does haemorrhagic infarction occur in the intestine? = Intestine is a soft tissue
and it has dual circulation.

24) GASTRIC ULCER


• Specimen of cut section of stomach showing solitary, small round to oval, punched out
lesion with clear base and mucosal folds converging towards the ulcer.
• Aetiology: disruption of the mucosal barrier, gastritis, bile reflux, alcohol, tobacco.
• Sites :first part of duodenum, pyloric antrum, gastroeosophageal junction in GERD or
Barret esophagus, jejunum in Zollinger Ellison syndrome( in addition to duodenum &
stomach), Meckel’s diverticulum.
• Complication: perforation, h’age, obstruction, malignant transformation.
• Histology from within outwards:
• Zone of necrotic debris
• Zone of non specific inflammation
• Granulation tissue zone
• Zone of fibrosis

25) CARCINOMA STOMACH


• Gastrectomy specimen showing an ulcero-proliferative growth, ill defined border with
irregular papillary projections.Floor is shaggy and necrotic.
• Risk factors: H. pylori infections, c/c gastritis, gastric adenomas, pernicious anemia.
• Early gastric Ca – confined to mucosa & sub-mucosa with or without lymph node
involvement
• Advanced gastric Ca.,Linitis plastic (leather bottle stomach), Krukenberg Tr., Sister
30
GOVT MEDICAL COLLEGE, TRIVANDRUM
LUMINAIRE-2015
Mary Joseph Nodule-???
• Lauren’s classification (Intestinal & Diffuse); WHO histological classification-??

26) INTUSSUSCEPTION
• Specimen showing cut open section of portion of intestine with telescoping of a
segment of intestine into the segment below. It occurs due to peristalsis of the
telescoped segment. The telescoped segment is called intussusceptum& the lower
receiving part is called the intussuscepiens.
• Causes: foreign bodies & tumors [adults], enlargement of lymphoid tissue in terminal
ileum mostly due to viral infection [children]
• Types: ileocolic ,ileoileocolic, colocolic
• Complications: obstruction, infarction, gangrene, perforation, peritonitis
• Signs & symptoms- Mass in the abdomen, bleeding PR, other symptoms of a/c
obstruction

27) FAMILIAL POLYPOSIS COLI


• Gross: Cut open segment of colon with the lumen showing homogenous brownish and
yellowish areas studded with multiple polyps
• APC gene mutation, chromosome 5q21
• Risk of early development of Ca. Colon
• Prophylactic colectomy advisable
• C/S: Homogenous, brownish, & yellowish.
• Types: Non neoplastic :Hamartomatous, Inflammatory, Lymphoid, Hyperplastic –
Neoplastic: Benign adenomas and malignant polypoid Ca.

28) ACUTE APPENDICITIS


• Specimen of appendix showing oedematous walls, ulcerated mucosa and blackish
exudates on surface.
• Stages: early acute, acute suppurative, acute gangrenous
• Causes: obstructive (fecolith, calculi, foreign bodies, tumor, worms), non-obstructive
(vascular occlusion, diet)
• Pathogenesis: Obstruction-- Continued mucus secretion--Increased intraluminal
pressure-- Collapse of draining veins-- Ischaemic injury-- Bacterial
proliferation--Inflammatory edema and exudation
• Complications: peritonitis, appendicular abscess, adhesions, mucocele.
• C/F – fever, vomiting, right sided abdominal pain and tenderness (at Mc Burney’s
31
GOVT MEDICAL COLLEGE, TRIVANDRUM
LUMINAIRE-2015
point)

29) HEMOCHROMATOSIS LIVER


• Cut section of liver, one half shows homogenous brownish appearance, other half
stained with Pearl’s stain
• Hemochromatosis is a condition in which systemic overload of iron in the form of the
hemosiderin pigment occurs
• Types: Primary [hereditary], Secondary[acquired]-due to thalassemia, sideroblastic
anemia, multiple transfusion, alcoholic cirrhosis.
• Sites : skin, liver, pancreas, heart, pituitary, joints (testicular atrophy occurs d/t
pituitary involvement)

30) FATTY LIVER


• Cut section of enlarged liver, bright yellow colour and soft greasy feel.
• Fatty change (Steatosis) occurs due to abnormal accumulation of triglycerides within
parenchymal cells.
• Special stains: Sudan III, Sudan IV, Oil red O, Osmic acid.
• Other organs affected- heart, skeletal muscle, kidney
• Causes: Alcohol, CCl4, OCPs, Obesity, Bacterial toxins

31) CIRRHOSIS LIVER


• Specimen of portion of liver having surface studded with numerous nodules. Cut
surface is nodular and greasy with firm consistency( due to fibrosis)
• Cirrhosis is the end stage of c/c liver disease characterised by :
a. Bridging fibrous septae
b. Parenchymal nodules encircled by fibrosis
c. Disruption of liver architecture
• Types:Micronodular, Macronodular, mixed
• Etiology: Alcoholic liver disease, viral hepatitis, biliary disease, wilsons disease.
• Complications: portal hypertension, hepatic failure, hepatocellular Ca

32) CAVERNOUS ANGIOMA LIVER


• Section of liver with fairly well demarcated blackish discolouration occupying the one
region.
• Types of hemangiomas – Cavernous, capillary, pyogenic granuloma
• Other sites – skeletal muscles, bones, lips, tongue, face,brain
32
GOVT MEDICAL COLLEGE, TRIVANDRUM
LUMINAIRE-2015
• Malignant counterpart - Hemangiosarcoma

33) AMOEBIC LIVER ABSCESS


• Section of liver shows large cystic cavity with irregular thickened shaggy inner lining,
normal compressed tissue at the periphery.
• Cause: Entamoebahistolytica
• Special stain: PAS, Gomorritrichrome, Fe hematoxylin

34) CARCINOMA LIVER


• Section of liver showing fairly well circumscribed grey white mass, periphery shows
compressed liver tissue.
• Etiology: HBV infection, c/c alcoholism, food contaminants (aflatoxin).
• Morphological types: Unifocal, multifocal, diffusely infiltrating
• Tumourmarkers : AFP, Des-gammacarboxyprothrombin, Alk. PO4ase
• Variant: fibrolamellar carcinoma which occurs in the absence of cirrhosis or HBV
infection, has good prognosis.

35) GALL STONES


• Types: cholesterol, pigment stones
• Pathogenesis: Cholesterol super-saturation in bile and enucleation (cholesterol
stones); Increased bilirubin forming calcium salts (pigment stones)
• Risk Factors- Female gender, OCPs, pregnancy, obesity, biliary infection
• Complication: cholecystitis, gall stone ileus, biliary fistula, mucocoel, gallblader Ca.

36) C/C CHOLECYSTITIS


• One half of cholecystectomy specimen cut opened, wall thickened, lumen dilated, and
shows two calculi.
• Mucosa shows brownish discolouration, shaggy serosa.
• Complication: obstructive jaundice, cholecystitis, biliary cirrhosis, cholangitis,
mucocele.

37) GRANULAR CONTRACTED KIDNEY


• Bisected kidney. Capsule from surface is peeled off. Exposed cortical surface shows
fine granularity. Kidney as such is decreased in size. Cut surface shows decreased
size of renal cortex and slight dilatation of calyces.
• Causes: c/c glomerulonephritis, c/c pyelonephritis, diabetic nephropathy,
33
GOVT MEDICAL COLLEGE, TRIVANDRUM
LUMINAIRE-2015
nephrosclerosis

38) PYELONEPHRITIS
• Section of kidney, capsule adherent. C/S shows yellowish black areas with irregular
scarring and distended calyces with focal abscess
• Types: Acute and chronic.
• Causes: UTI [E.coli, Enterobacter, Klebsiella, Proteus], obstructive pyelonephritis
• Complications: papillary necrosis, pyonephrosis, perinephric abscess.
• Variant of c/c pyelonephritis- xanthogranulomatous pyelonephritis.

39) HYDRONEPHROSIS
• Cut opened kidney which is enlarged and having dilated pelvis and calyces with
thinned out renal cortex.
• Cause- Obstruction to outflow of urine, eg: stricture,calculi, prostatic hypertrophy,
tumors
• Complications: glomerular atrophy, renal papillary necrosis, pyelonephrits.

40) POLYCYSTIC KIDNEY


• C/S shows multiple cysts of varying size
• Inheritance type: adult type- autosomal dominant; infantile- autosomal recessive
• The cyst do not communicate with the pelvis of kidney- a feature that helps to
distinguish polycystic kidney from hydronephrotic kidney on sectioned surface.
• Associated lesions: cysts in liver and pancreas; colonic diverticula, berry aneurysm,
MVP (in adult type)
• Complication: nephrosclerosis, fibrosis and c/c inflammation.

41) CALCULUS KIDNEY


• Cut opened kidney, showing calculi occupying pelvis, surface has brownish granules,
cortex grossly granular.
• Types: Calcium stones[75%], mixed stones[struvite]-l5%, Uric acid-6%, Cystine-2%.
• Struvite stones are formed after infection by urea splitting bacteria
• Complications: Hydronephrosis and hematuria.
• C/F: renal colic, hematuria,

42) STAGHORN CALCULUS (WITH HYDRONEPHROSIS)


• Cut opened kidney with dilated calyces and pelvis, a large brown white calculus
34
GOVT MEDICAL COLLEGE, TRIVANDRUM
LUMINAIRE-2015
creating a cast of pelvicalyceal system.
• Composition: magnesium ammonium calcium phosphate[triple phosphate stones]
• Etiology, urinary tract infections with urea splitting organisms like proteus.

43) NEPHROBLASTOMA
• Specimen of kidney showing a solitary well circumscribed mass.Cut surface is
soft,homogenous and tan to grey with occasional foci of haemorrhage,cyst formation
and necrosis
• lt is the most common primary renal tumor of childhood.
• Classical triphasic histology – blastemal, stromal & epithelial cell types.
• Syndromes associated – WAGR syndrome, Denys-Drash syndrome, Beckwith
Weidmann syndrome, Miller syndrome
• C/F-abdominal mass,hematuria,pain in abdomen,intestinal obstruction

44) CARCINOMA KIDNEY (RENAL CELL CARCINOMA/GRAWITZ TUMOUR)


• C/S of kidney showing spherical masses of bright yellow/grayish white tissue that
distorts renal outline with foci of h’age, necrosis and areas of softening (variegated
appearance)
• Causes: cigarette smoking, obesity, hypertension, long term dialysis and family history
(von Hippellindau syndrome, hereditary papillary ca.)
• Histo types: clear cell (most common), papillary, chromophobe, collecting duct.
• 3 classical features – costovertebral pain, palpable mass and hematuria
• Spread: hematogenous and local.
• Paraneoplastic syndromes: polycythemia, hypercalcemia, hypertension, feminization,
masculinization

45) CARCINOMA PENIS


• Specimen of Penis with scrotum, distal end of penis shows exophytic growth
encircling the shaft.
• Premalignant lesions: Bowens disease, bowenoidpapulosis.
• Etiological factors: Unidentified carcinogens contained in smegma, HPV 16, 18
• Commonest type: squamous cell carcinoma.

46) SEMINOMA TESTIS


• Enlarged testis, C/S shows homogenous grey white lobulated tissue; h’age, necrosis
not seen.
35
GOVT MEDICAL COLLEGE, TRIVANDRUM
LUMINAIRE-2015
• Commonest germcelltumour of testis and corresponds to dysgerminoma of ovary. (
peak incidence in 40’s)
• Tumour markers: AFP, HCG.
• Tumour cells +ve for PAS due to glycogen.
• Sheets of uniform cells divided into lobules by delicate fibrous septa
• Types: Typical( 85%), Anaplastic(5-10%), Spermatocytic.

47) ENDOMETRIAL CARCINOMA


• Gross: 2 patterns – diffuse(more common) or polypoid, tumor protrudes into
endometrial cavity as irregular friable grey tan mass
• Extension into myometrium maybe seen as soft, friable granular tissue on c/s
• Peak age: 55 – 65
• Risk factors : DM, HTN, Obesity, Infertility
• C/F: irregular vaginal bleed, excessive leucorrhea
• Diagnosis by curettage and histological examination

48) CARCINOMA CERVIX.


• Section of uterus, both lips of Cervix shows greyish white necrotic growth.
• Risk factors: early age sexual activity,high risk male sexual partner such as
promiscous male having history of penile condyloma, multiparity,
venerallytransmissable infections.
• Most common histological type – squamous cell carcinoma
• Morphologoical Types: fungating, ulcerative and invasive.
• Screening – Pap smear
• Types of HPV producing Ca Cervix: low risk-6 & 11; high risk-16; 18,31

49) TERATOMA
• Teratoma are tumors composed of different types of tissues derived from three germ
layers - ectoderm, mesoderm, endoderm in different combinations.
• Types: Mature(benign), Immature(malignant), monodermal or highly
specialisedteratomas.
• Benign Cystic Teratoma(Dermoid Cyst) - unilocular cyst lined by skin
• C/S-Unilocular Cyst filled with sebaceous secretions & desquamated keratin admixed
with masses of hair, tissue elements such as tooth, bone, cartilage. Cyst wall is thin
and opaque grey white. In one area of the cyst wall, a solid protuberance (Rokitansky’s
protuberance) maybe seen.
36
GOVT MEDICAL COLLEGE, TRIVANDRUM
LUMINAIRE-2015
• Other sites: Ovary, testes, mediastinum, retro peritoneum, sacrococcygeal region.
• Classification of Ovarian Tumors

50) ADENOMYOSIS
• Gross: Enlarged uterus showing diffuse thickening of uterine wall sometimes with
areas of reddish brown discolouration within the myometrium
• Presence of endometrial tissue in the myometrium
• C/F: dysmenorrhoea, abnormal bleeding, menorrhagia

51) LEIOMYOMA UTERUS


• Gross: distorted specimen of uterus, with circumscribed discrete round firm grey white
mass, in a whorled pattern.
• Leiomyomas/Fibroids-common uterine tumor of smooth muscle origin.
• Cause: estrogen, human growth hormone, Infertility, estrogen dependent tmr.
• Types: subserosal, submucosal, intramural
• Other sites- esophagus, erector pili muscles of skin , nipple, scrotum, labia
• 2° Changes- cystic change, myxomatous change, pathological calcification, red
degeneration, hyaline change
• Complications: torsion, inversion, capsule rupture, inflammatory changes, association
with endometrial carcinoma.

52) UTERUS W1TH VESICULAR MOLE


• Gross: hysterectomy specimen, Cut open uterus, endometrium dilated & filled with
friable,delicate, thin walled translucent small vesicles forming a cluster(like a bunch of
grapes)
• Significant investigation -urine HCG examination.
• Hydatidiform mole characterised by 2 features-hydropic change of chorionic villi &
trophoblastic proliferation.
• Complications-Trophoid disease, Choriocarcinoma.
• Classification-(l) non-invasive mole-complete& partial (2)invasive mole
(chorioadenomadestruens)

53) CHORIOCARCINOMA-UTERUS
• Gross-Cut section of uterus with cervix .Endometrial cavity filled with necrotic, friable,
blackish appearing growth, infiltrating the myometrium.
• Causes-Hydatidiformmole(50%), normal pregnancy (22%), previous abortion (25%),
37
GOVT MEDICAL COLLEGE, TRIVANDRUM
LUMINAIRE-2015
remainder occur in ectopics and genital teratomas.
• Types-(l) gestational {placental origin} (2)non-gestational (ovarian origin}
• Tumor marker-HCG
• Sites of metastasis-lungs (50%), vagina (40%), brain, liver, kidney

54) FIBROADENOMA
• C/S-well circumscribed grey white spherical / discoid mass with slit like spaces.
• Types-pericanalicular, intracanalicular.
• Most common benign tumor of breast., arises from intra lobular stroma.
• Also called ‘mouse-in-breast’ as its a mobile lump.
• DD’s - c/c abcess, fat necrosis, Ca breast
• Cytosarcomaphyllodes: bulky breast tumor with leaf like gross appearence.
• Phyllode’stumour histologically distinguished fromfibroadenoma by cellularity,
mitotic rate & infiltrative borders.

55) CARCINOMA BREAST WITH SECONDARIES IN LYMPH NODES


• C/S of breast( identify nipple) showing grey white firm to hard granular growth with
irregular borders.
• Types- non-invasive[intra-ductal Ca, Lobular Ca in situ], invasive[invasive ductal,
invasive lobular, medullary Ca, tubular, mucinous, papillary]
• Aetiology: sex, age, genetic (BRCA1, BRCA2), diet, endocrine, geographical, mammary
dysplasia.
• Spread: lymphatic, local, blood stream.
• Lobular breast Ca- Bilateral, Indian file appearance
• ScirrousCa-infiltrating ductal carcinoma-7 5% of all breast cancer, hard in consistency.
• Inflammatory Ca-breast cancers with redness, oedema, tenderness and rapid
enlargement. Associated with excessive invasion of dermal lymphatics; poor
prognosis.
• Paget’s d/s of nipple -rare type of breast Ca, presents as unilateral erythematous
eruption with a scaly crust.
• Clinical staging & prognostic factors

56) MALIGNANT MELANOMA SKIN/EYE/RIB


• Blackish deposit with intervening necrotic areas.
• Arise from melanocytes, Most rapidly spreading malignant tumor of skin
• Aetiology: excessive exposure of sunlight esp. to white skin.
38
GOVT MEDICAL COLLEGE, TRIVANDRUM
LUMINAIRE-2015
• Predisposing factors: naevi, heredity and local host immune response.
• Sites: skin {trunk, legs, face, soles, palms and nail bed), oral and anogenital mucosa,
oesophagus, conjunctiva, uveal epithelium, orbit and leptomeninges.
• Four types of cutaneous melanomas: lentigomalignant melanoma, superficial
spreading melanoma, a lentigenous melanoma, nodular melanoma.
• Histology: Tumor cells resembling epitheliod cells with pleomorphic nuclei and
prominent nucleoli are seen solid masses in the dermis. Multinucleate giant cells and
mitotic figures are often present.
• Special stain – Masson’s Fontana

57) BASAL CELL CARCINOMA


• Gross : Common pattern is nodulo-ulcerative type with a pearly/dark translucent colour
and a network of fiery red blood vessels on the surface, the surface may ulcerate with
melanin pigmentation.
• Sites: upper part of face - cheek, nose and ear. ( above a line joining tip of ear and
angle of mouth)
• Histology: dermis invaded by irregular mass of basaloid cells with characteristic
peripheral palisaded appearence .
• Highly radio sensitive, does not metastasise
• Aetiology - Chronic sun exposure, immunosuppression, defective DNA repair
• Associated with Gorlin’s syndrome (PTCH gene mutation – Nevoid BCC syndrome)

58) SQUAMOUS CELL CARCINOMA FOOT


• Specimen of foot showing greyish white warty growth having cauliflower appearance
• Types : Ulcerating, Fungating, Invasive
• Risk factor: Sun exposure, industrial carcinogens, chronic ulcers, old burns, scars,
AIDS
• Sites: Skin and mucosa of lip, tongue, pharynx, oesophagus, vocal cord, anal region,
vagina, ducts of glands etc

59) MALUNION
• Two pieces of bone covered by muscle fibre; Malunion means that the fracture has
united in a position of deformity with angulation, rotation or over riding of the
fragments, results from imperfect reduction and failure to stabilise a previously
satisfactory reduction.
• Other complications of fractures: fibrous union, pseudoarthrosis, non-union, delayed
39
GOVT MEDICAL COLLEGE, TRIVANDRUM
LUMINAIRE-2015
union.

60) SEQUESTRUM
• A tubular piece of bone, cortex shows marked sclerosis.
• Dead piece of bone formed due to combination of suppurartion and ishaemic injury in
pyogenic osteomyelitis.
• Complication: C/c osteomyelitis, amyloidosis.
• Formation of the reactive new bone as an encasing sheath around the necrosed bone -
Involucrum
• Long continued osteogenesis give rise to dense sclerotic pattern of osteomyelitis
called C/c sclerosing non suppurartive osteomyelitis of Garre.
• Brodie’s abscess: occasionaly a/c osteomyelitis may be contained to localised area
and walled off by reactive bone.

61) OSTEOGENIC SARCOMA


• Fusiform greyish white tumor involving metaphyseal end and destroying surrounding
cortex, epiphysis free, periosteal elevation.
• Most common primary malignant tumor of bone.Characterised by formation of
osteoid, bone or both, directly by sarcoma cells.
• Bimodal age distribution – 10 – 20 yrs and also in elderly
• Cells of origin: primitive osteoblast forming mesenchyme.
• Site: lower end of femur, upper end of tibia, upper end of humerus and femur.
• X ray pattern: Codman’s triangle and sunburst appearence.
• Two main categories: medullary and central, parosteal or cortical osteosarcoma.
• Associated with hereditary retinoblastoma.

62) OSTEOCHONDROMA
• Eccentric epiphyseal expansile growth continuous with the cortex of bone.
• Commonest benign tumor of cartilage, may occur as a solitary, sporadic, exostosis or
multiple hereditary exostosis.
• Sites: metaphysic of long tubular bones - lower femur, upper tibia, upperhumerus.
• Age: late childhood and adolescence
• EXT gene association

63) GIANT CELL TUMOUR OF BONE


• Gross: lower end of femur hemisection, grey white friable, expansile growth involving
40
GOVT MEDICAL COLLEGE, TRIVANDRUM
LUMINAIRE-2015
epiphysis. Thinned out shell of bony cortex.Area of H’ages seen.
• It is benign tumour with malignant potential. (age group – 20 – 40yrs)
• Soap bubble appearance in X-ray.
• Tumor arises from epiphysis of long bone close to the articular cartilage

64) MULTINODULAR GOITRE


• Specimen of enlarged and distorted thyroid showing surface nodules. C/S multiple
nodules of different sizes filled with colloid (brownish). Large nodules may show
cystic changes, scarring &haemorhages.
• Aetiology - Impaired thyroid hormone synthesis leads to compensatory rise in
serum TSH level. This results in compensatory hypertrophy & hyperplasia of
follicular cells leading to enlargement of the gland.
• Plummer’s disease (toxic nodular goitre): similar to grave’s disease but lack
features of ophthalmopathy and dermatopathy.
• Two forms: simple goitre (diffuse nontoxic/colloid) & nodular goitre (MNG
adenomatous goitre)
• Section from thyroid showing uniform enlargement of the gland.. C/S: greyish white,
with faint lobulation showing salivary gland like appearance.

65) HASHIMOTO’S THYROIDITIS


• Diffuse goitrous enlargement of thyroid, lymphocytic infiltration of thyroid,
Occurrence of auto antibodies are 3 principle features of hashimoto’s thyroiditis.
• Also called chronic lymphocytic thyroiditis (age group – 45 -65)
• HLA association-HLA DR5, HLA DR3.
• Microscopy: loss of architecture, follicles atrophied, lymphoid follicles,
degeneration of follicular lining cells.
• Hurthle cells and fibrosis present. Hurthle cells (askanazy/ oxyphil/ oncocytes)
are follicular epithelial cells which undergo metaplastic change- abundant
oxyphilic granular cytoplasm due to large number of mitochondria, large bizarre
nucleus.
• Most common antibody- thyroid microsomal auto antibody. Other autoimmune
d/s associated with H’s thyroiditis- pernicious anemia, SLE, rheumatoid arthritis,
IDDM, Addison’s disease, myasthenia gravis.
• Association with malignant lymphoma: increased risk of developing B-cell non
Hodgkin’s lymphoma,(associated with unregulated helper T cell participation in B
cell synthesis of auto antibodies)
41
GOVT MEDICAL COLLEGE, TRIVANDRUM
LUMINAIRE-2015
• Four type of thyroiditis: hashimotos, riedel’s, sub acute(dequervain’s), chronic
(silent)thyroiditis

66) THYROTOXICOSIS
• Enlarged, homogenous, reddish brown, meaty appearance on fresh sections
• Causes: grave’s disease, toxic MNG, toxic adenoma.
• Jodbasedow thyrotoxicosis: (Jod = iodine) when large doses of iodine is given
therapeutically for hyperplastic endemic goiter, temporary hyperthyroidism and
occasional persistent hyperthyroidism may occur.
• Conditions producing uniform enlargement of thyroid: physiological goitre, colloid
goitre, primary thyrotoxicosis, thyroiditis

67) ADENOMA THYROID


• Encapsulated, usually solitary, well circumscribed nodule with greyish white
appearance.H’age& cyst formation may occur.Normal thyroid tissue seen at one
end.
• Clinically- pressure symptoms, sudden enlargement and pain due to intra
lesionalh’ge.
• Follicular adenoma is the most common type of benign thyroid tumor.
• Features to distinguish from a nodule of MNG: solitary nodule, complete
encapsulation, clearly distinguishable architecture inside and outside the capsule,
compression of the thyroid parenchyma outside the capsule.
• Types: micro follicular(foetal adenoma), normo follicular(simple adenoma), macro
follicular(colloid adenoma), trabecular (embryonal adenoma), Hurthle cell, atypical

68) PLEOMORPHIC ADENOMA


• Section of salivary gland showing a well demarcated, faintly lobulated greywhite
lesion. Myxoid& blue translucent areas are noted at places.
• Pleomorphic adenoma is commonest tumor of salivary gland .Women 40-60
years.
• Benign epithelial tumor arising from mixture of ductal(epithelial) &myoepithelial
cells; hence show both epithelial &mesenchymal differentiation.
• Histology: mixed appearance- epithelial elements in a matrix of mucoid ,myxoid
and chondroid tissue.
• Radiation is a risk factor.
• Ca arising in pleomorphic adenoma - ‘Ca ex pleomorphic adenoma ‘
42
GOVT MEDICAL COLLEGE, TRIVANDRUM
LUMINAIRE-2015
• Salivary gland tumor common in smokers - Warthin’s tumor
• Most common malignant tumor of salivary gland - Mucoepidermoidca
• Salivary gland Ca associated with perineural infiltrates- adenoid cystic Ca

69) SUPPURATIVE MENINGITIS


• Section of brain showing opaque leptomeninges, vessels engorged & prominent
• Organisms causing -
1. Neonates - Ecoli, Group B strep
2. Infants & children - H.influenzae
3. Adolescents & young adults - N. meningitidis
4. Elderly - Strep.Pneumonia

70) DYSGERMINOMA
• Solid mass with cut section showing greyiih white lobulated, soft, fleshy areas
with hemorrhage and necrosis
• Female counterpart of seminoma testis
• Most common malignant ovarian tumor

71) TRANSITIONAL CELL CARCINOMA / CARCINOMA BLADDER


• Cut open bladder showing greyish white granular mass
• 90% bladder ca – transitional cell tumour
• Classification- transitional cell papilloma, transitional cell carcinoma (I,II,III)
• Investigations – cystoscopy, biopsy, urine cytologic examination (markers – CEA,
hyaluronidase, human complement factor – H related protein)
• Schistosomahematobium infestation can predispose to squamous cell ca bladder

HISTOPATHOLOGY
1. Fatty liver
Normal liver cells, central vein, portal triad are seen.
Liver cells with vacuolated cytoplasm showing fatty change and washed out
appearance.

2. .Atheroma
Intima is thickened and show cleft like spaces.
Innermost endothelial surface is damaged and show adhesion of RBCs
Media is thinned out and fibrosed.
43
GOVT MEDICAL COLLEGE, TRIVANDRUM
LUMINAIRE-2015
3. Melanoma
Epidermis of the skin showing pigment layden cells in the basal layer.
The groups of cells extend down into deeper tissue
The cells are polygonal spindle or round and arranged in sheets and
contain pigment.

4. Calcinosis cutis
Epidermis of the skin, blood vessels and skin appendages are seen.
Deep in the dermis there are irregular bluish calcified masses lying in spaces or in
the tissue itself.

5. CVC liver
The central vein is congested.
The sinusoids in the centrizonal region dilated and congested.
The liver cells are compressed and in the periphery of the lobule show
fatty change and brownish pigments scattered in the parenchyma.

6. CVC lung
Thickened alveolar wall and congested capillaries
Heart failure cells or large macrophages filled with brown pigment in the alveolar
wall and in the lumen.

7. Granulation tissue
The spindle shaped fibroblasts and loose oedematousstroma.
The newly formed capillary spaces lined by endothelial cells.
Inflammatory cells chiefly polymorphs, round cells and macrophages.

8. Cirrhosis liver
Loss of normal architecture of liver lobules
Central vein is eccentric.
Liver cells arelarge in size and some are binucleated.
Irregular lobules are separated by dense fibrous septa.
The portal triad show round cell infiltration.

9. .Acute appendicits
The destruction of mucosal lining epithelium in places seen.
44
GOVT MEDICAL COLLEGE, TRIVANDRUM
LUMINAIRE-2015
Acuteinflalnrnatory exudates in the lumen.
Cellular infiltration in the mucosa, muscle coat and serosa.

10.Lobar pneumonia
All alveoli are filled with inflammatory exudates and cells are mainly
polymorphs
Clear spaces are seen between alveolar wall and exudates.

11.Tuberculous lymphadenitis
The thickened capsule at the periphery and the subcapsular lymphoid follicles
The normal architecture of the lymphnode is lost in the centre
It is replaced by large homogenous eosinophilic area of caseation
surrounded by multinucleated Langhans giant cells, epitheloid cells and
round cells.

12.Lepromatous leprosy
The thinned out and atrophied epidermis of the skin seen.
Clear homogenous zone just beneath the epidermis.
The dermis shows diffuse collection foamy macrophages and round cells.

13.Tuberculous Leprosy
• Normal epidermis,
• Dermis shows granulomatous lesions

14.Rhinosporidiosis
The submucosa shows rounded capsulated sporangia of variable size.
The interior of the sporangia shows small rounded spores.

15.Osteochondroma
The lobules of cartilage cells separated by fibrous septa.
The cartilage cells which are arranged in groups or singly are embedded in
a bluish homogeneous matrix.
Areas of calcification may also be seen.

16.Lipoma
Lobules of fat cells separated by fibrous septa.
The intervening fibrous septa show blood vessels and nerve bundles.
45
GOVT MEDICAL COLLEGE, TRIVANDRUM
LUMINAIRE-2015

17..Leiomyoma
The endometrium of the uterus with tubular glands lined by epithelium.
The myometrium showing well demarcated cellular areas composed of
spindle cells running in interlacing bundles.

18.Schwanoma
Thickened nerve bundles.
Groups of spindle cells arranged in a palisading manner separated by
nuclear free zones of eosin stained fibres.

19.Capillaryhemangioma
Benign vascular tumours

GROSS-Nodular, polypoidal/ pedunculated , well circumscribed , red/ blue masses ,


measuring <2cm in diameter

MICRO- small thin walled blood vessels lined by plump epithelial cells divided into
lobules by fibrous tissue septa

20.Fibroadenoma
Proliferated elongated dilated or distorted ducts lined by cuboidal
epithelium and the surrounding stroma shows spindle cells.
Thestroma in places show myxomatous degeneration and round cell
infilteration.

21.Teratoma
Various types of tissues of tridermalorgin.
Lobules of cartilage cells, mucus secreting glands, lobules of fat, hair,
muscle, brain, sebaceous glands etc.

22.Pleomorphic adenoma
The pleomorphic appearance of the neoplasm.
Epithelial cells arranged in sheets, strands and acinar pattern and acini show
colloid like or keratinizing material.
Groups of cartilage like cells arranged in groups or singly in a homogenous
matrix.

46
GOVT MEDICAL COLLEGE, TRIVANDRUM
LUMINAIRE-2015

23.Basal cell carcinoma


The cells are of the basal cell type with scanty basophilic cytoplasm and
hyperchromatic nuclei.
The cells in the periphery shows pallisading arrangement

24.Adenocarcinoma colon
Normal colon mucosa with an ulcerated proliferating growth at one end.
Groups of carcinomatous cells with adenomatous pattern are seen
infiltrating into the deeper tissue.

25.Osteogenic sarcoma
Eosinophilic areas of new bone formation with strands of osteoid tissue.
Spindle cells, round cells and multinucleated cells with areas of
haemorrhage and / necrosis.

26.Giant cell tumor


The characteristic multinucleated giant cells and cellular stroma composed
of spindle cells and oval cells.
Plenty of vascular spaces, areas of haemorrhage, cystic spaces and areas of
degeneration.

27.Hashimto’s thyroiditis
The follicles are atrophied and variable in size.
Someacini are lined by large cells with abundant eosinophilic granular
cytoplasm (Askanazy or Hurthle cells) and the colloid is scanty or absent.

28.Papillary Ca thyroid
Papillae composed of fibrovasular stalk and covered by tumour cells
Thetumour cells have overlapping pale nuclei (ground glass appearance)
Psammoma bodies may be seen.

29.Adenoma thyroid
The capsule at the junction of the adenoma and normal thyroid tissue.
The adenoma is composed of follicles lined by cuboidal or flattened epithelium.

47
GOVT MEDICAL COLLEGE, TRIVANDRUM
LUMINAIRE-2015
30.Squmous cell carcinoma
Section of the tissue showing normal epidermis at one end and a
proliferating growth at the other end.
Keratin pearls can be seen.
Groups of cells are seen infiltrating in the deeper tissue.
The infiltrating cells are polygonal and show hyperchromatic nuclei

31. Renal Cell Carcinoma


• GROSS- usually affects 1 pole of the kidney , fairly circumscribed.Cut
surface(variegated appearance) with yellowish areas , brown areas, white areas
and haemorrhagic red areas.
• MICRO- clear cell-clear cytoplasm due to high content of lipid and glycogen,and
prominent cell border
Granular cell-granular due to increase content of mitochondria
Cells are arranged in sheets,nests,cords,tubular or papillary pattern;separated
by fibrous septa
Little cellular/nuclear pleomorphism or mitosis seen

changes-necrosis,fibrosis,cholesteroldeposit,heamorrhage,calcification,cystic
change.

32.Hodgkin lymphoma
• Reed –sternbergcell:large cell with bilobed nucleus seen as a mirror image of
each other
• Each lobe contain prominent eosinophilic inclusion with clear halo around it.
• Owls-eye appearance;and cytoplasm abundant and eosinophilic
• Classifcn-nodular
sclerosis,mixedCellularity,lymphocyterich,lymphocytedepletion,lymphocyte
predominance
• RS cell positive for CD15,CD30 except in lymphocyte predominance(CD20
+,other markers are –ve)

HEAMATOLOGY SLIDES
1. LE CELL
• Mature granulocyte phagocytose homogenous nuclear material
• Phenomenon due to antinuclear Ab against nucleprotein
• Tart cell = engulfed by monocyte; inclusions not homogenous and will show
chromatin

48
GOVT MEDICAL COLLEGE, TRIVANDRUM
LUMINAIRE-2015
• Seen in – SLE,Multiplemyeloma,Rheumatoidarthritis,Leukemia

2.MEGAKARYOCYTE

• Largest cell in bone marrow(30-100 micron)


• 2 types – inactive and active
• Normal bone marrow contains 1-2 megakaryocytes/HPF
• Multilobulated nucleus
• No. Increase in-megakaryocytic leukemia,polycythemiavera,ITP,Hypersplenism
• No. Decrease in – Toxic bone marrow
depression,Heatstroke,Perniciousanaemia,acuteleukemia,aplasticanaemia

3.NORMOBLAST

• 3 STAGES –early,intermediate,late
• Increases in- hemolytic
anaemia,erythroleukemia,leukoerythroblasticpicture,splenectomy

4.PLASMODIUM VIVAX(TROPHOZOITE STAGE)

• Infected red cell large


• Early trophozoite – signet ring shaped
• Late trophozoite – amoeboid trophozoite with SCHUFFNER’S DOT
• Infective form – sporozoite

5.PLASMODIUM FALCIPARUM (GAMETOCYTE)

•Cause malignant malaria


•Male gametocyte – banana shaped
•Female gametocyte – crescent shaped.

6.MICROFILARIA

• Thick smear used to find out microfilaria


• 2 common species- W.bancrofti , B. Malayi
• W.bancrofti – pointed tail tip , gentle flowing smooth curved , nuclear
column-nuclei discreate, cephalic length same as breadth
• B.malayi – kinked end with two terminal nucleus , crinkled secondary curves ,
nucleus coloums-smudge, cephalic end longer than broad

7.PLASMA CELL

• Size 14-20 microns


49
GOVT MEDICAL COLLEGE, TRIVANDRUM
LUMINAIRE-2015
• Nucleus – eccentric and small
• Increase in – plasma cell leukemia , measles &german measles , serum
sickness, IMN, Chronic infection
• MYELOMA CELL- atypical and immature plasma cell

CLIP CHARTS

CASE 1

8 year old boy brought with c/o fever, vomiting and neck rigidity-1 day. LP done.
C.S.F
Appearance - clear
Pressure - 250mm of water
Protein - 110mg%
Sugar - 20-mg%
Chlorides - 400 mg%
Microscopy - 200 cell/c.mm mainly lymphocytes
Special stain done

Diagnosis- Tuberculous meningitis

CASE 2
10 year old girl brought with c/o fever, vomiting-2 days. O/E
febrile.Neckrigidity+.LP done.
C.S.F
Appearance - turbid
Pressure - 250mm of water
Protein - 150 mg%
Sugar - 10mg%
Chlorides - 400 cells/c.mm
Cells - polymorphs

Diagnosis- Pyogenic meningitis

CASE-3
6 year old boy. Puffiness of face - 1 day
50
GOVT MEDICAL COLLEGE, TRIVANDRUM
LUMINAIRE-2015
Urine -
Color - Smoky
Specific gravity - 1.030l
Sugar - nil
Albumin - ++
Blood - ++
Deposit: - RBCs + + pus cell+, hyaline casts ++, RBC cast ++,
Granular casts ++
Diagnosis-Glomeruonephritis

CASE 4
Male 50 years, C/o Abdominal discomfort -1 week, deepening jaundice - 2 days
Urine
Color - deep yellow
Bile pigment - ++
Bile salt - ++
Urobilinogen - absent

Diagnosis -Obstructive Jaundice

CASE 5
35 year old female c/o generalized edema -2 weeks
Urine - 1035
Specific gravity - opalescent
Sugar - nil
Albumin - ++++
Deposit - granular cast ++, hyaline cast +, RBC cast – occasional.

Diagnosis – Nephrotic syndrome

CASE 6
55 year old female-clinical diagnosis-uremia
Urine -
Specific gravity - 1035
51
GOVT MEDICAL COLLEGE, TRIVANDRUM
LUMINAIRE-2015
Albumin +
Sugar - >2g%
Acetone - nil

Diagnosis- Diabetes mellitus

CASE 7
21 years old female c/o fever, vomiting, loss of appetitie- 3days
Color Urine - deep yellow
. Bilepigmen1 - ++
Bile salt - ++
Urobilinogen - absent

Diagnosis – Obstructive jaundice


CASE 8
7 years old girl presented with recurrent jaundice and pallor- 6months
Urine
Color . - Yellow
Bilepigment - absent
Bile salt - absent
Urobilinogen - +++

Diagnosis – Hemolytic anemia, Hemolytic / pre-hepatic jaundice

CASE 9
8 years old boy c/o fever with cervical lymphadenopathy- 3 days.
Blood - 12g%
Hb - 13,500 cells/c.mm
TC - - P25 L 65 E5 M5
DC - RBCs normochromic normocytic WBCs-
lymphocytesPeripheral smear
Numerous atypical lymphocytes, Platelets- adequate

Diagnosis – Lymphocytosis with atypical lymphocytes ( suggestive of IMN)

CASE 10
52
GOVT MEDICAL COLLEGE, TRIVANDRUM
LUMINAIRE-2015
Male 38 years c/o fatigue O/E pallor ++, splenomegaly ++
Blood
Hb - 6 g%
TC - 3,00,000/c.mm
DC - P12,L4,E7,M2,B10
Peripheral smear - Metamyelocyte - 40%, Myelocyte - 23%,
Blast cells - 2%
Plateletcount - 3,20,000/c.mm

Diagnosis- Chronic Myeloid Leukemia (to be confirmed by karyotyping to demonstrate


Philadelphia chromosome & LAP to differentiate

CASE 11
25 year old male c/o sore throat, fever - 3 days. H/o treatment with antibiotic
Blood
Hb - 12g%
TC - 4000/c.mm
DC - P30 L60 E8 M2
Platelet - 2,00,000/c.mm

Diagnosis - Agranuclocytosis

CASE 12
10 years old girl c/o fever - 1 week, bleeding from gums - 3 days.O/E Pallor
+Cervical lymphadenopathy.
Blood
Hb - 4g%
TC - 50,000/c.mm
DC - P3 E0 L3 M2 Abnormal cells - 92%
Platelet - 30,000/c.mm
Peripheral smearRBC - normochromic normocytic, WBC - large
number of blasts of lymphocytic series,
Platelets - reduced in number

53
GOVT MEDICAL COLLEGE, TRIVANDRUM
LUMINAIRE-2015
Diagnosis – ALL (Diagnosis confirmed by PAS staining and IHC)

CASE 13
3 year old c/o paroxysmal bouts of coughing -1 week
Blood
Hb - 12 g%
WBC - 30,000/c.mm
DC - P20L70E5M5
Platelet - 200,000/c.mm
Peripheral smear - no immature cells

Diagnosis – Absolute lymphocytosis (On clinical correlation, Pertussis)

CASE 14
30 year old female H/O whole body irradiation
Blood
Hb - 5g%
RBC count - 2million/c.mm
WBC count - 1000/c.mm
Platelet - 20,000/c.mm
ESR - 120mm/lslhour

Diagnosis- Pancytopenia ( suggestive of aplastic anemia)

CASE 15
20 year old female c/o high fever -10 days, abdominal discomfort and loose
motion - 3 days. O/E spleen palpable.
Blood findings
Hb - 11.5 g%
TC - 3000/c.mm
P35 L58 E2M5, no abnormal cells
Platelet - 200000/c.mm
Diagnosis- Leucopenia with lymphocytosis ( suggestive of typhoid)

CASE 16
54
GOVT MEDICAL COLLEGE, TRIVANDRUM
LUMINAIRE-2015
13 year old male c/o loss of appetite and pallor -1 month
Blood
Hb - 5 g%
TC - 7000/c.mm
DC - P40 L30 E26 M4
MCV - 80 fl
Blood picture - RBC anisopoikilocytosis, marked hypochromia,
pencil shaped cells, Target cells +, WBC -
eosinophilia, Platelet seen scattered.

Diagnosis-Hypochromic microcytic anemia with eosinophilia

CASE 17
3 year old boy c/o pallor and stunted growth O/E mild icterus, Splenomegaly
Blood
Hb - 4.5 g%
TC - 7000/c.mm
Platelet - 250000/c.mm
Peripheral smear
RBC - Show anisocytosis, hypochromia,
microcytosis. Numerous polychromatic cells,
few normoblasts,
WBC - normal,
Platelet - Normal

Diagnosis-Hypochromic microcytic anemia with evidence of hemolysis, probably


Thalassemia
CASE 18
60 year old male ,weakness,back pain.
Investigations:
Hb - 6gm%
ESR - 110mm 1st hour
x-ray skull and spine - multiple lytic lesions
Peripheral smear - normocytic normochromic anemia
with increased rouleauxformation.Bone
marrow aspiration done.
55
GOVT MEDICAL COLLEGE, TRIVANDRUM
LUMINAIRE-2015

Diagnosis - Multiple myeloma (to be confirmed by serum electrophoresis)

CASE 19
Male 14 yr old presented with fever ,cervical lymph node enlargement and sore
throat
Hb - 12 gm%
Tc - 16,000/mm3
Dc - pol 20%,lymp- 80%
Atypical lymphocyes constitutes about 22% of lymphocytic series
Platelet - 1 lakh/mm3
Interpret the chart and give your diagnosis.

Diagnosis: Absolute lymphocytosis with atypical lymphocytes – IMN (to be


confirmed by Paul Bunnel test)

INSTRUMENTS

1. HB PIPETTE
Vol - 20mm3
Use: Hb estimation by Sahlishemoglobinometer

2. WINTROBE’S TUBE
Length: 11 cm, Diameter: 3mm
Calibration: 0-10 mm
Use: PCV,ESR

3. WESTERGEN’S TUBE
Length: 30 cm, diameter: 2.5 mm
Calibration: 0-200[top to bottom]
Use:ESR

4. WBC PIPETTE
Whitehead; Markings: 0.5,1,11
Use: WBC count, RBC count in erythropenia, semen, CSF
56
GOVT MEDICAL COLLEGE, TRIVANDRUM
LUMINAIRE-2015

5. RBC Pipette
Uses: WBC count, RBC count, Platelet count, Sperm count

6. LANCET
used to collect blood by finger puncture
made of steel up to 3 mm tip

7. NEUBAUR’S Counting chamber


depth : 0.1 mm
uses: WBC count, RBC count, Platelet count, Sperm count
8. Urinometer
to determine the specific gravity of the urine
minimum 25 ml of urine

9. Esbach’sAlbuminometer
Quantitative estimation of albumin in urine
‘U’ marking for urine, ‘R’ marking for reagent
Reagent + Urine mixed and kept vertically for 24 hrs
Height of coagulation measured and expressed in g/dl
Esbach’s reagent: Picric acid, Citric acid, Distilled water

10. MULTISTIX REAGENT STRIPS


to detect urinary constituents like Glucose, Albumin etc
Indicators: tetrabromophenol blue, orthotoluidene

11. BONE MARROW BIOPSY NEEDLE


Identification: short and stout needle
sharp pointed stylet
Indications: leukemia, Megaloblasticanemia , Parasitic infections like
leishmaniasis, malaria etc.
Sites: sternum opposite to 2nd intercostal space,
uppermeial end of tibia iliac crest

12. LUMBAR PUNCTURE NEEDLE


Made of platinum iridium
57
GOVT MEDICAL COLLEGE, TRIVANDRUM
LUMINAIRE-2015
Site: between L3-L4 or L4-L5
Indications: Meningitis, Encephalitis , Subarachnoid h’age
Contraindication: Presence of papilloedema, Bleeding tendency , Infection of site
13. LIVER BIOPSY NEEDLE
Has 3 parts: Cannula or outer needle, Trocar , 3rd part as a bifid or split needle
Indication: Cirrhosis, Hepatoma, Granuloma: Tb, schistosomiasis
Contraindication: coagulative disorders, hydatid cyst of liver ,hemangioma liver
Site: midaxillary line in the 10th intercostal space
14. AYRES SPATULA
made of soft wood and has two ends: one plain and one bifid
bifid end: used to collect cervical smear
plain end: buccal smear
15. NEUBARS COUNTING CHAMBER
depth is 0.1 mm
Uses: WBC count, RBC count, platelet count, Sperm count
16. L BLOCK
Leuckhart’s L block
used as a mould while producing wax blocks
17. TISSUE PROCESSING CAPSULE
Use: to bath the tissue in the processing fluid.
18. PARAFFIN BLOCK
Used for embedding the tissue in wax.

Blood grouping (imp)


Oxalated blood used
Methods: Direct (tube & tile/slide methods), Indirect (serum grouping)
Bombay Group
H gene A, B, or O gene
Precursor substance→a H antigen→a A/B antigen

If H gene is recessive , H antigen is not formed hence A/B antigens are also not
formed. So person’s blood group is O. Since A, B or H antigens are absent on RBC
membrane of Bombay blood group, their plasma contains anti-A, anti-B and anti-H
antibodies. Due to presence of anti-H antibody, their blood can not be transfused with
any other blood other than Bombay group.
58
GOVT MEDICAL COLLEGE, TRIVANDRUM
LUMINAIRE-2015
Dangerous O group – persons with O group having very high titres of anti-A and anti-B
antibodies.
Landsteiner’s laws
Other blood group systems – MN system, lewis, kell, duffy Lutheran
Colouring agent - methylene blue(anti-A)
- acriflavin yellow(anti-B)
Du antigen – it is a variant of D antigen and is weak. So, during group determination,
patient may be labeled –ve, and when he becomes a donor , he may be giving Du
positive blood to a Rh-ve person and can cause transfusion reaction.
Rh incompatability
Coomb’s test (direct & indirect)
Erythroblastosisfoetalis
Transfusion reactions, transfusion transmitted diseases (HIV, Hepatitis B, Malaria)
Blood component preparation(FFP, cryopptetc), blood substitutes
Plasma substitutes – Albumin, Hydroxy ethyl starch, Dextran
Anticoagulants used in blood bank (CPD, CPD-A, APD)
Rh: chromosome 1; ABO: Chr 9; H: Chr 19
Why saline preparation of RBCs for ABO grouping??? (prozone phenomenon)
Why whole blood for Rh grouping? (weak antigenicity of Rh )
Use of control (to detect auto agglutination if present)
Difference between rouleaux formation and agglutination (rouleaux formation is
reversible while other is not, on adding a drop of water – rouleaux disperses, while
agglutination persists)
Importance of blood grouping

Hb estimation
PCV
ESR ( note the time immediately after dipping the esrite tube into the vial)
Anticoagulant of choice – 3.8% sodium citrate 1:4
WBC count
Peripheral smear preparation and staining.
Stains used :
Romanowsky group of sdyes – Leishman, Wright, Giemsa, May –
Grumwald and Jenner stain.
Leishman’s stain – composition
1.Methylene blue acts as nuclear stain
59
GOVT MEDICAL COLLEGE, TRIVANDRUM
LUMINAIRE-2015
2.Eosin acts as cytoplasmic stain
3.Acetone free methyl alcohol - acts as solvent and fixative
STAINING TECHNIQUE
1. Keep the slide in a horizontal position
2. Leishmann’s stain is added to cover the entire smear
3. Add distilled water( double the amount of stain)
4. Mix the stain and water properly by blowing air
5. Keep it for 7- 10 minutes.
6. Pour water and wash thoroughly with running water.
7. Wipe the other side of slide with cotton
8. Dry the smear by placing the slide in a slanting position with tail end upwards

Criteria for Good staining


1. Should be pinkish in colour.
2. There should not be any stain deposits
3. Should be tongue shaped
4. No serrations and vacuoles
5. RBCs coppery red in color, no shrinkage or swelling, eosinophils orange red

IMP – if there is excess staing, add stain again. if it is understained, repeat the staining
Preparation of thick smear –

Prepared by placing 4 drops of blood on a slide and join the 4 drops of blood together
into an area of 1sq:cm or a similar sized circle.
Use – for demonstration of parasites such as malaria and microfilaria

Peripheral smear examination format

1. Under low power


Preparation and staining( smear shows adequate staining with no stain deposits and
cells are evenly distributed)
WBC count increased or not
Parasites seen or not

2. Under oil immersion


RBC
60
GOVT MEDICAL COLLEGE, TRIVANDRUM
LUMINAIRE-2015
Shape and color of RBC
Nucleated RBCs – present or not
Polychromasia – present or not
Inclusion bodies – present or not
Parasites –seen or not
Excessive rouleoux formation – seen or not
WBC
Apparent count, distribution, nuclear and cytoplasmic characteristics, immature
cells, blast cells, any other cells
Platelets
Apparent count, distribution, morphology

DIAGNOSIS / IMPRESSION

( For apparent WBC count: No. of WBC / HPF X 1000


For apparent platelet count: No. of platelets / oil immersion field X 15000 )

ALGORITHM FOR PERIPHERAL SMEAR EXAMINATION

STEP1
Low Power – focus on areas of uniform cellularity and staining

STEP2
High power – assess WBC count, if high(more bluish) think in terms of AML and CML, if
normal, think in terms of neutrophilia and eosinophilia.
If WBC count appears normal, RBCs hypochromic and different shapes of RBCs seen,
think in terms of HMA

STEP3
Under oil immersion –
If all immature blast cells – AML
If all series of maturation and basophilia – CML
Look at granules in case of mature WBC
If pink granules – neutrophilia
If orange granules – eosinophilia.

61
GOVT MEDICAL COLLEGE, TRIVANDRUM
LUMINAIRE-2015

NORMAL VALUES

Specific gravity 1.003 – 1.030

Urea 25 – 30g/ 24 hr urine excreted

Creatinine 1 – 1.8g/ 24 hr urine excreted

Protein 30 -150mg/ 24 hr urine excreted

Glucose 2 – 20mg/dl in fasting urine

PRESERVATIVES

Toluene all round preservative

Formal excellent for formed elements

Sodium bicarbonate preserve UBG and


porphyrin

Hibitane preserve glc in 24hr urine


(Chlorhexidine) sample

SPECIAL PRESERVATIVES

Addis count 40% formaldehyde

Estimation of VMA conc.HCl

Estimation of conc.H2SO4
catecholamines

Estimation of aldosterone CHCl3

TESTS OF PROTEIN

62
GOVT MEDICAL COLLEGE, TRIVANDRUM
LUMINAIRE-2015
1. Heat and acetic acid test
Drop of acetic acid – turbidty due to phosphate & carbonate disappear
Drop of nitric acid – turbidty due to mucin disappear
2. Sulphosalicylic acid test(done in CLIP lab, MCH)
3. Heller’s test
4. Esbach’sreagent test
5. Dipstick method

INTERPRETATION

Trace barely visible cloudiness

1+ cloudiness without granular ppt

2+ cloudiness with granular ppt but no


flocculation

3+ loudiness with granular ppt and


flocculation

4+ thick curdy ppt and coagulation

BJP tests
1. Heat and coagulation test – BJP ppt at45ºC, max ppt at 60ºC;At 80ºC ppt start to
disappear,at 100ºC completely disappear& reappear on cooling
2. Toluene sulphonic acid test
3. HCltest(bradshaw”s test:2ml urine+concHCl—>white ring)

DDs for BJP


1. Multiple myeloma
2. Amylodosis
3. Walderstrom’smacroglobulinemia
4. CML
Quantitative estimation of protein
Albumin - Esbach’s method
63
GOVT MEDICAL COLLEGE, TRIVANDRUM
LUMINAIRE-2015
Globulin – urine + amm.sulphate

REAGENT COMPONENTS

Bendict’s reagent coppersulphate,anhydroussod.bicarbonate,sod. citrate,


D/W

Esbach’s reagent picric acid,citricacid,distilled water

Fouchet’s reagent trichloroaceticacid,ferricchloride,distilled water

Ehrlich’s reagent p-dimethylaminobenzaldehyde,HCl

TESTS FOR GLUCOSE

1. Benedict’s test –semiquantitative test


Sod.bicarbonate – provide alkaline medium
Sod.citrate – prevent pptn of cupric oxide by forming deep blue complexes which
dissociates to supply cupric ions for oxidation

INTEPRETATION

Trac slight yellow ppt with greenish blue/bluish green <0.5 g/dL
e mixed soln

1+ greenish yellow ppt with apple green mixed soln =0.5 g/dL

2+ large amount of yellowish green ppt =0.75


g/dL

3+ orange ppt =1 g/dL

4+ large amount of yellow to red ppt. no blue in >2 g/dL


supernatant

2. Glucose oxidase test(specific for glucose)


TESTS FOR KETONE BODIES

64
GOVT MEDICAL COLLEGE, TRIVANDRUM
LUMINAIRE-2015
Ketone bodies are acetone, aceto acetic acid and beta hydroxy butyric acid.
Ketonuria is seen in anorexia, fasting, starvation, DKA, prolonged vomiting.

ROTHERA’S TEST acetone &aceto acetic


acid

Beta hart test beta hydroxy


butyric acid

Gerhardt’s test acetoacetic acid

In Rothera’s test, urine saturated with amm.sulphate to keep liquor ammonia above the
urine solution.

TEST FOR BILE PIGMENTS


MODIFIED FOUCHET’S TEST
BPs adsorbed on barium sulphate
Fouchet’s reagent: 10% Ferric chloride, trichloro acetic acid, distilled water
Biliverdin(green), Bilicyanin (blue)
TEST FOR BILE SALTS
HAY’S TEST
Based on the physical property of surface tension
TESTS FOR UROBILINOGEN
EHRLICH’S TEST
Pale pink color – UBG normal
Cherry red color - UBG increased
Watch the color through the mouth to increase the depth of liquid column for better
appreciation of the color
Clinical condition Bile UBG
pigment Bile salt

Prehepatic - - +++ to ++++

Hepatic + to ++++ + ++

65
GOVT MEDICAL COLLEGE, TRIVANDRUM
LUMINAIRE-2015
Posthepatic ++ to + +/-
++++

TEST FOR BLOOD


BENZIDINE TEST
Boiled and cooled urine is used to destroy peroxidase enzyme present in pus cells,
bacteria etc. positive test in hematuria, hemoglobinuria&myoglobinuria
Trace Faint
green

1+ Green

2+ Greenish blue

3+ Blue

4+ Deep
blue

Sample A
Male, 60yrs, presented with loss of appetite, nausea & vomiting in casualty.
Diagnosis: Diabetic ketoacidosis
Tests to be done: 1. Benedicts test-for sugar
2. Rothera’s test-for ketone bodies
Sample B
6 yr old male child, complains of puffy face of 1 day duration.
Diagnosis: acute glomerulonephritis.
Test to be done: 1. heat and acetic acid test-for protein ,
2. benzidine test-for blood
Sample C
Male 50 yrs complains of abdominal discomfort 1 week and deepening of
jaundice 3 days.
Diagnosis: obstructive jaundice
66
GOVT MEDICAL COLLEGE, TRIVANDRUM
LUMINAIRE-2015
Test to be done: 1. Fouchet’s test-for bile pigments
2. Hay’s test-for bile salts
3. Ehrlichs test-for urobilinogen

CRYSTALS IN ACIDIC URINE

Type Appearance Disease associated

Uric yellow brown rhombic plates rossets c/c nephritis,


aci gout
d

Ca colourless envelope/dumb bell liver ds, DM ,


oxl shaped hyprparathyroidsm
ate

Sod.urat colourless/yellowish slender prism no clinical significance


e in fan/sheaf like pattern

Cystine large flat transparent with 1 or nephritis, chyluria,


more nephritic syndrome
Corners cut off

Leucine highly refractile yellow/brown severe hepatitis, MSUD


spheres
With radial/concentric striations

CRYSTALS IN ALKALINE URINE

Type Appearance Disease

Struvite/triple coffin lid appearance. Seen in normal urine or


phosphate UTI due to urease+ve
Organisms

67
GOVT MEDICAL COLLEGE, TRIVANDRUM
LUMINAIRE-2015
Calcium large yellow brown/colorless
carbonate spheroids or small
round/ovoid/dumb bell
forms

Amm.biurate Apple thorn crystals

Calcium Large thin irregular granular


phosphate colorless plates

CASTS

Type Disease associated

Hyaline Few in normal & increased in proteinuria or renal/extrarenal


origin

Granular c/c nephritis

RBC Glomerulo nephritis

WBC a/c pyelonephritis &glomerulo nephritis

Epithelial c/c renal d/s

Fatty Nephrotic syndrome & c/c glomerulo nephritis

HISTOPATHOLOGICAL TECHNIQUES
Registration→fixation→grossing labeling→decalcification→dehydration→Clearing
→impregnation→embedding→cutting→staining→mounting→reporting
Ideal fixative
1. Should penetrate tissue easily
2. Rapid in action
3. Be isotonic
4. Cause min.physical and chemical alteration
5. Cheap, stable and safe to handle
Volume 10-20 times the volume of the specimen except OsO4

68
GOVT MEDICAL COLLEGE, TRIVANDRUM
LUMINAIRE-2015
Routine fixative 10% neutral buffered formalin
Special fixatives
• Glycogen storage diseases - Carnoy fixative
• Testicular biopsy - Bowinsfluid(Picric acid)
• Enzyme histochemistry - Cold acetone
• Electron microscopy - 4% glutaraldehyde, Osmium tetroxide
• Formol saline is the fixative in renal, skin and eyeball biopsy

Disadvantages of formalin
loss of fat & glycogen
shrinkage of tissue
acid form of hematin in bloody tissue

Decalcifying agents 1.dilute mineral acids


2.chelating agents
3.organic acids(acetic acid & formic acid)

Tissue processing Ethyl alcohol (ascending grades)


(dehydration)
Agent

Clearing(dealcoholism) 1.chloroform
agents 2.xylene
3.toluene
4.benzene

Embedding Impregnation of tissue with a medium3


methods
1.paraffin wax
2.celloidin
3.gelatin

MICROTOME
• used for cut the section
• Normaly-6-8micrometer thickness
• For immunohistochemistry and AgNOR study-4-5 micrometer
• Types-rotary microtome(routinely)

69
GOVT MEDICAL COLLEGE, TRIVANDRUM
LUMINAIRE-2015
• Sledge
microtome,sawmicrotome,vibratingmicrotome,rocking
microtome

SPECIAL STAINS
( write appr.stains in short notes to get extra marks)

Van gieson yellow collagen and other


connective tissues

Verhoeff’s method elastic fibres

Von kossa stain calcium

Perl’s stain Iron

Congo red amyloid

Rhodanineorcein copper

Mayer’s mucin
mucicarmine

PAS glycine

CYTOPATHOLOGY
Ideal fixative -95% ethyl alcohol and ether in the ratio 1:1
Papanicolaou staining procedure

Fixation →hydration(descending grades of alcohol)→haematoxylin (nuclear stain)


→Water rinses→blueing soln.(raise pH,turns red to blue)→water rinses→
dehydration(prepare for alcohol counterstains)→OG(cytoplasmic stain) and rinses→
EA(cytoplasmic stain) rinses→alcoholic rinse(prepare for xylene)→clearing with xylene
FNAC (Fine Needle Aspiration Cytology)
10-20 cc syringe and 22-23 guage needle is used
Advantages -Done as OPD procedure , No anaesthesia required , Low cost procedure
70
GOVT MEDICAL COLLEGE, TRIVANDRUM
LUMINAIRE-2015
,Results on the same day ,No much trauma

PHARMACOLOGY

SYLLABUS
Paper I-General pharmacology, ANS, CVS, Blood, Diuretics, CNS, Autacoids,
Respiratory system
Paper II-GIT, Hormones, Antibiotics, Chemotherapy, Miscellaneous

Question Pattern
1. Essays -2x6=12
2. Short notes -5x3=15
3. Answer briefly -5x1=5
4. Name two drugs for -8x1=8

71
GOVT MEDICAL COLLEGE, TRIVANDRUM
LUMINAIRE-2015

INRODUCTION; ROUTES OF ADMINISTRATION


1. Drug ,Orphan drugs - Definition
2. Source of drugs (refer record)
3. Transdermal Therapeutic Systems
4. Sublingual , rectal , transdermal, Subcutaneous , inhalational routes -Example
5. Limitations of oral route of administration/ Advantages of i.v
6. Inhalational and sublingual routes of administration-advantages
7. Special drug delivery system

• Drugs given rectally-PEDI- Paraldehyde, Ergotamine, Diazepam, Indomethacin


• 2 orphan drugs- Fomepizole, Sodium nitrate
• 2 drugs from animal source- porcine insulin (pig), Heparin (ox lung)
• 2 drugs given sub lingually-GTN, Buprenorphine
• 2 drugs given as transdermal patches- Nicotine, Fentanyl

PHARMACOKINETICS
1. Pharmacokinetics- Definition
2. Bioavailability
3. Redistribution
4. Plasma Protein Binding
5. Volume of Distribution
6. Prodrug
7. Hoffman elimination
8. First Pass Metabolism
9. Microsomal Enzyme Induction and Inhibition.
10.HalfLife
11. Therapeutic drug monitoring
12.Kinetics of Elimination
• INDUCERS - Griseofulvin, Phenytoin, Rifampicin, Carbamezapine, Phenobarbitone

• INHIBITORS - Valproate, Ketoconazole, Cimetidine, Ciprofloxacin, Erythromycin, INH

• Drugs with ZERO ORDER kinetic - Warfarin, Alcohol & Aspirin, Theophilline, Tolbutamide, P
henytoin

PHARMACODYNAMICS
1. Pharmacodynamics-Definition
2. Affinity, Efficacy, Potency
3. Partial agonists, inverse agonists
72
GOVT MEDICAL COLLEGE, TRIVANDRUM
LUMINAIRE-2015
4. Receptors- Types, functions
6. Therapeutic window Phenomenon, Therapeutic index
7. Synergism / Antagonism.
8. Dose Response Curves.

PHARMACOTHERAPY
1. Pharmacogenetics, Pharmacogenomics
2. Tolerance
3. Tachyphylaxis.

ADVERSE DRUG EFFECTS


1. Adverse drug reactions- Definition, types
2. Idiosyncracy, Drug allergy( mechanism, types, treatment)
3. Teratogenicity
2 drugs showing tachyphylaxis- Ephedrine, Nicotine

CHOLINERGIC SYSTEM IN BODY


1. Types of cholinergic receptors, sites, agonists and antagonists
2. Pilocarpine and occusert, Bethanechol uses
3. Anti-ChEs – Classification, MOA, egs, uses
4. Physostigmine v/s Neostigmine
5. Belladona poisoning –clinical features, management with dose
6. OP poisoning –MOA, clinical features , Management
7. Myasthenia gravis- Rx,myasthenic&cholinergic crisis
8. Anti cholinergics- Classification, Uses, examples
10. Glycopyrrolate, dicyclomine, cyclopentolate, tropicamide
12. Vesicoselective anticholinergics
P.B.
1. Pralidoxime for tacrine poisoning (Tacrine-reversible Anti-ChE , Oxime- Enzyme
reactivator)
2. Betaxolol/physostigmine -1st choice in glaucoma (Betaxolol bcoz decreases IOP,
protective effect on retinal neurons by reducing Na-Ca influx, less bronchpulmonary and
cardiac side effects, no consistent effect on pupil size or accommodation)
3. Neostigmine/physostigmine in my.gravis (Neostigmine bcoz anti-ChE with prominent
action on skeletal muscles and less CNS effects)
4. Neostigmine+atropine in cobra bite (Atropine prevents cholinergic adv effects of
neostigmine)
5. PB of oximes in OP poisoning

73
GOVT MEDICAL COLLEGE, TRIVANDRUM
LUMINAIRE-2015
6. Hyoscine in motion sickness (more completely metabolized, better BBB penetration)
7. Glycopyrrolate in Pre Anaesthetic Medication (Reduces tracheobronchial & salivary
secretions→prevent laryngeal spasm)
8. Mydriatics of choice in elderly (phenylephrine) & children <5 (atropine)
9. Atropine – specific antidote of anti ChE
10. Ipratropium and its rationale in COPD (Reverse bronchoconstriction due to stim of M3
receptors,Inhaled- decreases bronchial secretions)

TIPS
1. Two drugs in Alzeimer’s disease-Donapezil, Rivastigmine
2. DOC in Belladonna poisoning -Physostigmine 0.5 to 2 mg i.v
3. DOC in Organophosphate poisoning - Atropine 2mg i. v every 1 Omts
4. DOC in Motion sickness-Hyoscine
5. Shortest acting carbamate- Edrophonium , use in myasthenia
6. Cerebroselective carbamates-Rivastigmine &Donepezil
7. Only water soluble OP & used in aphakic glaucoma-Echothiophate
8. 2 drugs used in Myasthenia gravis-Neostigmine, Pyridostigmine
9. Drugs c/I in myasthenia - aminoglycoside antibiotics, quinine, pancuronium , tubocurarine.
Anticholinergic drugs used in
a) Peptic ulcer-propanthelene, pirenzepine
b) COPD-ipratropium, tiotropium
c) Abdominal colic-dicyclomine
d) Refractory error testing in children- atropine 1% ointment

ADRENERGIC SYSTEM IN BODY


1. Distribution and actions of α&β receptors, agonists and antagonists
2. Adrenergic drugs- Therapeutic classification
3. Dales vasomotor reversal
4. Nasal decongestants-systemic and topical, Anorectic agents
5. α blockers- Classification (selectivity), uses and adverse effects
6. Phentolamine, Prazosin, Tamsulosin
7. β blockers- Classification, MOA, Uses and Adverse effects
8. Cardioselective β blockers,β blockers with ISA (Acebutolol, Pindolol)
9. Propranolol {Class of drug, MOA, Kinetics, Interactions, Adverse effects,C/I, Uses} )
11. α +β blockers- Labetalol, Carvedilol
12. Glaucoma- ESSAY
a) Rx of open angle, angle closure (Drugs with priority)
b) CA inhibitors used in glaucoma
c) Mydriatics c/I in a/c congestive glaucoma
74
GOVT MEDICAL COLLEGE, TRIVANDRUM
LUMINAIRE-2015
P.B
1. Dopamine/adrenaline in shock(Hint:dopamine-cardiogenic shock,septic
shock,adrenaline-anaphylactic shock)
2. Adrenaline in anaphylactic shock & its route (↑BP,counteracts bronchospasm/laryngeal
edema)
3. Lignocaine +adrenaline in local anesthesia (duration of anaesthesia↑,systemic toxicity of LA
and bleeding↓)
4. Tamsulosin +finasteride in BPH (Tamsulosin-↓prostatic/bladder neck mus tone,
finasteride-↓prostate size
5. Propranolol in thyrotoxicosis (rapidly controls sympathetic symptoms, inhibits peripheral
convn of T4→T3
6. Propranolol C/I in diabetes, variant angina & hyperlipedimia(delays recovery from
hypoglycaemia due to insulin and OHGA and warning signs of the same are suppressed,
exacerbates variant angina by unopposed mediated coronary constrctn, alters lipid
profile-TG,LDL↑,HDL↓)
7. Mannitol in acute congestive glaucoma(decongest eye by osmotic action))
8. Atropine C/I in glaucoma(IOP ↑ in narrow angle G))
9. α+β blockers in pheochromocytoma(↓BP by both 1and β1 blockade and β2 agonism, by
blocking, they maintain bld vol, ↓BP and antagonize fall in BP aftr surgry by maintaining bld vol, by
β blocking they ↓tachycardia and arrhythmia)
10. Verapamil C/I with propranolol (cardiac arrest, safe with nifedipine)
TIPS
• Clonidine congeners- Apraclonidine, Brimonidine
• 2 drugs used for hypotension- Ephedrine, Mephentermine
• Topically useful C.A-Dorzolamide
• β1 selective blocker used in the Rx of glaucoma-Betaxolol
• Topical carbonic anhydrase inhibitor used in glaucoma-Dorzolamide
• Selective β1 agonist-Dobutamine
• Ultra short acting β blocker-Esmolol
• Longest acting β blocker-Nadolol
• 2 lipid soluble β blockers- atenolol, Sotalol
• 2 β blockers with NO mediated vasodilatation-Nebivolol, Celiprolol
• β1 blocker with β2 agonism-Celiprolol

AUTACOIDS
1. 2nd generation v/s 1st generation anti histaminics
2. Fexofenadine, Loratadine, Cetrizine, Azelastine
3. Uses of anti histaminics
4. Drug therapy of Vertigo
75
GOVT MEDICAL COLLEGE, TRIVANDRUM
LUMINAIRE-2015
5. Ketanserin
6. Drug therapy & Prophylaxis of migraine, MOA of Sumatriptan
7. Clinical uses of Prostaglandins
8. NSAIDs – Essay
9. Selective COX-2 inhibitors
10. Aspirin, Diclofenac sodium
11. Acute salicylate poisoning
12. Ketorolac(Tripathi pg 194)
13. Acute paracetamol poisoning (NABQI metabolite, antidote N-acetyl cysteine)
14. Topical NSAIDS
15. DMARDS-Methotrexate, Leflunomide, Gold CLIPS-(refer mnemonic section)
16. Biological Response Modifiers- Infliximab, Adalimumab, Anakinra
17. Drugs used in gout- Colchicine, Probenecid, Allopurinol
P.B.
1. P.B. terfenadine cardiotoxicity. (Erythromycin+ terfenadine)
2. P.B. Promethazine in motion sickness(antag of histamine in brain/antimuscarinic ppty)
3. P.B NSAIDS in PDA(DA kept patent by PGE2 and PGI2)
4. PB. aspirin in post M.I. patients(inhibit platelet aggregation)
5. PB Sulphonyl ureas and Salicylates (Aspirin displaces sulfonyl ureas from plasma protein
binding site→toxicity)

TIPS
• DOC in Patent ductus arteriosus - indomethacin 0.1-0.2 mg/kg , 12th hrly
• A/C paracetamol poisoning - N acetyl cysteine 150 mg/kg i.v over 15 mts
• 2 antihistamines used in vertigo-cinnarizine, promethazine
• 2 5HT antagonists- Ketanserin, Cyproheptadine
• 2 5HT3 Antagonists – Ondansetron , Granisetron
• Drugs used for the induction of labour-PGE-2
• Drugs used for midterm abortion-PGF-2
• DMARDs-GOLD CLIPS(Gold, Chloroquine, Leflunomide, Immunosuppressants,
d-Penicillamine, Sulfasalazine)

RESPIRATORY SYSTEM
1. Drugs used for Dry Cough
2. Mucolytics – Bromhexine , Ambroxol
3. Drugs in Bronchial Asthma- classification (ESSAY)
4. Salbutamol (Class of drug, MOA, Kinetics, Interactions, Adverse effects, C/I, Uses)
5. Theophylline
6. LT antagonists- Montelukast , Zafirlukast
76
GOVT MEDICAL COLLEGE, TRIVANDRUM
LUMINAIRE-2015
7. Mast cell stabilizers-Ketotifen, sodium cromoglycate
8. Inhalational steroids- Budesonide, Fluticasone
9. Rx of Status Asthmaticus
10.Inhalation therapy in asthma
P.B
1. Ciprofloxacin + Theophylline (Cipro inhibits theophylline meta and↑plasma level)

PITUITARY
1. Somatostatin, octreotide ( GH inhibitors)
2. Bromocriptine ( Actions , uses, adverse effects- Livedo reticularis)
3. Uses of Gonadotropin
4. Examples of GnRH agonists (Nafarelin, Leuprolide), GnRH antagonists (Ganirelix, cetrorelix)

THYROID
1. Synthesis, storage and secretion of thyroid hormone
2. Uses of T3, T4
3. Thyroid inhibitors drugs – classification (Essay)
4. Propylthiouracil
5. Rx of thyrotoxicosis, Thryoid storm
P.B
1. PB. Propylthiouracil/ Carbimazole in pregnancy (propylyhiouracil-greater protein binding, less
transfer to foetus)
2. PB.Iodine + Carbimazole in Preop preparation for thyrotoxicosis (to attain euthyroid state)
3. Propanalol in thyrotoxicosis

DIABETES
1. Human insulin –indications
2. Insulin Analogues
3. RX of diabetic ketoacidosis
4. Insulin resistance
5. Insulin secretagogues (Sulphonyl ureas, meglitinide analogues)
6. Classification of Oral hypoglycemics (Essay)
7. Sulphonylureas {Class of drug, MOA, Kinetics, Interactions, Adverse effects, C/I, Uses}
8. Biguanides
9. MOA Thiazolidinediones
10. α Glucosidase Inhibitor
P.B
1. P.B biguanide/sulphonyl urea in obese pts (Anorectic action )

77
GOVT MEDICAL COLLEGE, TRIVANDRUM
LUMINAIRE-2015
2. P.B. Metformin safer than Phenformin (Lesser chance for lactic acidosis)
3. P.B. Tolbutamide preferred over Chlorpropamide in elderly(hypoglycaemia in elderly more
common with use of chlorpropamide due to its longer action)
4. P.B Regular insulin in DK (after i.v. injection hexameric reg.insulin dissociates rapidly to
produce prompt action, rapidly corrects metabolic abnormalities)

CORTICOSTEROID
1.Corticosteroids- MOA, Uses, Adverse effects, C/I
2.DOC of cerebral edema (Dexamethasone / Betamethasone)
3.Methyl prednisolone
4.Classification of steroids based on duration of action
5.2 uses of steroids where high dose administration & long term therapy is needed (organ
transplantation,collagen vascular d/s)
6. How to stop treatment with corticosteroids?
P.B
1. P.B hydrocortisone in Addisons d/s (acts rapidly, short duration of action, has both primary
glucocorticoid and mineralocorticoid activity)

GONADAL HORMONES
1. Androgens- side effects and uses
2. Anabolic steroids
3. Anti Androgens – Danazol . Flutamide
4. 5 α reductase inhibitors – Finasteride , Dutasteride
5. Sildenafil
6. Oestrogens – uses
7. Hormone replacement therapy
8. SERMs – Tamoxifen, Raloxifene, Ormeloxifene
9. Clomiphene citrate
10. SERD - Fulvestrant
11. Progesterone - uses
12.Mifepristone
13.Oral Contraceptive Pills – Classification , Uses , Adverse Effects
14. Post coital contraception
15. Injectable contraceptive preparations
16. Uses of oxytocin (Tripathi pg 320)
17.Uterine Stimulants and Relaxants – classification and egs
P.B
1. PB. Clomiphene citrate in Multiple pregnancy (blocks estrogen receptors→stim LH, FSH
secretion→induce ovulation and may→multiple pregnancy)
78
GOVT MEDICAL COLLEGE, TRIVANDRUM
LUMINAIRE-2015
2. PB. OCP + Rifampicin ( contraceptive failure )
3. Oxytocin / methyl ergometrine- for induction of labour(oxytocin-short t1/2 , complete relaxation
b/w contractions preventing fetal asphyxia, fetal descent not compromised)
4. PB methyl ergometrine in PP (sustained tonic uterine contractions→compress uterine bld
vessels→stops bleeding)
5. Rationale for the combination of oestrogen and progesterone in OCPs (both synergise to
inhibit ovulation, progestin ensures prompt bleeding at the end of a cycle and blocks risk of
developing endo.Ca due to estrogen)

CALCIUM BALANCE
1. Parathormone – MOA
2. Vitamin D –Uses
3. Bisphosphonates – MOA, Uses ( Alendronate, Zolendronate)
TIPS
• GnRH agonist used as nasal spray-nafarelin
• Anti-thyroid drug used in pregnancy-propyl thiouracil
• Commonly used iodine isotope-I-131
• Rx of hypoglycaemia- glucose, glucagon
• Hypoglycemic drug used in the Rx of PCOD-Metformin
• Corticosteroids used in cerebral oedema-Dexamethasone, betamethasone
• 2 synthetic androgens- methyl testosterone, fluoxymesterone
• PDE-5 inhibitor-sildenafil
• Pure estrogen antagonist, selective estrogen receptor down regulator-fulvestrant
• Oxytocin antagonist-atosiban
• 2 drugs for osteoporosis- Raloxifene, Alendronate
• Non steroidal estrogen antagonist used as contraceptive-centchroman
• 2 uterine relaxants/Tocolytics- Ritodrine, Nifedipine

SKELETAL MUSCLE RELAXANTS


1. Classify Skeletal Muscle relaxants
2. Difference between competitive and depolarization blockade
3. Pancuronium, Succinylcholine (uses)
4. Dantrolene sodium – MOA , Uses
5. Centrally acting muscle relaxants
TIPS
• DOC in Malignant hyperthermia - Dantrolene sodium 1 mg /kg i.v
• Hoffmans elimination-Atracurium
• Shortest acting NM blocker-Succinyl choline(2-5 min)
• Shortest acting competitive NM blocker-Mivacurium(10-20 min)

79
GOVT MEDICAL COLLEGE, TRIVANDRUM
LUMINAIRE-2015
• 2 Intermediate acting competitive Nm blockers- Atracurium, Vecuronium
• 2 long acting competitive Nm blockers- Pancuronium, Doxacurium
• 2 Nn blockers-Trimethaphan, Hexamethonium

LOCAL ANESTHETICS
1. Classification of local anesthetics and MOA
2. Lignocaine, Bupivacaine
3. Conduction block
4. Spinal anaesthesia- Complications and contraindications
5. Eutectic mixture
P.B
1. PB Adrenaline + local anesthetics
2. PB Oxethazaine in gastritis (unique in ionizing to a very small extent even at low pH. So
effective in anaesthetising gastric mucosa despite acidity of the medium)
TIPS
• LA with antiarrythmic ppty-Lignocaine
• LA with sympathomimetic ppty-Cocaine
• LA causing methaemoglobinemia-Prilocaine
• Pre anaesthetic medication-OHSANA-Opioids, H2 blockers, Sedatives, Anticholinergics,
Neuroleptics, Antiemetics
• Most cardiotoxic LA-Bupivacaine

GENERAL ANAESTHESIA
1. Define general anaesthesia, Stages, Second gas effect and diffusion hypoxia
2. General anaesthesia classification
3. Halothane
4. Inducing agents (Thiopentone, Propofol)
5. Ketamine, Fentanyl
6. Neuroleptics
7. Pre anaesthetic medication
8. Desflurane
TIPS
• GA C/I epilepsy- Enflurane
• Most common ADRs of Halothane-CV and Respiratory depression
• GA producing malignant hyperthermia-Halothane
• Dissociative anaesthesia-Ketamine
• PROPOFOL-Popular, Rapid acting, preferred for OP surgical prOcedures, causes FOL(fall) in
BP

80
GOVT MEDICAL COLLEGE, TRIVANDRUM
LUMINAIRE-2015
ALCOHOL
1. Uses of ethanol
2. Disulfiram, MOA, Drugs with disulfiram like action
3. RX of methyl alcohol poisonn
4. Fomepizole
P.B.
1. PB Ethanol in Methyl alcohol poisoning (competes with methanol for metabolic enzymes and
saturates them preventing formation of toxic metabolites)
TIPS
• Methanol poisoning - Ethanol loading dose 0.7ml/kg followed by 0.15 ml/kg /hr drip
• Drugs causing disulfiram like reaction-Sulphonyl urea, Cephalosporins(cefoperazone,
moxalactam, cefamandole), Metronidazole, Griseofulvin
• Drugs used to treat withdrawal syndrome-Benzodiazepines(chlordiazepoxide, diazepam),
Naltrexone, Acamprosate
• Specific toxicity of formic acid in methanol poisoning-Retinal damage

SEDATIVES AND HYPNOTICS


1. Benzodiazepines v/s barbiturates- MOA, advantages, Uses
2. Drugs affecting GABA receptor gated chloride channel (Fig 29.3)(Tripathi pg 395)
3. Acute barbiturate poisoning, Drug automatism
4. Classification of BZDs based on pharmacokinetic profile
5. Non benzodiazepine hypnotics (Zopiclone, Zolpidem)
6. Flumazenil- Uses
P.B
1. PB Barbiturates contraindicated in acute intermittant porphyria (hint: enzyme induction)
2. PB phenobarbitone in neonatal jaundice (hint: enzyme induction )

ANTI-EPILEPTICS
1. Phenytoin – MOA , ADVERSE EFFECTS , uses
2. Carbamazepine - Class of drug, MOA, Kinetics, Interactions, Adverse effects, C/I, Uses}
[note: DOC in trigeminal neuralgia, used in bipolar disease, has ADH like action, nd prefrd
in pregnancy]
3. Sodium valproate – MOA , Adverse Effects , Uses
4. Broad spectrum anti convulsant(Lamotrigine, Topiramate)
5. Newer generation anti convulsants
6. Gabapentin, Vigabatrine, Tiagabine

81
GOVT MEDICAL COLLEGE, TRIVANDRUM
LUMINAIRE-2015
7. RX of status epileptics, Febrile convulsions
8. Name the foll-DOC for absence seizures-Ethosuximide

TIPS
• 2 drugs for grandmal epilepsy- Phenytoin, Carbamazepine
• Drugs for absent seizure - ethosuximide, Sodium Valproate
• DOC in Absence seizures - sodium valproate
• DOC in Myclonic seizures -sodium valproate

PARKINSONISM
1. Drugs for PARKINSONISM – Classification (Essay)
2. Levodopa {Class of drug, MOA, Kinetics, Interactions, Adverse effects, C/I, Uses}
3. MAO- B inhibitor, Amantadine
4. Central anti- cholinergics in Parkinsonism
P.B
1. P.B. levodopa+ carbidopa
2. P.B levodopa and Vit B (Vit.B6 enhances peripheral decarboxylation)
3. Metoclopramide/ Domperidone in L-dopa induced vomiting (Domperidone doesnot cross
BBB)
TIPS
• Drugs causing secondary parkinsonism-Resepine, Methyl dopa, Chlorpromazine,
Metoclopromide
• 2 uses and 2 adverse effects of Amantadine-anti parkinsonian, anti influenza, ADR-
Livedo reticularis, Ankle edema
• DOC in Drug induced parkinsonism - benzhexol, procyclidine

DRUGS USED IN MENTAL ILLNESS

1. Classification of anti-psychotics

2. Chlorpromazine - MOA, Uses , Advere Effects


3. Non psychiatric uses of antipsychotics
4. Haloperidol
5. Atypical Antipsychotics ( Risperidone , Clozapine , Olanzapine , Quetiapine )
6. Extra pyramidal side effects of anti psychotics
7. Lithium
8. Antidepressants- Classification, Uses
9. RIMA (Moclobemide)
10. Tricyclic Antidepressants

82
GOVT MEDICAL COLLEGE, TRIVANDRUM
LUMINAIRE-2015
11. Cheese reaction
12.Selective Serotonin Reuptake Inhibitors
13. Atypical anti depressants
14. Anti anxiety drugs
P.B
1. PB Lithium plus diuretics esp Thiazides(as a compensatory response to excessive loss of Na
by the use of diuretics, renal excretion of Li is decreased→toxicity)
TIPS
• Plasma concentration of Li-0.5-0.8mEq/L for bipolar disease, 0.8-1.2mEq/L for acute
mania
• Rx of Bipolar disease- Carbamazepine and Valproate
• 2 uses and 2 ADRs of Clozapine-Anti-psychotic, Resistant Schizophrenia,
ADRs-Paradoxical hypersalivation, agranulocytosis, myocarditis
• HIT and RUN drugs-non-selective irreversible MAO inhibitors
• Rx of cheese reactn-i/v Phentoalmine
• DOC in A/C schizophrenia - haloperidol 2 -20 mg/day
• 2 anti-depressants effective in chronic pain-Imipramine, Amitriptyline

OPIOIDS
1. Morphine {Class of drug, MOA, Kinetics, Interactions, Adverse effects, C/I, Uses}
2. Morphine poisoning - Treatment
3. Pethidine
4. Methadone – Maintenance Therapy
5. Tramadol
6. Opioid receptors- MOA
7. Pentazocine, Buprenorphine
8. Naloxone , Naltrexone
P.B
1. PB Morphine in asthmatics (histamine releasing action)
2. PB Morphine contraindicated in head injury
3. PB Morphine in acute LVF
4. PB Pethidine preferred as obstetric analgesic (less neonatal respiratory depression)
TIPS
• 2 uses , 2 ADRs of Naloxone-A/C Opioid poisoning, Neonatal resuscitation following
morphine use in mothers,ADRs-↑ in BP, pulmonary edema
•Opioid used in non specific diarrhoea- Loperamide
•Buprenorphine- partial agonist at mu receptor
•Pentazocine- agonist(k) antagonist(mu)
•DOC in A/c morphine poisoning- naloxone 0.4- 0.8 mg i.v every 2-3min till resp picks up
83
GOVT MEDICAL COLLEGE, TRIVANDRUM
LUMINAIRE-2015

RAAS
1. RAAS, Inhibition of RAAS
2. ACE inhibitors (Captopril,Enalapril) {Class of drug, MOA, Kinetics, Interactions, Adverse
effects, C/I , Uses)
3. ARBs (Losartan) advantages over ACE inhibitors

P.B
1. PB -Lithium + ACEI (hint: reduce Li clearance→ toxicity)
2. NSAIDs + ACEI ( attenuate the hypotensive action)
3. ACEI +K sparing diuretics (hyperkalemia produced)
4. ACEI in HTN and CHF
5.Rationale behind combining ACEI &ARB

TIPS
• Longest acting- Ramipril
• All prodrugs EXCEPT captopril, lisinopril
• ADR ACEI Inhibitor- CAPTOPRIL: Cough, Anaphylaxis, Palpitation, Taste, Orthostatic
hypotension, Potassium increased, Renal impairment, Impotence, Leucopenia)
• Only ACEI available in i/v - Enalapril

DRUGS FOR HEART FAILURE


1. Drugs used in CCF – Classification
2. Digoxin-{Class of drug, MOA, Kinetics, Interactions, Adverse effects, C/I, Uses}
2. Digoxin Poisoning, drug holiday, slow digitalization
3. Inodialators (Amrinone, Milrinone)

P.B
1. PB digoxin in AF
2. PB digoxin + Quinidine (hint: decreases renal clearance of digoxin, displaces it from tissue
binding sites→ toxicity)
3. PB digoxin +frusemide (hint: frusemide→ hypokalemia→digoxin toxicity)
4. PB digoxin + Ppnl (hint : addictive depressivent action on SA and AV node, ppt AV block)
5. PB digoxin + Ca (hint : synergises with digitalis→ toxicity)
6. PB digoxin + CCB (hint-same as 4)
7. PB β-blockers in CHF (hint : sympathetic activity →ventricular wall stress, apoptosis
promotion, pathological remodelling, sinister arrhythmias; all these actions antagonised by
beta blocker)
TIPS
84
GOVT MEDICAL COLLEGE, TRIVANDRUM
LUMINAIRE-2015
• Specific antidote for digoxin : digibind
• ECG changes in digoxin therapy-T invn, ↑PR interval,↓QT interval
• DOC for digoxin induced ventricular arrhythmias-lignocaine
• Most characteristic arrhythmia d/t digoxin-non-paroxysmal supraventricular tachycardia
• Hypokalemia, hypomagnesemia, hypercalcemia ↑risk of digoxin toxicity

ANTI ARRYTHMICS
1. Classification of Anti- Arrythmic drugs, MOA
2. Procainamide
3. Lignocaine in Ventricular arrhythmia
4. Propranolol in Arrythmia
5. Adenosine
6. Beta blockers in Arrythmia
7. Amiodarone
P.B
1. PB Propranolol + Lignocaine (propranolol prolongs t1/2 of lidocaine by reducing hepatic bld
flow)
2. PB Amiodarone + Warfarin (increase warfarin level by reducing renal clearance{same with
digoxin})
TIPS
• DOC inSupraventricular tachycardia- verapamil
• PSVT - Adenosine , Propranolol
• ADRs of Amiodarone-The Periphery of My Lung, Liver and Cornea is Photosensitive
(Thyroid{both hyper and hypothyroidism}, Peripheral neuropathy, Myocardial
depression, Lung fibrosis, Liver toxicity, Corneal microdeposits, Photosensitivity)
• DOC of ventricular tachycardia-Lignocaine

ANTI ANGINAL DRUGS


1. Drugs in angina - Clasification
2. Nitrates -{Class of drug, MOA, Kinetics, Interactions, Adverse effects, C/I, Uses}
3. Calcium Channel Blockers- Classification, Uses , Adverse Effects
4. Nifedipine, Nimodipine ,Verapamil
5. Rational drug Combinations (Nitrates+ CCB, Nitrates +Ppnl)
6. Nicorandil
7. Dipyridamole (Steal phenomenon), Trimetazidine
8. Drug therapy in MI
P.B
1. P.B-sod Nitrite in cyanide poisoning (generate methaemoglobin which has high affinity for
cyanide forming cyanmethaemoglobin. Cytochrome & other oxidative enzymes are thus
85
GOVT MEDICAL COLLEGE, TRIVANDRUM
LUMINAIRE-2015
protected from cyanide)

TIPS
• Shortest acting nitrate- Amyl nitrite(inhalational)
• Other uses of nitrates-Cyanide poisoning, biliary colic, esophageal spasm
• Cerebroselective CCB-Nimodipine
• CCb which release NO-Nitrendipine
• Longest acting CCB-Amlodipine
• K+ channel openers-Nicorandil, Minoxidil
• DOC in vasospastic angina-CCB
• Coronary steal phenomenon-Dipyridamole, Hydralazine

ANTI HYPERTENSIVES
1. Classification of Anti Hypertensives
2. Clonidine, Methyl dopa
3. Hydralazine, Nitroprusside
4. Drugs for HT in Pregnancies
5. Hypertensive emergencies
P.B
1. PB diuretics in HT, Thiazides/ Loop diuretics in HT (refer MOA thiazides. Thiazides have long
duration of action, well tolerated and cheap. Loop D-short duration, used only when complications
present)
2., PB. CCB not in pregnancy (weaken labour)
3. PB. Verapamil + β blocker(marked bradycardia, AV block)
TIPS
• A/E of Methyl dopa-METHILDOPA-Mental retardation, Electrolyte imbalance, Tolerance, H
eadache, Impotence, Lactation, Dry mouth, Oedema, Parkinsonism, Anaemia
• Anti-hypertensives safe in pregnancy-Better Mother Care During Hypertensive Pregnancy-B
eta blockers, Methyl dopa, Clonidine, Dihydropyridine, Hydralazine, Prazosin
• DOC for hypertensive emergencies in pregnancy-Hydralazine
• First dose effect-Prazosin
• Antihypertensive drug in Rx of alopecia-Minoxidil
RENAL PHYSIOLOGY
1. Diagram of nephron with sites of action of diuretics
2. Classification of Diuretics
3. High Ceiling diuretics - {Class of drug, MOA, Kinetics, Interactions, Adverse effects, C/I,
Uses}
4. Non diuretic uses of diuretics
5. Complications of diuretic therapy
86
GOVT MEDICAL COLLEGE, TRIVANDRUM
LUMINAIRE-2015
6. Which is preferred in Ca stones of renal pelvis- Thiazides/ frusemide
7. Acetazolamide, Spironolactone
8. Inhibitors of renal epithelial Na channel- Triamterene, Amiloride
9. Mannitol- MOA, Uses
10. ADH , Desmopressin
P.B
1. PB Loop diuretics+ aminoglycosides (both oto and nephrotoxic→additive toxicity)
2. PB Frusemide in acute LVF (vasodilator action that precedes saluretic action→↓in bld volume
and ven.return)
3. PB. Thiazides/ frusemide in hypercalcemia (Frusemide ↑Ca excretion→↓S.Ca level)
4. PB Thiazide in diabetes insipidus (hint:paradoxical antidiuretic effect)
5. PB Mannitol used in cerebral edema, C/I in pulmonary and cardiac edema(by osmotic action it
↑movt of water frm brain parenchyma, CSF and aq.humor;hence used in cerebral edema, if
ARF already set in kidney is incapable of forming urine→↑plasma vol→pulm edema and heart
failure)
TIPS
• Least potent and most ototoxic loop diuretic-Ethacrynic acid
• DOC of Li induced Diabetes Insipidus-Amiloride
• K+ sparing diuretic used for Rx cystic fibrosis-Amiloride
• 2 drugs used in cerebral edema- Mannitol, Furosemide

BLOOD
1. Oral formulations of Fe and its adverse effects
2. Parenteral Fe therapy, A/C iron poisoning
3. Vitamin K
4. Anticoagulants- In vivo, In vitro
5. Heparin -{Class of drug, MOA, Kinetics, Interactions, Adverse effects, C/I, Uses}
6. LMW heparin
7. Protamine sulphate
8. Warfarin – MOA , Uses , Interactions ,warfarin toxicity
9. Oral anticoagulants MOA,adverse effects, interactions
10. Fibrinolytics MOA uses
11. Clot specific fibrinolytics(Hint:Alteplase)
12. Antifibrinolytics MOA,uses,source
13. Anti platelet drugs, uses
14. Aspirin, Uses
15. Ticlopidine, Clopidogrel
16.Hypolipidemic drugs – Classification with MOA
17. HMGCoA Reductase Inhibitor- ( Class of drug, MOA, Kinetics, Interaction, Adverse
87
GOVT MEDICAL COLLEGE, TRIVANDRUM
LUMINAIRE-2015
reaction, C/I, Uses)
18. Wide spectrum hypolipidimic drug - Nicotinic acid
19. Plasma expanders
P.B
• Folic acid /folinic acid in methotrexate therapy (folinic acid-active coenz, no need to be
reduced by DHFRase so methotrexate(DHFRase inhibitor)toxicity is antagonised by it)
• Menadione contra indicated in newborn (ppt kernicterus by inducing hemolysis→↑bilirubin
load, competitively inhibiting bilirubin glucuronidation)
• Heparin/ Warfarin in pregnancy (foetal warfarin syndrome)
TIPS
• Clot specific fibrinolytic agent-Alteplase
• Specific antidote for fibrinolytics-Epsilon Amino Caproic Acid(EACA)
• Antidote of heparin-Protamine sulphate
• Monoclonal antibody used as anti-platelet drug-Abciximab (Gp IIb-IIIa Inhibitor)
• Drug causing maximum HDL rise-Nicotinic acid
• 2 low molecular weight heparins-Enoxaparin, Reviparin
• 2 antiplatelet drugs-Aspirin, Dipyridamole

PEPTIC ULCER
1. Drugs in peptic ulcer – Classification with examples
2. Proton pump inhibitors
3. H2 antagonists
4. CBS
5. Anti- H Pylori drugs (atleast one triple drug regimen with dose)
P.B
1. Cimetidine/ Ranitidine in 30 yr old male suffering from severe ulcer (Ranitidine preferred since
cimetidine has anti-androgenic action)
2. Sucralfate ineffective in patients with antacid therapy (antacids reduce efficacy of sucralfate
by raising the pH. Sucralfate needs pH <4 for its action)
3. Ranitidine + sucralfate : basis of combination (same as prev. qn)
4. Tetracycline + antacid combination not used why? (antacids reduce absorption of
tetracycline)
5. P.B Mg(OH)2 + Al(OH)3 ; P.B NaHCO3/ Mg trisilicateas good antacid.
6. PB Rabeprazole in peptic ulcer (fastest acid suppression, aid gastric mucin synthesis)

ANTI-EMETICS
1. Classification with examples
2. Metoclopramide- { Class of drug , MOA, Kinetics, Interactions, Adverse effects, C/I, uses}

88
GOVT MEDICAL COLLEGE, TRIVANDRUM
LUMINAIRE-2015
3. Anti-dopaminergic S/E of metoclopramide
4. Domperidone-DOC in drug induced vomiting, DOC Chemo/radiotherapy, DOC levodopa
induced vomiting
5. Cisapride, Ondansetron
P.B
1. PB: Promethazine + Metoclopramid (promethazine supplement antiemetic action and reduce
the attending dystonic reactions)

CONSTIPATION AND DIARRHOEA


1. Classification – Bulk forming, Stool softners, Stimulants, osmotic purgatives
2. Lactulose- Why it is used in Hepatic encephalopathy?
3. WHO recommended ORS formula
4. Super ORS
5. Drug therapy of diarrohea
6. Loperamide
7. Laxative used in hepatic encephalopathy
TIPS
• Prokinetic drug with little anti emetic ppty-Cisapride(no D2 action)
• Somatostatin analogue used in secretory diarrhea-Octreotide
• PPIs given i/v-Pantoprazole, Lansoprazole
• DOC of chemo and radiotherapy induced vomiting-Ondansetron(5HT3 antagonist)

ANTIBIOTICS- GENERAL CONSIDERATION


1. Drug resistance- Mechanism , Cross resistance
2. Super infection
3. MOA of antimicrobials – 2 examples of each with diagram
4. Post antibiotic Effect( Long PAE – fluoroquinolones, Aminoglycosides, B- lactam)
P.B
1. Combination of antibiotics – Rationale
- Broadens the spectrum of activity (Eg: Metronidazole+Ceftriaxone)
- Supradditive synergism (Eg: Penicillin+Gentamicin)
- Prevents emergence of resistance (Multi-drug therapy in TB & Leprosy)
- Reduces duration of therapy ((Multi-drug therapy in TB & Leprosy)
- Reduces adverse effects (Amphotericin+Flucytosine)
- Combination acts faster (Sulfadoxine+Pyrimethamine)
- Sequential blockade (Eg: Sulfamethoxazole+Trimethoprim,
Sulfadoxine+Pyrimethamine)
- Combination may produce cidal effect (Eg: Sulfamethoxazole+Trimethoprim

89
GOVT MEDICAL COLLEGE, TRIVANDRUM
LUMINAIRE-2015
SULFONAMIDES
1. Sulfonamides - MOA, Development of resistance, Adverse effects
2. Topically used sulfonamides
3. Cotrimoxazole { Class of drug, MOA, Kinetics, Interactions, Spectrum,Adverse effects, C/I,
Uses}
4. Site of action of sulfonamides and trimethoprim
P.B
1. PB: Sulfadoxine + Pyrimethmine in malaria (supra-additive synergistic combination due to
sequential blockade in bac.folate meta. However use is limited to chloroquine resistant
falciparum malaria since the combination cause serious adverse effects)
2. PB: Sulfamethoxazole + Trimethoprim (cotrimoxazole-supra-additive synergism due to
sequential blockade in bacterial folate metabolism)
TIPS
• DOC in Chancroid - cotrimoxazole (800+160)BD x 7 days / erythromycin 2 gm /day x 7
days
• DOC in Pneumocystis carinii - cotrimoxazole(800+ 160) BD 4-6 times
• DOC inToxoplasmosis - pyrimethamine + sulfadiazine
• Sulphonamide used in ophthalmia neonatorum-Sulfacetamide
• Sulphonamide used in the presence of pus-Mafenide
• ADRs of Sulfonamides-RASH ABC(RAsh, SLE, Hemolysis in G6PD, Aplastic anemia, B
ilirubin displacement{Kernicterus}, Crystaluria)

QUINOLONES
1. Nalidixic acid uses
2. Fluroquinolones - Ciprofloxacin {Class of drug, MOA, Kinetics, Interactions, Spectrum,
Adverse effects, C/I, Uses }(Tripathi pg 688)
3. Sparfloxacin, Levofloxacin, Gatifloxacin , Ofloxacin
4. Rx typhoid ( also in pregnancy)
P.B
1. PB: Pefloxacin/Ciprofloxacin in meningitis
- Pefloxacin
- Passage into CSF is higher than other FQs

TIPS

• DOC in Typhoid - ciprofloxacin 500mg BD x 10 day


• Urinary antiseptics-Nalidixic acid, Nitrofurantoin, Methenamine
• F.Q effective against M.leprae-Ofloxacin
• F.Q effective against Atypical MB and anaerobe-Sparfloxacin

90
GOVT MEDICAL COLLEGE, TRIVANDRUM
LUMINAIRE-2015
• F.Q causing QT prolongation-Lomefloxacin, Sparfloxacin, Gatifloxacin

BETALACTAM ANTIBIOTICS
1. MOA, Classification
2. Benzyl Pencillin ( Class of drug, MOA, Kinetics, Interactions, Spectrum, Adverse effects, C/I,
Uses)
3. Semi synthetic pencillins - classification
4. Repository Pencillin G injection
5. Ampicillin, Amoxicillin
6. Rx of MRSA
7. β Lactamase inhibitors
8. Anti pseudomonal penicillins (Carbenecillin, Ticarcillin, Piperacillin)
9. Cephalosporins- Classification, Adverse effects, Uses
10. Ceftriaxone , Cefuroxime , Cefotaxime
P.B
• PB: Amoxicillin + Clavulanic acid, Augmentin
• PB: Ampicillin + Sulbactum
• PB: Procaine Pencillin should not be used as IV(I.V. Inj. Of Procaine Penicillin produces CNS
stimulation, hallucination & convulsions due to Procaine Causes microembolism)
• PB: Procaine Pencillin + Probenecid –Beneficial (Probenecid competes for tubular secretion &
thus retards excretion of Penicillin/Cephalosporin- Increases the blood levels and volume of
distribution)
• PB: Imipenem + Cilastatin in hospital acquired infection –Beneficial -Cilastatin inhibits
Dihydropeptidase I on the brush border of renal tubular cells, which rapidly metabolises
Imipenem
TIPS

• DOC in A/C rheumatic fever – Penicillin


• DOC in Syphilis-penicillin
• DOC in Anthrax, actinomycosis, trench fever - PnG
• DOC in Rheumatic fever (prophylaxis) - benzathine penicilline 1 - 2 mg every 4 weeks
• DOC in Meningitis (pseudomonas) - ceftazidime + gentamycin
• DOC in Meningitis(H.infl, enterobact) - ceftriazone, cefuroxime
• DOC in Gonorrhea- non penicillinase producing organisms -amoxicllin3.5 g oral /
procaine penicillinG 4.8 mu i .m ; penicillinase producing - ceftriaxone 250mg i.m
• Cefazolin-Surgical Prophylaxis

TETRACYCLINE AND CHLORAMPHENICOL


1. MOA of Tetracycline and Cholramphenicol
91
GOVT MEDICAL COLLEGE, TRIVANDRUM
LUMINAIRE-2015
2. Tetracycline- Spectrum, Uses and Adverse Effects
3. Choloramphenicol- Uses and Adverse Effecta
TIPS
• DOC in Atypical pneumonia - tetracycline / erythromycin
• DOC in Cholera, plague, relapsing fever, rickettsial infection - tetracycline
• DOC in Brucellosis - doxycycline 200mg/dayx 6 weeks
• Minocycline-Vestibular toxicity
• Tetracycline with enterohepatic circulation-Doxycycline
• Advantages of Doxycycline over Tetracycline

AMINOGLYCOSIDE
1. Classification , MOA of Aminoglycoside
2. Toxicity of Aminoglycosides, Mechanism of resistance
3. Neomycin , Gentamycin , Streptomycin , Amikacin
P.B
1. PB: Aminoglycoside + Amphotericin B not used – Adverse- Additive nephrotoxicity
2. PB: Aminoglycoside + Frusemide not used – Adverse - Additive ototoxity
3. PB: Gentamycin + Penicillin in SABE
- Beneficial
- Penicillin inhibits cell wall synthesis & facilitate entry of Gentamicin
- Other relevant points from “Combination Of Antibiotics”

TIPS

• DOC in Tularemia-streptomycin
• DOC in SABE-penicillin/gentmicin
• Aminoglycoside with widest spectrum of action-Amikacin(resistant to inactivating enzymes)
• AG with highest nephrotoxicity-Neomycin
• AG with lowest nephrotoxicity-Streptomycin
• AG used topically-Neomycin, Framycetin

MACROLIDES AND OTHER ANTIBACTERIALS


1. MOA of Macrolides
2. Erythromycin {Class of drug, MOA, Kinetics, Interactions, Spectrum, Adverse effects, C/I,
Uses}
3. Azithromycin
4. Clarithromycin
5. Clindamycin (causes Cl. difficile superinfection)

92
GOVT MEDICAL COLLEGE, TRIVANDRUM
LUMINAIRE-2015
7. Vancomycin - Redman’s syndrome
8. Linezolid (MOA)(
9. Rx of UTl
TIPS
• DOC in Whooping cough - erythromycin
MRSA- vancomycin , Linezolid
Chlamydia trachomatis - azithromycin 1 g oral OD
Pseudomembanous enterocolitis – vancomyin, metronidazole
• Uses of Macrolides-CLAW(Chancroid, Legionella, Atypical pneumonia, Whooping cough)
• Prokinetic macrolide antibiotic-Erythromycin
• Erythromycin esteolate salt causes cholestatic jaundice

ANTI TB DRUGS
1. Treatment of TB – DOTS- Essay
2. Rifampicin {Class of drug, MOA, Kinetics, Interactions, Spectrum, Adverse effects, C/I, Uses}
3. INH , Pyrazinamide , Ethambutol
4. Adverse effects of anti TB drugs
5. PB of employing multiple drug therapy
6. TB in pregnancy
7. TB in AIDS
8. 2nd line drugs- Cycloserine, Clarithromycin
9. Role of steroids in TB
TIPS
• A/E of INH-Hepatotoxicity, neuropathy
• A/E of Pyrazinamide-Hyperuricemia
• A/E of Ethambutol-Retrobulbar neuritis, hyperuricemia
• Rifampicin-Uses-TB, leprosy, prophylaxis of streptococcal nd meningococcal carriers,
A/E-Orange-red discoloration of urine
• Streptomycin-Only parenteral anti-TB drug, primary anti TB drug C/I in pregnancy
• Standard drug for chemoprophylaxis of TB-INH 300mg daily , 6 to 12 months
• Rifampicin is the safest drug in renal failure
• Rifampicin- active against persisters

ANTI LEPROTICS
1. Dapsone - MOA . Uses , Adverse Effects
2. Clofazimine ( in lepra reaction)
3. MDT of Leprosy
4. Reactions in leprosy, Management

93
GOVT MEDICAL COLLEGE, TRIVANDRUM
LUMINAIRE-2015

TIPS
• Leprostatic drug with anti-inflammatory property- clofazamine
• Dapsone- DOC of dermatitis herpetiformis
-produces sulfone syndrome
• DOC for type 1 and type 2 lepra reaction- steroid
• THALIDOMIDE- gold std for treatment of type 2 reaction

ANTI FUNGALS
1. Amphotericin B
2. Nystatin
3. Griseofulvin {Class of drug, MOA, Kinetics, Interactions, Spectrum, Adverse effects, C/I,
Uses}(Tripathi pg 760)
4. Clotrimazole
5. Ketoconazole {Class of drug, MOA, Kinetics, Interactions, Spectrum, Adverse effects, C/I
, Uses )
6. Fluconazole
7. Terbinafine
8. Mention 2 uses and adverse effects of Fluconazole
P.B
1. PB Terfinadine/ Astemizole/ Cisapride +Ketoconazole (ketoconazole causes excessive rise in
plasma levels of these drugs→polymorphic ventricular tachycardia)
2. PB. Amphotericin B+ Flucytosine in mycosis
- Beneficial
- Amphotericin B increses the permeability of fungal cell membrane by forming pores 7
channels and facilitate the entry of Flucytosine
- Other relevant points from “Combination Of Antibiotics”
3. PB Fluconazole in fungal meningitis (good CSF penetration)
- Good CSF penetration & less toxic

TIPS

• Antifungal drug with disulfiram like reaction-Griseofulvin


• AF with anti-androgenic action-ketoconazole
• Luminal antibiotic with antifungal and antibacterial- Quiniodochlor
• Dose limiting activity of Amphotericin B- Nephrotoxicity
• Doc for candidiasis, coccidioidal and cryptococcal meningitis- Fluconazole

94
GOVT MEDICAL COLLEGE, TRIVANDRUM
LUMINAIRE-2015
• 2 drugs for superficial mycosis- AMB, Nystatin
• 2 drugs used in dermatophytosis- Griseofulvin, Ketoconazole
• DOC for Blastomycosis, Histoplasmosis, coccidiomycosis, paracoccidiomycosis,
sporotrichosis-Itraconazole
• DOC for Invasive aspergillosis-Voriconazole

ANTI VIRAL DRUGS


1. Acyclovir
2. Classify AntiRetroviral Drugs
2. HAART
3. Zidovudine
4. Anti-influenza drugs (amantadine,oseltamivir)
5. NNRTIs
6. Amantidine- uses
7. Interferon alpha
8. Protease inhibitors
TIPS
• DOC in Herpes infection -acyclovir
CMV infection - gancyclovir
• Acyclovir-Least toxic antiviral drug, prophylaxis for recurrent genital herpes
• Gancyclovir-used only i/v.A/E- Myelosuppression, CNS S/E(Headache to convulsions)
• Stavudine- NRTI causing peripheral neuropathy, lactic acidosis
• NRTI causing bone marrow suppression-Zidovudine
Pancreatitis-Didanosine
Increased risk of MI-Abacavir

ANTI MALARIAL DRUGS


1. Classify AntiMalarial Drugs
2. Chloroquine
3. RX of cerebral malaria
4. Primaquine
5. Artemisinine derivatives
6. ACT
7. Diagram showing site of action of drugs
P.B
1. Quinine,why is it given in 5%dextrose
- Beneficial
- Quinine produces hypoglycemia due to hyperinsulinemia
2. Chloroquine+Primaquine

95
GOVT MEDICAL COLLEGE, TRIVANDRUM
LUMINAIRE-2015
- Beneficial
- Chloroquine is mainly blood schizonticidal (Erythrocytic stage) whereas Primaquine is
tissue schizonticidal (Pre-erythrocytic stage & Hypnozoites) & gametocidal, providing
complete cure

TIPS

• DOC in F. malaria- Chloroquine 600mg stat followed by 300mg daily - next 2 days.
Cerebral malaria - Quinine i. v
• DOC for radical cure of vivax and ovale malaria-Primaquine
• DOC for malaria in pregnancy-Chloroquine
• Chloroquine uses-MALARID(MAlaria, Lepra reactions, Amoebiasis, Rheumatoid arthritis,
Infectious mononucleosis, Discoid lupus erytthematoses)
• Chloroquine causes Bull’s eye maculopathy

ANTI AMOEBIASIS
1. Classification of Anti Amoebic Drugs
2. Metronidazole Class of drug, MOA, Kinetics, Interactions, Spectrum, Adverse effects, C/I,
Uses}
3. Diloxanide furoate
4. Pentamidine
P.B
1. PB.Metronidazole +Diloxanide furoate in intestinal amoebiasis
- Beneficial
- Metronidazole is a tissue amoebicide whereas Diloxanide furoate is a luminal
amoebicide, provoding complete cure

TIPS
DOC in Trichomonas vaginitis and Dracunculus mediensis- Metronidazole 400mg TDS
x 7 days
• DOC in Anaerobic infection-Metronidazole
• DOC for asymptomatic amoebiasis and effective against C/c cyst carrier-Diloxanide
furoate
• DOC for kala-azar-Sodium stibogluconate Drug used for extraintestinal
amoebiasis-Chloroquine

ANTI HELMINTHICS
1. Albendazole
2. Pyrantel pamoate
3. Piperazine

96
GOVT MEDICAL COLLEGE, TRIVANDRUM
LUMINAIRE-2015
4. DEC
5. Ivermectin
6. Niclosamide
7. Praziquantel
TIPS
• Filariasis-DEC2mg/kgTDSx7days
• Tropical eosinophilia -2-4 mg/kg DEC TDS x 2-3 wks
• Albendazole is the DOC for most nematode infections except filariasis(DEC), onchocerca
and strongyloides(Ivermectin)
• Praziquantel is the DOC for all trematode and cestode infections except fasciola(Bithionol)
and hydatid(Albendazole)
• Tinea infection-Niclosamide
• Anti-helminthic acting as immunomodulator-Levamisole
• Only orally effective drug for pediculosis and scabies-Ivermectin

ANTI-CANCER DRUGS
1. Toxicity of cytotoxic drugs
2. Cell cycle specific anti neoplastic agents
3. Alkylating agents
4. Cyclophosphamide,
5. Busulfan, Chlorambucil
6. Methotrexate
7. Leucovorin rescue
8. Mercaptopurine, Azathioprine
9. Anti cancer antibiotics
10.Vinca alkaloids
11. Paclitaxel
12. Camptothecin analogues
13. L- asparaginase,
14.Cisplatin
15.Hormonal treatment of cancer
16. Radioisotopes, Biological Response Modifiers (Molgramostim)(Tripathi pg 833)

P.B
1. P.B-Imatinib in CM (inhibits tyrosine protein kinases in CML cells)
2. PB. Mercaptopurine +Allopurinol (6-MP meta by xanthine oxidase, which is inhibited by
allopurinol;dose must be reduced if allopurinol given concurrently)
TIPS
97
GOVT MEDICAL COLLEGE, TRIVANDRUM
LUMINAIRE-2015
• DOC in Hemorrhagic cystitis - MESNA
• DOC in Multiple myeloma-Melphalan
• DOC in C/c lymphoid leukaemia—Chlorambucil
• DOC in C/c myeloid leukaemia—Busulphan
• DOC in Ca breast—Tamoxifen,Methotrexate
• DOC in Ca cervix—Mitomycin C,methotrexate
• Cardiotoxic anticancer antibiotics- Doxorubicin, Daunorubicin
• Most common side effect of Cyclophosphamide- Hemorrhagic cystitis
• Glove and stocking neuropathy is the s/e of- Paclitaxel

IMMUNOSUPPRESSANTS
1. Cyclosporine
2. Tacrolimus
2. Immunosuppressant antibodie
4. Drugs used for graft rejection- Methyl prednisolone, Muromonab

CHELATING AGENTS
1. BAL
2. D- Penicillamine
3. Desferrioxamine
4. Desferrioxamine/ Dimercaprol in Fe poisoning
NAME TWO DRUGS
1. Given Sublingually – GTN , Buprenophrine
2. Given Rectally – Diazepam , Indomethacin
3. Given as Transdermal patches - Nicotine , Fentanyl
4. Microsomal Enzyme Inhibitors – Erythromycin , Ketoconazole
5. Microsomal Enzyme Inducers - Phenobarbitone , Rifampicin
6. Low First Pass Metabolism – Phenobarbitone , Theophylline
7. High First Pass Metabolism- Lignocaine , Isoprenaline, Propranolol
8. Zero Order Kinetics – Phenytoin , Warfarin
9. Low Safety Margin – Digoxin , Anticonvulsants , Lithium
10. Competitive Inhibiton – Neostigmine , Sulfonamides , Captopril
11. Non Competitive Inhibition – Acetazolamide , Aspirin
12. Therapeutic Window Phenemenon- Tricyclic Antidepressants , Clonidine
13. Tachyphylaxis – Tramine , Ephedrine , Nicotine
14. Idiosyncrasy – Barbiturates , Quinine , Chloramphenicol
15. Phototoxicity – Tetracycline , Amiodarone
16. Teratogenicity – Thalidomide , Sodium Valproate , phenytoin

98
GOVT MEDICAL COLLEGE, TRIVANDRUM
LUMINAIRE-2015
17. Centrally Actind anti-AChE – Rivastigmine , Donapezil
18. Treatment of Alzhimers – Rivastigmine , Donapezil
19. Cobra bite , Post Operative Decurarisation – Neostigmine + Atropine
20. AntiCholinesterase Poisoning – Atropine and Pralidoxime
21. Miotics – Pilocarpine , Physostigmine
22. Mydriatics – Atropine , Cyclopentolate , Tropicamide
23. Vasicoselective AntiCholinergics – Oxybutynin , Flavoxate
24. Centrally Acting Anticholinergics – Benzhexol , Procyclidine (AntiParkinsonian)
25. Anticholinegics used in Bronchial Asthma – Ipratropium Bromide , Tiotropium Bromide
26. Anticholinergics used in Peptic Ulcer – Pirenzepine , Propantheline
27. Anticholinergics used in motion sickness – Hyoscine , Dicyclomine
28. Anticholinergics used in Pre Anaesthetic Medication – Glycopyrrolate , Atropine
29. Selective β2 Agonists – Salbutamol , Terbutaline
30. Selective β1 Antagonists – Atenolo , Metoprolol
31. Selective α1 Agonists- Phenylephrine , Methoxamine
32. Nasal Decongestants – Phenylephrine , Xylometazoline , Oxymetazoline
33. Indirectly Acting Sympathomimetics – Amphetamine , Tyramine , Methamphetamine
34. Anoretics – Fenfluramine , Dexfenfluramine
35. Uterine Relaxants – Isoxsuprine , Ritodrine
36. Selective α1 Blockers – Prazosin , Terazosin
37. Treatment of BPH – Prazosin , Terazosin
38. Selective α2 Blockers – Yohimbine , Rauwolscine
39. Non selective β blockers – Propranolol , Timolol
40. Cardio selective β blockers (β1Selective blocker ) – Metoprolol , Atenolol
41. β + αAction – Labetalol , Carvedilol
42. Used for Open Angle Glaucoma – Timolol , Betoxolol , Latanoprost
43. Used for Angle Closure Glaucome – Hypertonic Mannitol , Acetazolamide
44. Carbonic Anhydrase Inhibitor – Acetazolamide , dorzolamide
45. H1 receptor Antagonists – Promethazine , Diphenhydramine , Dimenhydrinate
46. Second Generation Antihistaminics – Certrizine , Loratidine , Azelastine
47. Vertigo – Cinnarizine , Betahistine
48. H2 Antagonists – Cimetidine , Ranitidine
49. 5HT3 Antagonists – Ondansetron , Granisetron
50. Ergot Alkalois – Ergotamine , Ergometrine
51. Migraine – Sumatriptan , Ibuprofen
52. Prophylaxis of Migraine – Beta blocker + CCB + Amitriptyline
53. 5HT2 Antagonists – Ketanserin , Cyproheptadine , Methysergide

99
GOVT MEDICAL COLLEGE, TRIVANDRUM
LUMINAIRE-2015
54. Postpartum Haemorrhage – Carboprost , Dinoprost
55. Abortion – Mifepristone , Misoprostol
56. Induction of Labour – Misoprostol , Dinoprostone
57. Preferential COX 2 Inhibitors – Nimesulide , Meloxicam
58. Selective COX 2 Inhibitors – Etoricoxib , Parecoxib (Gastro Protective)
59. Topical NSAIDS – Diclofenac , Ibuprofen
60. Acute Musculoskeletal and osteoarthritic pain – Paracetamol , Diclofenac
61. DMARDS – Methotrexate , Sulphasalazine
62. Biologic Response Modifiers – Etanercept , Infliximab , Anakinra
63. Acute Gout – NSAIDs , Cochicine
64. Uricosuric Drugs – Probenecid , Sulfinpyrazone
65. Uric acis synthesis Inhibitor – Allopurinol , Febuxostat
66. Pharyngeal Demulcents – Lozenges , Linctuses
67. Mucolytics – Bromhexine , Ambroxol ,Acetylcystiene
68. Antitussives – Codiene , Dextromethorphan
69. Broncho Dilalators – Salbutamol , Salmeterol
70. Anticholinergics used in Bronchial Asthma – Ipratropium Bromide , TioTropium Bromide
71. Leukotriene Antagonists – Montelukast , Zafirukast
72. Mast Cell Stabilizers – Na Cromoglycate , Ketotifen
73. Inhalational Corticosteroids in Bronchial Asthma – Fluticasone , Budesonide
74. GH Inhibitors – Somatostatin , Octreotide
75. Prolactin Inhibitors – Bromocriptine , Cabergoline
76. GnRH Agonists – Gonadorelin , Triptorelin , Nafarelin
77. GnRH Antagonists- Ganirelix , Cetorelix
78. AntiThyroid Drugs – PropylThiouracil , Methimazole
79. Inhibit Iodide Trapping – Thiocyanates , Perchlorates
80. Radioactive Iodine – I 131 , I 125
81. Inhibit peripheral conversion f T4 to T3 – Propylthiouracil , Propranolol , Amiodarone
82. Treatment of Thyrotoxicosis – PropylThiouracil , Propranolol
83. Rapid Acting Insulin – Insulin Lispro , Insulin Aspart
84. Long Acting Insulin – Insulin Glargine , Insulin Detimer
85. Intermediate Acting Insulin – Insulin Lente , Isophane Insulin
86. First Generation SulfonylUrea – Tolbutamide , Chlorpropamide
87. Second Generation SulfonylUrea- Glibenclamide , Glipizide
88. BIguanide-Metformin , Phenformin
89. Thiazolidinediones – Rosiglitazone , Pioglitazone
90. α glucosidase inhibitor – Acarbose , Miglitol
91. Meglitinide – Rosiglitazone , Pioglitazone
92. Treatment of Ulcerative Colitis- Hydrocortisone , MethylPrednisolone
100
GOVT MEDICAL COLLEGE, TRIVANDRUM
LUMINAIRE-2015
93. Treatment of Cerebral Edema – Dexamethasone , Betamethasone
94. Treatment of Adrenal Insufficiency – Hydrocortisone , Dexamethasone
95. Anabolic Steroids – Nandrolone , Stanozolol , Oxymetholone
96. AntiAndrogens- Flutamide , Danazol
97. 5α reductase Inhibitor – Finasteride , Dutasteride
98. Treatment of Erectile Dysfunction – (PDE-5 inhibitors) – Sildenafil , Tadalafil
99. Synthetic Estrogens – Ethinyl Estradiol , Diethylstilbestrol
100. AntiEsrtogens – Clomiphine Citrate , Fulvestrant
101. SERMs – Tamoxifen , Raloxifene
102. Aromatase Inhibitors – Letrozole , Anastrozole
103. Antiprogestins – Mifepristone , Onapristone
104. Tocolytics – Isoxsuprine , Ritodine , Nifedipine , Atosiban
105. Bisphosphonates – Alendronate , Zoledronate
106. Treatment of Osteoporosis – Alendronate , Raloxifene
107. Non depolarizing Neuromuscular blockers- Tubocurarine , Pancuronium
108. Depolarizing Neuromuscular blockers – Succinylcholine , Decamethonium
109. Directly Acting Skeletal Muscle Relaxants – Dantrolene sodium , Quinine
110. Injectable Local Anaesthetic – Procaine , Lignocaine , Tetracaine , Bupivacaine
111. Surface Anaesthetics – Lidocaine , Tetracaine , Benzocaine
112. Inhalational GAs – Nitrous Oxide . Halothane , Isoflurane
113. Intravenous GAs Inducing Agents – Thiopentone Sodium , Propofol
114. Disulfuram Like Reaction – Metronidazole , Gresiofulvin
115. Withdrawal Syndrome – Naltrexone , Acamprostate
116. Nonbenzodiazepine Hypnotics ( Z hypnotics ) – Zopiclone , Zolpidem
117. Long Acting Benzodiazepene – Diazepam , Flurazepam
118. Short Acting Benzodiazepenes – Alprozolam , Triozolam
119. Generalised Tonic Clonic Seizures (Grand Mall Epilepsy) – Sodium Valproate ,
Carbamezepine
120. Absence Seizure – Ethosuximide , Sodium Valproate
121. Simple Partial Seizures- Sodium Valproate , Carbamezepine
122. Complex Partial Seizures – Sodium Valproate , Carbamezepine
123. Newer Antiepileptics – Vigabatrine , Topiramate
124. Myoclonic Seizures- Sodium Valproate , Lamotrigine
125. Atonic Seizures – Valproate , Clonazepam
126. Trigeminal Neuralgia – Carbamezepine (DOC)
127. Status Epilepticus – Lorazepam , Phenytoin
128. Peripheral Decarboxylase Inhibitor – Carbidopa , Benserazide
129. Dopaminergic Agonists – Bromocriptine , Ropinirole , Pramiprexole
130. MAO-B Inhibitors – Selegiline , Rasagiline
101
GOVT MEDICAL COLLEGE, TRIVANDRUM
LUMINAIRE-2015
131. MAO- A Inhibitors (RIMA) – Moclobemide , Clorgyline
132. COMT inhibitors – Entacapone , Tolcapone
133. Centrally Acting Anticholinergics in Parkinsonism – Benhexol , Procyclidine
134. Antipsychotics (Neuroleptics) – Chlorpromazine , Fluphenazine , Haloperidol
135. Atypical Antipsychotics- Clozapine , Risperidone , Olanzapine , Quetiapine
136. Schizophrenia - Haloperidol , Risperidone , Quetiapine
137. Mania- Haloperidol , Lithium , Olanzapine
138. Bipolar Disorders – Lithium , Carbamezepine
139. Hallucinogens – LSD , Cannabinoids
140. TCAs – Amitriptyline , Imipramine
141. SSRI- Fluoxetine , Fluoxamine , Citalopram
142. SNRI – Venalafaxine , Duloxetine
143. Atypical Antidepressants SNRI , Trazodone
144. AntiAnxiety – Diazepam , Buspirone , Hyroxyzine
145. Natural Opium Alkalois – Morphine , Coedine
146. Semisynthetic Opiates – Pholcodiene , DiacetylMorphine
147. Synthetic Opiods – Pethidine , Fentanyl , Tramadol
148. Opioid Antagonist – Naloxone , Naltrexone
149. CNS stimulants – Amphetamine , Modafinil
150. ACE Inhibitors – Captopril , Lisinopril
151. ARBs – Losartan , Candesaratan
152. Direct Renin Inhibitors – Aliskiren , Remikiren
153. Cardiac Glycosides – Digoxin , Digitoxin
154. Inodilators ( PDE-3 inhibitors) – Milrinone ,AMrinone
155. Used In CHF – Digoxin , Carvedilol
156. PSVT – Adenosine , Propranolol
157. Widens Action Potential – Amiodarone , Bretylium
158. AV block – Isoprenaline , Atropine
159. Prolong QT interval – Quinidine , Sparfloxacin ,Terfenadine
160. Venticular Fibrillation – Lignocaine , Amiodarone
161. Atrial Fibrillation – Esmolol , Verapamil
162. Nitrates – GTN , Isosorbide mononitrate
163. CCBs – Verapamil , Diltiazem , Nifedipine
164. Potassium Channel Openers – Nicorandil , Minoxidil
165. Central Sympatholytics – Clonidine , MethylDopa
166. Antihypertensives Safe in Pregnancy – Hydralazine , MethylDopa
167. Antihypertensive Unsafe In Pregnancy – Diuretics , ACE inhibitors , Propranolo
168. Hypertensive Emergencies – Sodium Nitroprusside , GTN , Hydralazine
169. High Ceiling Diuretics – Furosemide , Bumetanide
102
GOVT MEDICAL COLLEGE, TRIVANDRUM
LUMINAIRE-2015
170. Thiazide Diuretics- Hydrochlorthiazide , Benzthiazide
171. Thiazide Like Diuretics – Chlorthalidone , Xipamide
172. Potassium Sparing Diuretics – Spirinolactone , Triamterene
173. Epithelial Sodium Channel Inhibitors – Triamterene , Amiloride
174. Osmotic Diuretics – Mannitol , Glycerol
175. Aldosterone Antagonists – Spirinolactone , Elperenone
176. Vasopressin Analogues – desmopressin , Terlipressin
177. Oral Iron Preparations – Ferrous Sulfate , Ferrous Gluconate
178. Parentral Iron – Iron Dextran , Iron Sorbitol Citric Acid
179. Vit B12 Preparations – Cyanocobalamin , Hydroxycobalamin
180. Synthetic Vitamin K – Menadione , Acetomenaphthone
181. Parentral Anticoagulants – Heparan Sulfate , Heparin , LMW Heparin
182. Oral Anticoagulants – Dicumarol , Warfarin Sod
183. LMW heparin – Enoxaparin , Reviparin
184. Fibrinolytics (Thrombolytics)- Streptokinase , Alteplase
185. Antifibrinolytics – Epsilon amino Caproic Acid , Tranexaemic Acid
186. Antiplatelet Drugs (AntiThrombotic) – Apirin , Dipyridamole , Clopidogrel
187. GP IIb/IIIa receptor antagonists – Abciximab , Eptifibatide
188. HMG-CoA reductase inhibitors – Atorvastatin , Rosuvastatin
189. Bile acid sequestrants – Cholestyramine , Colestipol
190. Activate lipoprotein Lipase (Fibric Acid Derivatives )- Gemfibrozil , Fenofibrate
191. Plasma Expanders – Human Albumin , Dextran
192. H2 AntiHistaminics – Cimetidine , Ranitidine
193. Proton Pump Inhibitors – Omeprazole , Pantoprazole
194. Systemic Antacids – Sodium bicarbonate , Sodium Citrate
195. Nonsystemic Antacids – Magnesium Hydroxide , Aluminium Hydroxide
196. Ulcer Protectives – Sucralfate , Colloidal Bismuth subcitrate
197. Anti-H pylori Drugs – Amoxicillin , Clarithromycin , Tinidazole
198. Prokinetic Drugs – Metoclopramide , Domperidone , Cisapride
199. Motion Sickness – Hyoscine , DIcyclomine, Promethazine
200. GERD – Omeprazole , Ranitidine
201. 5HT4 Agonists – Cisapride , Tegaserod
202. Carminatives – Sodium Bicarbonate , Oil Peppermint
203. Gall stone dissolving Drugs – Chenodiol , Ursodiol
204. Bulk forming Laxatives – Dietary Fibre : Bran , Methylcellulose
205. Stool Softners – Docusates , Liquid Paraffin
206. Stimulant Purgatives – Bisacodyl , Phenolpthalein , tegaserod
207. Osmotic Purgative s- Lactulose , Sod. Pot. Tartrate
208. Travellers Diarrhoea – Cotrimoxazole , Norfloxacin , Doxycycline
103
GOVT MEDICAL COLLEGE, TRIVANDRUM
LUMINAIRE-2015
209. Shigella Enteritis – Ciprofloxacin , Nalidixic Acid , Cotrimoxazole
210. Antisecretory Drugs used in Diarrhoea – Sulfasalazine, Mesalazine
211. Anti Motility Drugs Used in Diarrhoea- Diphenoxylate , Loperamide
212. Cause Superinfection ( Pseudomembraneous Enterocolitis) – Aminoglycosides ,
Clindamycin
213. Treatment of Pseudomembraneous Enterocolitis – Metronidazole , Vancomycin
214. UTI and Enteritis caused by Proteus – Cefixime , Gentamicin
215. Psedomonas Infection – Carbenicillin , Piperacillin
216. Drugs that show Post Antibiotic Effect – FQs , Aminoglycosides
217. Surgical Site Infection – Cefazolin , Vancomycin
218. Infection on burn surfaces – Silver sulfadizine , Mafenide
219. First Generation Fluroquinolone – Norfloxacin , Ciprofloxacin , Ofloxacin
220. Second Genaration Fluroquinolone – Levofloxacin , Sparfloxacin , Gatifloxacin
221. Chancroid – Ceftriaxone , Erythromycin , Ciprofloxacin
222. Typhoid – Ciprofloxacin , Ceftriaxone
223. Fluroquinolones that prolong QT interval – Gatifloxacin , Sparfloxacin , Moxifloxacin
224. MRSA – Vancomycin , Linezolid
225. Penicillinase resistant Penicillins – Methicillin , Cloxacillin
226. Extended Spectrum Penicillins – Ampicillin , Amoxycillin ,
227. β Lactamase Inhibitors – Clavulanic Acid , Sulbactam
228. Anti Pseudomonal Penicillins – Carbenicillin , Piperacillin
229. First Generation Cephalosporins – Cefazolin , Cephalexin
230. Second Generation Cephalosporins- Cefuroxime , Cefuroxime axetil
231. Third Generation Parenteral Cephalosporins – Cefotaxime , Ceftriaxone , Cefoperazone
232. Third Generation Oral Cephalosporins – Cefixime , Cefpodoxime
233. Fourth Generation Cephalosporin – Cefepime , Cefpirome
234. Carbapenems – Imipenem , Meropenem
235. Topical Aminoglycoside s- Neomycin , Framycetin
236. Lincosamide Antibiotics – Clindamycin , Lincomycin
237. Glycopeptide Antibiotics – Vancomycin , Teicoplanin
238. Urinary Antiseptics – Nitrofurantoin , Nalidixic Acid
239. MAC – Clarithromycin , Ethambutol , Rifabutin , Ofloxacin
240. Lepra Reaction – Clofazimine , Prednisolone, Thalidomide
241. Multibacillary Leprosy – Rifamoin , Dapsone , Clofazimine
242. Paucibacillary Leprosy – Rifampi , Dapsone
243. Topical Antifungals – Clotrimazole , Econazole , Miconazole
244. Systemic Antifungals – Fluconazole , Voriconazole , Ketoconazole
245. Antiviral Drugs – Acyclovir , Valacyclovir
246. Cytomegalovirus – Ganciclovir , Foscarnet
104
GOVT MEDICAL COLLEGE, TRIVANDRUM
LUMINAIRE-2015
247. NRTI – Zidovudine , Stavudine , Lamivudine
248. NNRTI – Nevirapine , Efavirenz
249. Protease Inhibitor – Ritonavir , Indinavir , Saquinavir
250. Anti- Influenza Drugs – Amantidine , Oseltamivir
251. Antimalarial Drugs – Chloroquine , Primaquine
252. Vivax Malaria – Cholroquine , Primaquine
253. Cholroquine resistant vivax malaria – Quinine , Doxycycline , Primaquine
254. Falciparum Malaria – Artesunate , Sulfadoxine , Pyrimethamine
255. Severe and Complicate Falciparum Malaria – Artesunate , Artemether
256. Tissue Amoebicides – Metronidazole , Tinidazole
257. Luminal Amoebicides – Diloxanide Furoate , Nitazoxanide
258. Giardiasis – Metronidazole , Tinidazole , Nitazoxanide
259. Trichomonas Vaginitis – Metronidazole , Tinidazole , Nimorazole
260. Leishmaniasis – Sodium Stibogluconate , Pentamidine
261. Ascariasis – Mebendazole , Albendazole
262. Filariais – DEC , Ivermectin , Albendazole
263. Tapeworm – Praziquantel , Niclosamide , Albendazole
264. Hydatid Disease , WhipWorm , Hookworm – Albemdazole , Mebendazole
265. Alkylating Agents – Cyclophosphamide , Chlorambucil ,Busulfan
266. Antimetabolites – Methotrexate , Azathioprine
267. Vinca Alkaloids – Vincristine , Vinblastine
268. Taxanes – Paclitaxel , Docetaxel
269. Anticancer Antibiotics – Actinimycin D , Doxorubicin , Mitomycin C
270. Cell Cycle Nonspecific Anticancer Drugs – Cyclophosphamide , Chlorambucil , Cisplatin
271. Cell Cycle Specific – Vinblastine , Vincristine , Methotrexate , Paclitaxel
272. S phase selective Cytotoxic Drugs – Methotrexate , 5 Fluro Uracil , Doxorubicin
273. G2 phase selective cytotoxic Drugs – Bleomyci , Etoposide , Topotecan
274. M phase Selective Cytotoxic Drugs – Vincristine , Vinblastine , Paclitaxel
275. Hodgkins Lymphoma – Vinblastine , Doxorubicin , Dacarbazine
276. Breast CA – Tamoxifen , Methotrexate
277. Ewings Sarcoma – Vincristine , Doxorubicin
278. CML – Imatinib , Busulfan
279. Acute Leukemias – Daunorubicin , Cyclophosphamide
280. Calcineurin Inhibitors (Specific T cell Inhibitors) – Cyclosporine , Tacrolimus
281. Immunosuppresants – Cyclosporine , Tacrolimus , Azathioprine
282. Chelating Agents – BAL , Penicillamine , Disodium Edetetate

ESSAYS
PAPER 1
105
GOVT MEDICAL COLLEGE, TRIVANDRUM
LUMINAIRE-2015
1) Devi had prosthetic valve replacement 2 yrs back. She was prescribed some drugs
which she was taking regularly, she developed bleeding from the gums and on investigation
prothrombin time was prolonged 5 times the control.
a) What is the drug she is taking
b) How will you manage the case
c) Name 2 other drugs which has same action
d) Mention an important drug interaction of this drug
2) Latha 40 yrs was brought to doctor with complaints of stiffness of body and tremor for
3 days, she has been taking chlorpromazine for the last 1 month for some mental illness. The
Dr. diagnosed it as drug induced parkinsonism
a) Explain the mechanism of this side effect of CPZ
b) How will you manage
c) Name 3 other drugs used in parkinsonism with MOA
d) Role of carbidopa in parkinsonism
e) Anti emetic which can induce parkinsonism

3) 65 yr old male admitted in hospital. O/E his BP was 150/90, LDL 250mg/dl, he is a
known asthmatic on salbutamol inhalation
a) Which anti hypertensive will you use on this patient? Why
b) Name anti hypertensives to be avoided in this patient with reasons
c) which drug will you use to lower his LDL levels what is its MOA?
d) Name two other drugs that lower LDL levels
e) What life style modification will you advice for this patient
f) Name four drugs used to treat hypertensive emergencies

4) A farmer while spraying and insecticide develops profuse sweating and muscle
weakness with bronchospasm O/E his pulse rate is 68/min and pupils constricted.
a) What type of insecticide could have caused this poisoning? Explain
b) How do you confirm the diagnosis
c) Outline the management of this poisoning

5) 50 yr old female patient suffering from chronic Rheumatoid arthritis, on treatment. He


was brought to hospital with following features tinnitus, nausea, vomiting, sweating,
hypoventilation
a) Diagnosis ?
b) What drug has produced these effects ?
c) Therapeutic effects of this drug in RA
d) Biochemical abnormalities to be present in the patient
e) Outline treatment approach giving pharmacological basis
106
GOVT MEDICAL COLLEGE, TRIVANDRUM
LUMINAIRE-2015

6) Working woman with h/o mild asthmatic attacks in the past. Had an acute atta
bronchoconstriction, repeated self administration from inhaler did not provide relief and
symptoms progressed until she became cyanotic.She was admitted to hospital in severe
respiratory distress. She was given a medication IV which lead to marked improvement of
symptoms in 15 mins
a) What are the probable mediators or bronchial asthma
b) What are the medications commonly used for the treatment of asthma
c) What drug was administered in the hospital room
d) What agents are used as nebulisers
e) What instructions will you give the patient regarding the use of metered dose
inhaler

7) After consuming country made alcohol, people developed vomiting, blurring of vision and
within a few hours they went to coma.
a) How will you account for the symptoms
b) Outline treatment of this condition with rationale

8) 20 yr old man brought to casualty with h/o seizures. His father said he is an epileptic
patient on phenobarbitone which he was taking irregularly. He again developed seizures while
in the casualty
a) Mention dosage schedule and MOA of phenobarbitone
b) What other drugs can you use for long term control of seizures
c) Mention A/E of long term use of phenobarbitone
d) Mention 4 recently developed anti epileptics
e) Which drug will you use to treat the seizure immediately

9) 53 yr old house wife, admitted with complaints of dyspnoea, palpitation and b/l pedal
edema, diagnosed as CCF
a) Enumerate drugs used in CCF
b) Explain PB of each drug usage
c) 3 important A/E of 3 drugs used in this patient

10) 70 yr old patient brought with dyspnoea, froathy sputum, BP 200/120, later patient was
given aminophylline
a) Which drugs will you use. Justify
b) What is the dose, route and rationale of using aminophylline

11) 30 yr old male presentswith severe retrosternal pain precipitated by exertion, he was sweating
107
GOVT MEDICAL COLLEGE, TRIVANDRUM
LUMINAIRE-2015
profusely and was confirmed as having angina pectoris. He was given a tablet under the tongue
a) What can be the drug given by the doctor
b) What are the advantages of this route of administration
c) Write MOA of the above drug
d) Mention other routes by which this drug can be administered
e) Name a poisoning for which this drug can be administered

12) Lady aged 45 weighing 50 kg attented clinic with signs and symptoms of severe
anemia, on investigation she was found to have microcytic hypochromic anemia,
she was started on oral iron but could not continue as sshe developed severe nausea
and vomiting. Her Hb was 69%
a) Name 2 parenteral iron preparation that can be used here
b) What is the total dose of iron required and in how many injections/day can
the treatment be completed
c) Name 4 indications of parenteral iron therapy

13) 40 yr old patient with a/c attack of gout treated with 10 day course of naproxen
a) Why naproxen was given for a/c gout
b) What other drugs can be given for a/c gout
c) Is naproxen sufficient for long term treatment
d) Which drug do you like to give with naproxen
e) Write its MOA

14) A diabetic patient was found to have a BP of 140/100 mm of Hg and was prescribed an
antihypertensive. After one week the patient complained of dry cough.
a) Which is the most probable antihypertensive that can produce cough? Why is
this drug preferred in diabetes?
b) Mention two other indications of the above drug.
c) Mention four other adverse effects of this drug.
d) Name two groups of antihypertensives not indicated in this patient.

15) A 60 year old man with hypertension and type2 diabetes comes for a follow up
He is found to have elevated total cholesterol and elevated LDL levels. The doctor
prescribes some hypolipidemic drugs.
a) Enumerate the different types of hypolipidemic drugs with one example each.
b) Mention two groups of hypolipidemics which can selectively lower
elevated LDL levels.
c) Write their mechanism of action
d) Name one newer hypolipidemic drug which can be combined with statins.
108
GOVT MEDICAL COLLEGE, TRIVANDRUM
LUMINAIRE-2015
Write the mechanism of action of the new drug.

16) Mrs. Lobo , A 30year old female, mother of a 4year old boy, was admitted to the
emergency ward with following complaints of giddiness, headache, slight breathless
ness, amenorrhoea for 5months, fainting attacks. On examination, pulse – 110/min,
BP – 150/104 mmHg. Urine analysis – proteinuria ++ Hb – 9g/dl, CVS – NAD, RS – occasional
crepitations, rhonchi, pregnancy test +. Past history – not a known hypertensive, had
seasonal asthma.
a) Mention safer antihupertensive drugs for her.
b) What advice would you give for her asthma attacks?
c) What drugs with route of administration and duration would you advice to
improve her Hb levels?
d) Enumerate the drugs used in hypertensive emergencies , mentioning the
routes of administrations and precautions.
e) What antihypertensive drugs are avoided in this patient with reasons?

17) Sudha 34years was brought to casuality with history of continuous wheezing
past 3 hours which was not responding to Salbutamol orally or Adrenaline s/c.
a) What is her condition and which drug will you give for immediate relief ?
mention the dose and route of administration.
b) Mention the various drugs in a classified manner used foe wheezing.
c) On chest examination she had chest infection. What drug will you prescribe?
d) If she complains of allegry to jute, what non hormonal preparation will you
advice and what is the method?
e) What is the hormonal inhalation useful for Sudha , and how will you advise
her the use?
f) What is the advantage of this preparation ?what adverse reaction can you
expect in her?

18) An injection of succinylcholine IV to provide muscle relaxation during an operation patient


developed prolonged apnoea.
a) What is the probable cause for this?
b) Can you treat this with edrophonium?
c) Mention the line of treatment with reasons.
d) What other drugs can be given instead of succinylcholine ?
e) Name 2 centrally acting muscle relaxants.

19) A man aged 40 years was on a maintenance dose of warfarin 5mg/day for his angina attacks.
Subsequently he developed rheumatic joint pain for which he was advised to take
109
GOVT MEDICAL COLLEGE, TRIVANDRUM
LUMINAIRE-2015
aspirin(300mg tablets) 4-4-4 after food. One week later, he was brought to casuality in a state of
shock due to severe hematemesis.
a) Explain the reason for this.
b) How it could have been prevented?
c) How will you treat this emergency?
d) Name 2 oral anti-coagulants

20) A patient who was on large doses of Haloperidol for two weeks , has developed rigidity, tremor
and mask like facies.
a) What can be the clinical condition for which the patient was taking
haloperidol?
b) Name 2 other drugs useful in the management of same conditions?
c) What is the probable cause of side effects?
d) How will you treat the adverse effects?
e) Name another group of drug with same side effect?

21) Dr. Mohan a lecturer in anaesthesia injected 2mg of 2.5% solution of thiopentone sodium iv
for open reduction for fracture humerus. 15minutes after the injection, patient recovered from
anaesthesia and hence thiopentone sodium was repeated many times making a total of 5
injections.
a) What is the reason for quick recovery from anaesthesia and does the repeated
injections modify recovery?
b) What are the drawbacks of thiopentone sodium?
c) Name other iv administered general anaesthetics.
d) Write 2 other uses of thiopentone sodium.
e) What are the components of neuroleptic anaesthesia and write its advantages.
22. Ramesh, 28 yrs was administered an injection of adrenaline with lignocaine at base of his
thumb for nail extraction. Later he developed severe gangrene of thumb:
a) Why has gangrene developed and how could this have been avoided.
b) Mention the mechanism & pharmacological actions of adrenaline and lignocaine
c) List 2 other surface anaesthetics and mention their a/e.
d) Mention the cardiac uses of lignocaine with dose and rationale.
23. A 24 yr old man is bought to the hospital with complaints of unconsciousness, sustained
contractions (tonic) of all body muscles followed by periods of muscle contraction alternating with
periods of relaxation (clonic) lasting 1-2 min.
a)What are the drugs that can be prescribed for this patient?
b)Describe the pharmacokinetics and adverse effects of any one such drug
c)What is the treatment of status epilepticus                         (2+3+1=6)
24. A 50 year old man develops pedal edema and also has difficulty in
110
GOVT MEDICAL COLLEGE, TRIVANDRUM
LUMINAIRE-2015
breathing.Echocardiography confirms mild congestive cardiac failure. Answer the following:
a)What are the first line drugs in congestive cardiac failure.
b)What is the rationale for use of each of them in congestive cardiac failure
c)Mention the life style modification that you would advise to this patient
d)Add a note on treatment of digoxin over dosage     (1+2+1+2=6)

25. Dhruva, 49 yrs, was admitted to the hosp with complaints of palpitation, tachycardia, chest
pain. The diagnosis was PSVT.

a)Name the drug used for this patient.

b)Mention its MOA

c)Name 1 drug used in ventricular tachycardia

d)Mention its MOA

e)Name 2 drugs used in Heart block.

PAPER 2

1) Ravi 38 yr old recently diagnosed as pulmonary TB


a) What are the 1st line drugs given in TB
b) Give MOA of each
c) Name drug effective against persisters
d) Mention its 2 other uses and also 2 drug interactions
e) His 4 yr old daughter was also diagnosed as pulmonary TB but was given
pyrazinamide instead of ethambutol. Why?
f) List 2 important adverse effects of pyrazinamide
g) Mention 2 situations where prednisolone is used with anti TB drugs
h) Write a short note on DOTS

2) 25 yr old business man from Bangalore admitted at hospital with high grade fever,
malaise and chills in 3 day cycle, his peripherl smear was positive for P. vivax.
a) Prescribe suitably for him
b) With the help of schematic diagram of life cycle of parasite, Classify anti
malarial agents
c) What are the important side effects of the prescribed drugs
d) Outline management of cerebral malaria
e) Mention 2 drugs for chemoprophylaxis of malaria with dosage schedule

3) 30 yr old lady was admitted with tremor, loose stools, sweating, loss of weight inspite of
increased appetite. O/E HR was 120/min, she had a uniform thyroid swelling.
111
GOVT MEDICAL COLLEGE, TRIVANDRUM
LUMINAIRE-2015
a) Provisional diagnosis
b) Drug for immediate control of symptoms
c) Drug used if she was pregnant
d) What is the MOA of this drug
e) Mention pre-operative drug therapy in this condition

4) Male truck driver detected to be HIV positive, develops cough, fever, loss of weight. He
was admitted for investigation
a) Most likely cause of his present complaints
b) Name four drugs to be prescribed to him for treatment of HIV
c) If there is associated infection with pneumocystis carni, what is the drug of
choice
d) What drugs will you use if his sputum shows growth of TB bacilli
e) Discuss limitations of anti HIV therapy

5) Ravi 48, habitual smoker and alcoholic complaints of recurrent paint in epigastrium,
nausea and belching. Complaints started 2 yrs back and he used to take antacids for
pain relief. Endoscopy confirmed gastric ulcer. On serological examination, he had H. pylori
infection.
a) What drugs will you prescribe. Justify and write dosage schedule
b) MOA of omeprazole, pirenzepine, ranitidine
c) Mention 4 drugs C/I in this patient
d) List advantages and disadvantages of 1) aluminium hydroxide gel
2) Mgtrisilicate 3)NaHCO3
e) Classify drugs used in peptic ulcer
6) 20 yr old male hospitalized with dehydration, kussmauls breathing and
unconsciousness, urine shows severe glycosuria and ketonuria
a) Diagnosis
b) Management
c) Rationale of all the drugs used

7) 75 yr old woman, cholesterol level of 325 mg/dl and is put on anti cholesterol therapy
a) Mention drugs used as hypolipidemics
b) Mention MOA
c) What are the different statins and their dose level
d) What are the common adverse effects of statins

8) 30 yr old female patient suffering from chronic asthma well controlled with
aminophylline developed respiratory infection was brought to the medical op for treatment, she
112
GOVT MEDICAL COLLEGE, TRIVANDRUM
LUMINAIRE-2015
was prescribed erythromycin 500mg QID
a) Is erythromycin appropriate for the patient
b) Mention 2 antibiotics suitable and 2 antibiotics that should not be used in this
patient
c) Write two clinical conditions where erythromycin is used as DOC
d) Write down MOA of erythromycin

9) The leprologist Dr. Madhavan, on clinical examination found a patient suffering from
pauci bacillary leprose
a) Classify anti-leprosy drugs
b) Write the MOA and important S/E of drugs used
c) Mention 2 reaction states that follow the chemotherapy. Explain
d) Write two drugs effective in this reaction
e) Write the dosage schedule in pauci bacillary leprosy

10) A 40 yr old man attended the op with a h/o dysentery. The attending doctor p
metronidazole 400mg for 5 days. The dysentery subsided temporarily and recurred after 2
weeks
a) Was it appropriate to start metronidazole initially
b) How will you confirm your diagnosis
c) What is the reason for relapse

11) A 45 yr old male with type II diabetes mellitus had been on treatment with a
combination of two oral hypoglycaemic drugs. As his blood sugar was not lowered with the
above drugs, 40 units of insulin was added to the therapy. Later the dose of insulin had to be
increased to control hyperglycaemia.
a) What is insulin resistance and how will you manage it?
b) Name the different insulin preparations and mention the methods of its
delivery.
c) Give two important adverse effects of insulin and how will you manage it.

12) A middle aged man was admitted to the hospital with severe epigastric pain w
aggrevated with food intake. Diagnosis is peptic ulcer.
a) Enumerate the different groups of drugs that can be used in such patients. Name one
drug from each group.
b) Name 2 drugs which can be used in NSAID induced peptic ulcer.
c) What is the cause for recurrence of peptic ulcer ?outline the drug treatment for the
same.
d) What advice on lifestyle modification should be given to him?
113
GOVT MEDICAL COLLEGE, TRIVANDRUM
LUMINAIRE-2015
e) Name two drugs from different groups that can cause peptic ulcer.

13) A 50 year old male patient was admitted to emergency ward with the following
: unconscious , rapid shallow breathing with smell of acetone, tachycardia, thready pulse , dry
tongue, wasting of muscles. HIS TEMPERATURE WAS 100 F .skin dry , non elastic, blood sugar
– 600 mg%, urine sugar +++. BP 80/60 mmHg .
a) What is your diagnosis? Which drug would you administer initially ? Name the
preparation , dose, route.
b) What are the follow up measures in treatment you would undertake for this
patient?
c) What is insulin resistance? How will you manage?
d) Mention different groups of oral antidiabetic grugs with examples.
e) Enumerate the various preparations of human insulins available with a note
on their advantages.

14) Ragini, 26years complains of a thyroid swelling , sweating , shivering of hands


loss of weight . heart rate 126/min. she has exophthalmos.
a) Name the drugs that you will give to get rid of the symptoms at the earliest.
Mention the dose , route and rationale for use.
b) Which are the specific drugs you can give for long term management for this
condition? What is the mechanism of action and ADR of one of this?
c) Which drug is contraindicated in this age group? Why ?
d) Will you give cyanates? Justify

15) A diabetic patient receiving tolbutamide is also suffering from right knee joint
for which he administered salicylates. He suddenly developed flushing, headache and
sweating.
a) Explain the reason for this .
b) Mention 2 other drugs with similar action.
c) Alternatively , how will you treat the patient for joint pain?
d) Name 2 selective cox-2 inhibitors.
e) What are the adverse effects of tolbutamide?

16) A female patient receives rifampicin therapy for tuberculosis and is also admi
oral contraceptive pills to prevent pregnancy. However she conceives.
a) Explain the reason for contraceptive failure.
b) Name 2 other drugs with similar effects.
c) How will you prevent this?
d) Mention 2 adverse effects of oral contraceptives.
114
GOVT MEDICAL COLLEGE, TRIVANDRUM
LUMINAIRE-2015
e) Mention 2 other indications of rifampicin.

17) Raju 35 years , presents with fever for more than a week duration and his WBC
was 4000/mm3 . widal test was found to be positive.
a) What is the probable diagnosis?
b) How will you treat?
c) Name 4 other drugs effective in managing this condition and mention the side
effect of any of two?
d) What is the DOC for the same condition in a child of 6 years? Give reason.
e) What is the drug treatment for carrier state?

18) A 30 year old lady came to gynaec OP with complaints of itching vulva and pro
discharge from vagina. After proper investigations she was diagnosed to have
trichomonal vaginitis.
a) How do you treat this patient?
b) What advice would you give to this patient?
c) Mention two drugs not advisable in this patient during treatment and why.
d) Write 4 other indications of the drug to be used in this patient.
e) Write 2 differences between tinidazole and metronidazole.
19)A young businessman with amoebic dysentery was given Metronidazole and Diloxanide
furoate. While on Rx he went for a party and consumed liquor. Immediately he developed
flushing of the face, throbbing headache and tightness in the chest.
a) Why did he develop these symptoms while on Rx?
b) Why is Metronidazole combined with diloxanide furoate?
c) Write the MOA and two other uses of Metronidazole
d) List 4 groups of drugs used in amoebiasis
20)A 20 years old pregnant woman developed a mild upper respiratory tract infection .She was given
cap.   Tetracycline 250mg thrice daily for three days. 
a)Comment on the treatment for a mild bacterial infection.
b)Comment on the use of tetracycline in pregnancy.
c)If it was a severe infection, what will be your choice of antibiotic in a pregnant woman.
(2+2+2=6)

SPOTTERS
DRUGS
1. Streptokinase
Use – dissolve clots; used in MI
Loading dose is required since antibodies from past streptococcal infections can
neutralize the drug

115
GOVT MEDICAL COLLEGE, TRIVANDRUM
LUMINAIRE-2015
Longest acting fibrinolytic – Tenectiplase
Drug with highest fibrin selectivity – Tenectiplase
Drug used in Streptokinase overload – Epsilon amino caproic acid
2. Enalapril
Active metabolite – Enalaprilat (administered IV in hypertensive emergency)
Not used along with spironolactone – the combination causes fatal hyperkalemia
3. Frusemide
Causes hypocalcemia
Frusemide = Lasix (last six hours)
Longest acting loop diuretic – Torsemide
Most potent loop diuretic – Bumetanide
4. Dorzolamide
Carbonic anhydrase inhibitor used topically in glaucoma (8th hourly)
Other topical drugs in glaucoma – Latanoprost (Prostaglandin), Betaxolol (beta blocker)
5. Propranolol
Causes bronchoconstriction; hence not used in asthmatics
Sudden withdrawl causes rebound hypertension d/t receptor upregulation and
hypersensitivity
Used in pheochromocytoma with phentolamine (propranolol prevents arrhythmia,
phentolamine controls hypertension)
6. Low dose Aspirin
Dose – 75 mg OD orally
Monoclonal antibody used in unstable angina – Abciximab
Other antiplatelet drug – Clopidogrel
7. Amlodipine
Higher vascular selectivity – Felodipine, Lacidipine
Congener which prevents atherosclerosis – Nitrendipine
Cerebro selective CCB - Nimodipine
8. Indapamide
Thiazide diuretic
Endocrine side effects – Diabetes mellitus, Impotence
9. Propofol
IV induction agent
Used for day care surgery (short duration of anesthesia, rapid induction & recovery, lacks
airway resistance, residual impairment less)
Disadv: Pain during induction, No analgesia & skeletal muscle relaxation
10. Lignocaine
Dosage forms – Topical solution, gel, ointment, spray
ADR: arrhythmia, hypotension, respiratory depression
116
GOVT MEDICAL COLLEGE, TRIVANDRUM
LUMINAIRE-2015
Not used in liver disease since it is metabolised in liver
11. Olanzapine
Atypical antipsychotic
Used with caution in Diabetes, Obesity, Hypertension
Adv. Over Clozapine – no agranulocytosis, no hypersalivation
12. Fluoxetine
Longest acting SSRI
Dose; 20-40 mg OD daily
Other uses: Social phobias, Obsessive Compulsive Disorder
Not used in acute depression d/t slower onset of action
13. Zolpidem
Used in short term insomnia (5-10mg orally at bedtime)
Adv over BZD: minimal residual daytime sedation, no rebound insomnia, less dependence &
tolerance
Antidote: Flumazenil
14. Tizanidine
Centrally acting alpha 2 agonist with muscle relaxant property
Uses: Spastic neurologic diseases, muscle spasms, spinal diseases
Other centrally acting SMR: Baclofen (GABAB agonist), Diazepam
15. Paracetamol
Preferred over aspirin in 1) children 2) patients with peptic ulcer
Toxicity affects liver and kiney (analgesic nephropathy)
Antidote: N-acetyl cysteine
16. Diclofenac
Route: IM, topical
Uses: Toothache, Rheumatoid arthritis (50 mg TID, orally)
Aceclofenac is its congener with chondroproctive action and selective COX-2 inhibition
17. Cefixime
3rd gen cephalosporin given orally
Dose in severe UTI-
Not sensitive against Staphylococcus, Pseudomonas
ADR- Diarrhea, Stool changes
18. Levocetrizine
Uses: Allergic rhinitis, Urticaria
Adv: No sedation, no anticholinergic side effects
Dose: 10 mg OD orally
19. Atorvastatin
Inhibits HMG CoA reductase; reduces LDL fraction
Has antioxidant property & long t1/2
117
GOVT MEDICAL COLLEGE, TRIVANDRUM
LUMINAIRE-2015
Most potent statin: Rosuvastatin
20. Metoclopramide
Antiemetic with prokinetic action (10mg TID orally)
Can be given orally, IM, IV
21. Ciprofloxacin
MOA- inhibits DNA gyrase
Dose in Enteric fever: 500mg BD X 7 days
Not sensitive against Bacteroids, Clostridia
Pefloxacin is safe in renal insufficiency
Sparfloxacin has highest incidence of photophobia
QT interval prolongation: Sparfloxacin, Gatifloxacin
22. Anti TB Combination
First line anti TB drug given parenterally – Streptomycin
4 drugs / pellet - thrice weekly X 2 months; 2 drugs / pellet – thrice weekly X 4 months
Rifampicin – acts on persistor bacilli preventing relapse
23. Acyclovir
Dose in Chicken pox: 400 mg QID X 5 days
Other uses: Herpes simplex encephalitis, Mucocutaneous herpes
Causes Nephrotoxicity
Congener active against CMV- Ganciclovir
24. Fluconazole
Adv. over ketoconazole: less potent enzyme inhibitor, less anti androgenic side effects
Preferred in fungal meningitis (good CNS penetration)
Voriconazole – used in aspergillosis
25. Efavirenz
Used in HIV infection
Other anti HIV drugs – Zidovudine, lamivudine
Nevirapine: prevents vertical transmission of HIV from mothers to kids
26. Esomeprazole
Longer half life
Lansoprazole: partly reversible blocker H+ K+ ATPase
Fastest acting PPI- Rabeprazole
PPI given IV- Pantoprazole, Lansoprazole
27. Albendazole
Uses: Ascariais, Giardiasis
Preferred in neurocysticercosis (short course, cheaper)
28. Tinidazole
Uses: Amebiasis, Trichomoniasis
Dosage: 2gm orally X 3 days
118
GOVT MEDICAL COLLEGE, TRIVANDRUM
LUMINAIRE-2015
29. Dicyclomine
Uses: Abdominal colic, Morning sickness, Motion sickness
Dose for abd. Colic: 20 mg SOS
Not used in children below 6 months (exhibit fever symptoms)
Other drugs for morning sickness: Doxylamine, Cyclizine
30. Bisacodyl
Route: Oral, suppository
Contraindications: pregnancy, Intestinal obstruction
Drug with similar action: Sodium picosulphate
31. Azhithromycin
Adv. Over erythromycin: longer duration, less enzyme inhibition, intracranial penetration
Not active against MRSA
DOC: Mycoplasma pneumonia, Chlamydiasis
Dose in Respiratory infection: 500 mg OD X 3 days; 1 hour before or after food
32. Artemesia annua
Part used-Alkaloids from leaves and stem
Active principle - Artemesine
MOA
Derevatives- Artether, Artemether, Artesunate
Uses- Multi drug resistant malaria
ADR- Nausea, Vomiting, abdominal pain, QT prolongation
33. Adathoda vasica
Parts used-Alkaloids from leaves, roots , flowers,fruits, bark
Active principle-Vasicine, Vasicinone
MOA
Semisynthetic derivatives- Bromhexine, Ambroxol
Uses- Bronchitis, Asthma
34. Atropa belladona
Parts used - Alkaloids from roots and berries
Active principles- Atropine , Hyoscinamine, Belladona
Semisynthetic derivative - Tropicamide
35. Digitalis - Digitalis purpura, Digitalis lanta
Parts used - alkaloids from leaves
Active principles - Digoxin, Digitoxin
MOA,Uses, Toxicity, Treatment
36. Papaver somniferum
Part used - exudate from unripe capsule of poppy seed
Active principle - Opium
MOA, Uses ,Poisoning, Treatment
119
GOVT MEDICAL COLLEGE, TRIVANDRUM
LUMINAIRE-2015
37. Cinchona pubescens
Part used - Alkaloids from bark
Active principle - Cinconin, Cinconidine, Quinine, Quinidine
MOA,Uses,Poisoning
38. Chondrodendron tomentosum
Parts used-Alkaloids from leaves and roots
Active principle - d-tubocuramine
MOA - Competitive blocker of Nm receptors
Uses, Poisoning, Treatment
39. Physostigma venenosum
Parts used - Alkaloids from bean
Active principle - Physostigmine
MOA-reversible anticholinesterase
Uses, Poisoning, Management
40. Vinca rosea
Alkaloids from dried leaves
Active principle- Vincristine, Vinblastine, Vindesine
MOA,Uses,ADR

INSTRUMENTS
1. Tuberculin syringe
2. Frontal writing lever
3. Metered dose inhaler
4. Spacer
5. Insulin syringe
6. Nebuliser
7. Rotahaler
8. Needles

120
GOVT MEDICAL COLLEGE, TRIVANDRUM
LUMINAIRE-2015
ADVERSE EFFECT DRUG CAUSING
Teeth discoloration Tetracycline
Ciprofloxacin
Fe preparations
Alopecia Anticancer drugs (Cisplatin, Paclitaxel)
Anticoagulants (Heparin, Warfarin)
Gentamicin, Valproate
Cushing syndrome Corticosteroids
OCPs
Steven –Johnson syndrome Sulfonamides, Penicillin
Phenytoin
Allopurinol
Hirsuitism Phenytoin
Cyclosporine
Metoclopramide
Spina bifida Sodium valproate
Carbamazepine
Gingival hypertrophy Phenytoin
Amlodipine
Cyclosporine
Oral thrush Inhalational corticosteroids, Antibiotics
Cleft palate Phenytoin, Valproate, Methotrexate,
Aminopterin
Gynecomastia Digoxin
Spirinolactone
Ketoconazole
Risperidone

Drugs Type of Clinical significance


interaction

Captopril + SpironolactonePharmacodyn Dangerous hyperkalemia


amic;
Synergism

Aspirin + thiazide Pharmacodyn Diuretic action of thiazide is PG mediated; Aspirin inhibit PG synt
amic; Failure of diuretic therapy
Antagonism

121
GOVT MEDICAL COLLEGE, TRIVANDRUM
LUMINAIRE-2015
OCP + Rifampicin Pharmacokine Rifampin is a microsomal enzyme inducer à increase OCP metab
tic; contraceptive failure
Metabolism
Sucralfate + Anatacids Pharmacodyn Sucralfate polymerizes at pH <4; Antacids increase pH à prevents
amic; of sucralfate
Antagonism

Sucralfate + Pharmacokine Sucralfate prevents absorption of tetracycline / phenytoin à thera


tetracycline/phenytoin tic; Absorption failure

Insulin + Propranolol Pharmacodyn


amic;
Antagonism
Diclofenac + Salbutamol Pharmacodyn NSAID inhibits PG synthesis à Arachdonic acid channelled to
(Asthma) amic; Leukotriene synthesis à bronchoconstrictionà worsens asthma
Antagonism

Astemizole + Pharmacokine Erythromycin is a enzyme inhibitor à decreases Astemizole meta


Erythromycin tic; à arrhythmia
Metabolism

Diuretic + Lithium Pharmacokine Diuretic cause Na+ loss à Body tries to increase Na conc. à Li get
tic; Excretion reabsorbed along with Naà Lithium toxicity

Neostigmine + Atropine Pharmacodyn Neostigmine given to reverse muscle paralysis after ansesthesia
amic; causes muscarinic side effectsà reversed by atropine (anti musc
Antagonism

Ciprofloxacin + Pharmacokine Ciprofloxacin is a enzyme inhibitor à decrease theophylline metab


Theophylline tic; à CVS & CNS toxicity
Metabolism

Tetracycline + Pharmacokine Tetracycline absorption decreased à therapeutic failure


Iron/Antacid tic; Absorption

Digoxin + Quinidine Pharmacokine Quinidine displaces Digoxin from PPB sites à Digoxin levels incre
tic; (Quinidine also decreases the renal clearance of digoxin)
Distribution

Digoxin + propranolol Pharmacodyn Both decreases AV conduction à precipitates AV block


amic;
Synergism

122
GOVT MEDICAL COLLEGE, TRIVANDRUM
LUMINAIRE-2015
Aspirin + Methotrexate Pharmacokine Aspirin displaces MTx from PPB site à precipitates MTx toxicity
tic;
Distribution
Penicillin + Probenecid Pharmacokine Blocks tubular secretion of penicillin à decrease penicillin excreti
tic; Excretion prolong its duration of action à therapeutic efficacy increased

Warfarin + Ceftriaxone Pharmacodyn Hypoprothrombinemia- ADR of Ceftriaxone à potentiates the


amic; anticoagulant effect of warfarin à increased risk of bleeding
Synergism

Clopidogrel + Warfarin Pharmacodyn Clopidogrel- antiplatelet action; Warfarin-anti coagulant à increas


amic; of bleeding
Synergism
Frusemide + Pharmacodyn Both are ototoxic drugs à additive toxicity
Aminoglycoside amic;
Synergism

Penicillin + tetracycline Pharmacodyn Penicillin- bactericidal; Tetracycline – baceriostatic à efficacy of


amic; combination reduced
Antagonsim

Levodopa + Pharmacodyn L-dopa relieves parkinsonian symptoms by D2 agonism;


Metoclopramide amic; Metoclopramide blocks central D2 receptors reduces efficacy of
Antagonism

Statins + Gemfibrozil Pharmacodyn Both causes myopathy à additive toxicity


amic;
Synergism

Amitryptiline + Atropine Pharmacodyn Both have anticholinergic action


amic;
Synergism

Sulfamethoxazole + Pharmacodyn Sequential blockade of folate metabolism pathway in microbes


Trimethoprim amic;
Synergism

Amoxicillin + Clavulanic Pharmacodyn Amoxicillin is destroyed by beta lactamase ; Clavulanic acid is a


acid amic; beta-lactamase inhibitor increases amoxicillin concentration sam
Synergism = 10 hours

Nitrates + beta blockers Pharmacodyn Tachycardia caused by nitrates is nullified by beta blocker action
(angina) amic;

123
GOVT MEDICAL COLLEGE, TRIVANDRUM
Tachycardia caused by nitrates is nullified by beta blocker action
(angina)
LUMINAIRE-2015
Antagonism

Nitates + CCB (angina) Pharmacodyn Nitrates reduces preload(venodilators)


amic; CCB reduces afterload (arteriodilators) à reduces the workload of
Synergism ischemic heart

TCA + Clonidine Pharmacodyn TCA abolishes antihypertensive action of clonidine by inhibiting i


amic; transport into adrenergic neurons.
antagonism

Cholestyramine + Pharmacokine Effectve blood concentration of digoxin is decreased


Digoxin tic; absorption
Tolbutamide + Salicylate Pharmacokine Salicylates displaces sulfonylureas from PPB sites à cause dang
tic; hypoglycemia
Distribution

Lignocaine + Adrenaline Pharmacokine Adrenaline causes vasoconstriction à decreases absorption of


tic-Absorption lignocaine decreasing the systemic toxicity and increasing the du
of action

L-Dopa + Pyridoxine Pharmacokine Pyridoxine as co-factor for carboxylase enzyme à increased perip
tic decarboxylation of L-Dopa à decreased efficacy
Metabolism

CHARTS
1. Effects on the cat’s tibialis anterior muscle nerve preparation
a. Explain the graph
b. Classify neuromuscular blockers.
c. Write the uses of neostigmine
ans: 1. post operative paralytic ileus
2. post operative urinary retention
3. myasthenia gravis
4. cobra bite
d. Shortest acting Nm blocker: succinyl choline
e. Longest acing Nm blocker: doxacurium
2. Experiment on frog rectus abdominis muscle
a. Name the different drugs which blocks the action of ACh at diff. sites.
b. What is the nature of Drug D? Ans: sk.muscle relaxant
c. Mention 2 drugs with similar MOA
d. How can you reverse the action of drug D. Ans: increase the dose of ACh
3. Experiment on rat phrenic nerve
124
GOVT MEDICAL COLLEGE, TRIVANDRUM
LUMINAIRE-2015
a. What type of Nm blocker is drug A? give reason
Hint: depolarising blocker.
b. Name 2 other drugs with MOA similar to drug A.
Ans: dexamethonium, succinyl choline.
c. Write 3 uses of Nm blocker.
Ans: 1. adjuvant to general anaesthesia
2. endotracheal intubation
3. electroconvulsive therapy
4. Effect of drugs on anaesthetised dogs BP
a. Explain the graph .
b. What is drug A
Ans: alpha 1 blocker
c. Mention the alpha adrenergic receptors with its distribution and action.
5. Effect of drugs on anaesthetised dogs BP
a. Explain the graph. Hint: Tachyphylaxis
b. Mention 2 other drugs producing the above effects
Ans: ephedrine, nicotine
c. Write their MOA Hint: NA release
6. Graph of failing heart
a. Explain the graph
b. What are the effects of digoxin on a failing heart
Hint: positive inotropic effect
c. Write its cardiac adverse effects
Ans: arrythmia, AV block, bradycardia
7. Nicotinic action of ACh on dogs BP
a. Explain the graph
b. Explain mechanism behind nictinic action of Ach
c. Write the distribution of nicotinic receptors.
8. Effect of drugs on anaesthetised dogs BP
a. Explain the graph
b. Identify drug A and write its 2 non cardiac uses
Ans: non selective β blocker [propranolol]
Uses: prophylaxis of migraine, thyrotoxicosis
c. Write 3 most important clinical uses of adrenaline
Ans: anaphylaxis, to control bleeding from skin and mucous membrane, given
along with local anaestheic.
9. Effect of drug on respiratory smooth muscle
a. What is drug D?
Ans: Bronchodilator-it relaxes respiratory smooth muscle.
125
GOVT MEDICAL COLLEGE, TRIVANDRUM
LUMINAIRE-2015
b. Classify drugs with actions similar to drug D
c. Mention their routes of administration
Hint: 1. Injection: salbutamol
2. Inhalation: salbutamol, ipratropium bromide
3. Subcutaneous: adrenaline
4. Oral: salbutamol, theophylline
10. Effects of drugs on respiration on anaesthetised cat
a. Expain the graph
b. Why is morphine used cautiously in asthmatic patients
1. Histamine release
2. Respiratory depressant
3. Cough depressant
4. Smooth muscle contraction.
c. Which effect of morphine is not antagonosed by nalorphine
Ans: sedation
11. Effects of drugs on intestinal smooth muscle
a. Explain the graph
b. What type of drug is bethanechol, mention its clinical use
Ans:- cholinergic drug
Uses :- postoperative urinary retention, congenital megacolon, GERD
c. Which is the unknown drug ? What are its clinical use?
Ans:- Antimuscarinic agent
Use:- AV block, OP poisonong, refraction testing in children, abdominal colic
12. Effects of drugs on intestinal smooth muscle
a. Explain the graph
b. What is drug A? What is its MOA?
Hint: Smooth muscle relaxant- directly acting.
c. Mention 2 other drugs with similar MOA.
Hint: Directly acting: KCl and nitrates
d. Why barium sulphate is used instead of barium chloride in barium enema?
Hint: Barium enema material should be radioopaque

BaCl2 BaSO4

Slimulate smooth muscle radio opaque

Not radio opaque does not produce systemic effects

No smooth muscle contraction

126
GOVT MEDICAL COLLEGE, TRIVANDRUM
LUMINAIRE-2015
13 Graph on human uterus
a. What is the action of oxytocin and isoxsuprine on uterus?
b. Write 2 clinical uses of oxytocin
Ans: milk ejection and induction of labour
c. What is a tocolytic agent? Give 2 reasons

14. Action of mepyramine maleate on histamine receptor of Guin


a. What is the effect of histamine on intestine.
b. What is pretreatment with antihistamine.
c. Name 2 second generation antihistamines with dose.
15.. Dose Response Relationship
a.Define Bioassay
b. Mention 2 types of Bioassay where rectus muscle is used
c.Define supramaximal dose
d.What is dose-response relation
e.Advantages of frog rectus muscle
f.Receptor responsible for above action
16.Effect of drugs on rabbits eye
DrugA Nl 2 mts 4mts 6mts 8mts 10mts

Pupil size 6 6 6 7 8 8

Corneal reflex + + + + + +

Light reflex + + + + + +

conjunctiva Pink Pink Pink Pale Pale Pale

DrugB Nl 2 mts 4mts 6mts 8mts 10mts

Pupil size 6 6 6 7 8 8

Corneal reflex + + + + + +

Light reflex + + + + - -

conjunctiva Pink Pink Pink Pink Pink Pink

DrugC Nl 2 mts 4mts 6mts 8mts 10mts

Pupil size 6 6 6 5 4 3

Corneal reflex + + + + + +

127
GOVT MEDICAL COLLEGE, TRIVANDRUM
LUMINAIRE-2015
Light reflex + + + + + +

conjunctiva Pink Pink Pink Pink Pink Pink

DrugD Nl 2 mts 4mts 6mts 8mts 10mts

Pupil size 6 6 6 7 8 8

Corneal reflex + + + - - -

Light reflex + + + + + +

conjunctiva Pink Pink Pink Pink Pink Pink

a)Interpret the findings

b)Identify the drugs (Hint:- A –Phenyl ephrine B- Atropine C-Miotic D-Local Anaesthetic )

c)Other drugs with same effect

d)Opthalmologic Uses

128
GOVT MEDICAL COLLEGE, TRIVANDRUM
LUMINAIRE-2015

MICROBIOLOGY

QUESTION PAPER PATTERN - THEORY

• Clinical essay (structured)- 10 marks


• Short essays – 2 X 5 = 10 marks
• Short notes- 10 X 2 = 20 marks

QUESTION PAPER PATTERN - PRACTICAL

• Gram staining- 5 marks


• AFB staining- 5 marks
• Clinical question- 5 marks
• Spotters – 10 mark
GENERAL MICROBIOLOGY
129
GOVT MEDICAL COLLEGE, TRIVANDRUM
LUMINAIRE-2015

Historical introduction

1. Louis Pastuer
2. Contributions of Robert Koch
3. Koch’s postulates

Morphology & Physiology of Bacteria


1. Dark ground microscopy
2. Negative staining
3. Cell wall
4. L- form
5. Mesosomes
6. Intracytoplasmic inclusions
7. Capsule
8. Flagella
9. Fimbriae
10.Spore
11.Bacterial growth curve

Sterilisation and Disinfection


1. Sterilization- definition, agents, methods (short essay)
2. Disinfectants & antiseptics
3. Incineration
4. Hot air oven
5. Pasteurization
6. Inspissation
7. Tyndallisation
8. Autoclave (short essay)
9. Filters with examples
10.Cold sterilization
11.Aldehydes
12.Gaseous disinfectants (formaldehyde, ethylene oxide, BPL)

Culture media
1. Culture media – definition and types (short essay) 130
GOVT MEDICAL COLLEGE, TRIVANDRUM
LUMINAIRE-2015

2. Agar
3. Enriched media with examples
4. Enrichment media with examples
5. Differential media with examples
6. Indicator media with examples
7. Transport media with examples
8. MacConkey agar and its constituents

Culture methods
1. Anaerobic cultural methods (short essay)
2. McIntosh - Fildes jar
3. Robertson’s cooked meat medium

Bacterial genetics
1. Plasmids
2. Episomes
3. Mutation- types (short essay)
4. Modes of transmission of genetic material in bacteria
5. Lysogenic conversion
6. Prophage
7. R factor
8. Genetic mechanisms of drug resistance
9. Diff. between mutational and transferable drug resistance
10.Transposons
11.Genetic engineering
12.Southern blotting
13.Polymerase chain reaction
14.Phage typing

Microbial pathogenicity
1. Nosocomial infection
2. Iatrogenic infection
3. Latent infection
4. Pathogenicity & virulence
5. Exaltation & attenuation
6. Differences between exotoxins & endotoxins 131
GOVT MEDICAL COLLEGE, TRIVANDRUM
LUMINAIRE-2015

7. Carriers- types ; Paradoxical carrier


8. Septicemia
9. Endemic, epidemic and pandemic disesases.

IMMUNOLOGY
Immunity
1. Types of immunity, examples
2. Acute phase proteins
3. Differences b/w active & passive immunity
4. Artificial immunity- Types- Active and passive
5. Local immunity
6. Adoptive immunity, Transfer factor
7. Herd immunity
8. Immune elimination
9. Premunition

Antigen
1. Antigen
2. Hapten
3. Paratope and Epitope
4. Heterophile antigen- egs
5. Super antigen

Antibodies
1. Immunoglobulins- types , structure and functions(short essay)
2. Bence Jones Protein
3. Reagin antibody

Antigen-Antibody reactions
1. Precipitation- mechanism, applications (short essay)
2. Flocculation- types , mechanisms, applications(short essay)
3. Prozone phenomenon
4. Immunoelectrophoresis
5. Immunodiffusion
6. Agglutination reactions- types, mechanisms, applications (short essay)
132
GOVT MEDICAL COLLEGE, TRIVANDRUM
LUMINAIRE-2015

7. Coomb’s test- types, mechanisms (short essay)


8. Co-agglutination reactions, co-agglutinin
9. Complement fixation test
10.Opsonisation
11.Immunofluorescence
12.Enzyme immunoassays

Complement system
1. Complement - definition, classical & alternative pathways (short essay)
2. Biological effects of complement
3. Hereditary angioneurotic edema

Immune system & Immune response


1. Helper T cells, Cytotoxic T cells
2. Null cells, Naive cell
3. MHC (HLA antigen)- Application
4. MHC restriction
5. Antigen presenting cells
6. Clonal selection theory
7. Monoclonal antibodies
8. Hybridoma (short essay)
9. Adjuvants
10.Cell mediated immunity
11.Cytokines
12.Transfer factor
13.Immunological tolerance

Immunodeficiency diseases
1. Give examples of humoral immunodeficiencies
2. Give examples of cellular immunodeficiencies

Hypersensitivity
1. Hypersensitivity- definition, types, mechanisms and examples in
detail(short essay)
2. Type I hypersensitivity reaction 133
GOVT MEDICAL COLLEGE, TRIVANDRUM
LUMINAIRE-2015

3. Type II hypersensitivity reaction


4. Type III hypersensitivity reaction
5. Type IV hypersensitivity reaction
6. Type V hypersensitivity reaction
7. Shwartzman reaction

Autoimmunity
1. Autoimmunity- definition, pathogenesis, mechanism, examples(short
essay)
2. Sequestered antigen

Transplantation and Tumour immunity


1. Autograft
2. Isograft
3. Allograft
4. Xenograft
5. Graft versus host reaction
6. Immunological surveillance

Immunohaematology
1. ABO blood group
2. Bombay group
3. Rh incompatibility, prevention, HDN
4. Diseases spread through blood transfusion

SYSTEMIC BACTERIOLOGY
Staphylococcus (essay)
1. Morphology, Characteristics of colonies
2. Coagglutination
3. Heat cold phenomenon
4. Toxins of Staphylococcus
5. TSS Toxin
6. Staphylococcal food poisoning
7. Coagulase test- slide and tube
134
GOVT MEDICAL COLLEGE, TRIVANDRUM
LUMINAIRE-2015

8. Staphylococcal Scalded Skin Syndrome (SSSS)


9. Penicillin resistance- MRSA
10.Lab diagnosis and treatment
11.Micrococci
12.Coagulase negative staphylococcus

Streptocoocus and Enterococcus (essay)


1. Streptokinase and Streptodornase
2. Group A Streptococci - Lab diagnosis, pathogenesis, treatment
3. Morphology and classification
4. Streptolysin
5. ASO titre- Importance
6. Dick test
7. Suppurative and non suppurative infections caused, puerperal sepsis)
8. Group B streptococci
9. CAMP reaction
10.Group D streptococci

Pneumococcus (essay)
1. Pneumococcal meningitis&Pneumococcal pneumonia- Pathogenesis,
lab diagnosis, treatment
2. Morphology and characteristics of colonies
3. Optochin sensitivity
4. Soluble specific substance
5. Quellung reaction
6. CRP- Importance
7. SR variation
8. Streptococcus viridans

Neisseria (essay)
1. Meningococcus- Pathogenesis, lab diagnosis, treatment
2. Gonococcus- Pathogenesis, lab diagnosis, treatment
3. Kovac’s method
4. Meningococcal septicemia
5. Waterhouse Friderichsen syndrome 135
GOVT MEDICAL COLLEGE, TRIVANDRUM
LUMINAIRE-2015

6. Non gonococcal urethritis


7. Fitz Hugh Curtis Syndrome
8. Watercan perineum
9. Reiter’s syndrome

Corynebacterium (essay)
1. Diphtheria- Pathogenesis, complications, lab diagnosis, treatment &
prophylaxis
2. Volutin granules
3. Special stains for Corynebacterium
4. Diphtheria toxin, MOA
5. Lysogenic conversion
6. Eleks test
7. Virulence tests
8. Diphtheroids
9. Danysz phenomenon
10.Ehrlich phenomenon

Bacillus
1. Cultural characteristics
2. String of pearl reaction
3. PLET medium
4. Human anthrax- Types, Pathogenesis, lab diagnosis, treatment,
prophylaxis
5. Malignant pustule
6. McFadyean’s reaction
7. Ascoli’s test
8. Bacillus cereus

Clostridium- (essay)
1. Gas gangrene- pathogenesis, lab diagnosis, prophylaxis & treatment.
2. Tetanus - Pathogenesis, lab diagnosis, prophylaxis & treatment
3. Clostridium spores
4. Stormy fermentation
5. Lecithinase and Lecithinase effect
6. Nagler reaction 136
GOVT MEDICAL COLLEGE, TRIVANDRUM
LUMINAIRE-2015

7. Tetanospasmin & tetanolysin


8. Botulism- types, lab diagnosis
9. Cl. difficile- Antibiotic associated colitis, pseudomembranous

Nonsporing anaerobes
1. Classification, examples
2. Peptostreptococcus
3. Doderlein’s bacillus/ Lactobacillus
4. Treatment of anaerobic infection and predisposing factors

Enterobacteriaceae
1. Classification of enterobacteriaceaea.
2. IMViC test
3. Shiga like toxin
4. E. coli- Pathogenesis, lab diagnosis, treatment
5. Traveler’s diarrhoea
6. EPEC, ETEC, EIEC, EAEC & EHEC [VTEC]
7. Bacteriological diagnosis of UTI (essay) ; Screening techniques
8. Significant bacteruria
9. Klebsiella pneumonia (Friedlanders bacillus)

Shigella (essay)
1. Shigellosis- pathogenesis, lab diagnosis & treatment

Salmonella (essay)
1. Enteric fever- Pathogenesis, lab diagnosis, Prophylaxis & treatment
2. Craigie’s tube
3. HO variation
4. V-W variation
5. Clot culture
6. Widal test
7. Coagglutination
8. Typhoid carriers
9. Typhoid vaccines
10.Salmonella gastroenteritis 137
GOVT MEDICAL COLLEGE, TRIVANDRUM
LUMINAIRE-2015

Vibrio (essay)
1. Cholera – Pathogenesis, lab diagnosis, complications, prophylaxis &
treatment
2. Special culture media
3. Cholera red reaction , String test
4. Classification, El tor strain, serotypes
5. Halophilic vibrios
6. Kanagawa phenomenon- Wagatsuma agar
7. Copro antibodies (local antibodies)

Campylobacter and Helicobacter


1. Campylobacter
2. H. pylori- Pathogenesis, lab diagnosis, treatment

Pseudomonas and Burkholderia


1. Pigment production
2. Pathogenesis
3. Nosocomial infections- Control & treatment
4. Melioidosis

Yersinia and Francisella


1. Stalactite growth
2. Plague- Pathogenesis, lab diagnosis, prophylaxis, treatment
3. Clinical features of plague
4. Tularemia (Francisella)

Legionella
1. Pontiac fever
2. Legionnaire’s disease

Haemophilus and Gardnerella


1. Satellitism
2. H. influenza- Pathogenesis, lab diagnosis, prophylaxis, treatment
138
GOVT MEDICAL COLLEGE, TRIVANDRUM
LUMINAIRE-2015

3. H. ducreyi
4. Chancroid
5. HACEK bacteria
6. Clue cells
7. Gardnerella vaginalis , bacterial vaginosis

Bordetella
1. Antigens & virulence factors
2. Pathogenesis, lab diagnosis, prophylaxis and treatment

Brucella
1. Casteneda’s method
2. Brucellosis- Lab diagnosis,Prophylaxis, Treatment
3. Milk ring test

Mycobacterium tuberculosis (essay)


1. Tuberculosis- Pathogenesis, lab diagnosis & treatment
2. BCG vaccine & complications
3. Koch’s phenomenon
4. Culture media – Lowenstein Jenson media
5. Petroff’s method
6. Mantoux test
7. Antibiotic sensitivity testing of Mycobacterium tuberculosis

Non-tuberculous mycobacteria
1. Classification with examples, lab diagnosis & treatment (short essay)
2. MAC (Battey bacillus)
3. Swimming pool granuloma

Mycobacterium leprae (essay)


1. Leprosy- pathogenesis, lab diagnosis & treatment
2. Lepra reactions
3. Lepromin test

Spirochaetes
139
GOVT MEDICAL COLLEGE, TRIVANDRUM
LUMINAIRE-2015

1. Syphilis- Stages, lab diagnosis, prophylaxis, treatment (essay)


2. Leptospirosis- Pathogenesis, lab diagnosis, prophylaxis, treatment
(essay)
3. Chancre & chancroid
4. Serological tests for syphilis
5. VDRL
6. Biological false positive reaction; examples (acute, chronic)
7. Relapsing fever
8. Vincent’s angina
9. Lyme disease

Mycoplasma
1. Atypical pneumonia
2. Cold agglutination test

Actinomycetes
1. Human actinomycosis- Lab diagnosis, treatment
2. Nocardiosis
3. Differentiate actinomycotic & fungal mycetoma

Miscellaneous bacteria
1. Listerial infections & their lab diagnosis
2. Rat bite fever (Streptobacillus moniliformis & Spirillum minus)
3. Donovaniosis (Klebsiella granulomatis)
4. Cold enrichment
5. Acinetobacter bowmani

Rickettsiaceae
1. Neil Mooser reaction
2. Epidemic typhus, endemic typhus, tick typhus- Agent & Vector
3. Weil Felix reaction
4. Scrub typhus
5. Q-fever
6. Cat scratch disease

Chlamydia 140
GOVT MEDICAL COLLEGE, TRIVANDRUM
LUMINAIRE-2015

1. Chlamydia life cycle


2. TRIC agents
3. Trachoma, HP body
4. Inclusion conjunctivitis (swimming pool)
5. TWAR agents- C. pneumoniae
6. Lab diagnosis of chlamydial infection
7. LGV , Freis test
8. Psittacosis

CLINICAL MICROBIOLOGY (paper I)


1. Name some organisms causing
a. Pneumonia
b. UTI
c. Diarrhoea
d. Dysentery
e. Food poisoning
f. Meningitis
g. Sexually transmitted diseases
2. Antimicrobial sensitivity testing
a. Diffusion tests (Stokes method, Kirby-Bauer method, E test)
b. Dilution tests
3. Bacteriological analysis of drinking water
4. Presumptive and differential coliform count
5. Water borne diseases
6. Milk borne diseases

VIROLOGY
General Properties of Viruses
1. Haemagglutination test
2. Steps in viral multiplication
3. Von magnus phenomenon, Abortive virus, Defective virus
4. Pseudovirion, Viroids
5. Tissue culture, Cell culture
6. Cytopathic effect
7. Prions
8. Examples of DNA & RNA virus 141
GOVT MEDICAL COLLEGE, TRIVANDRUM
LUMINAIRE-2015

Virus-Host Interactions
1. Inclusion bodies
2. Interferons, cytokines
3. Immunoprophylaxis- live & killed vaccines with examples

Bacteriophage (short essay)


1. Lysogenic and Lytic Cycle
2. Phage conversions, transduction
3. Prophage
4. Bacteriophage typing
5. Bacteriocin

Poxviruses
1. Small pox vaccination
2. Molluscum contagiosum

Herpesviruses
1. Herpes simplex- Pathogenesis, clinical features, lab diagnosis,
treatment (essay)
2. Tzanck smear
3. Varicella zoster, Fetal varicella syndrome
4. Herpes zoster
5. CMV
6. EBV
7. IMN- Paul-Bunnel test (short essay)

Adenoviruses
1. Adenovirus- Pathogenesis
2. Epidemic keratoconjunctivitis

Picornaviruses
1. Polio virus- Pathogenesis, clinical features, lab diagnosis(essay)
2. Polio prophylaxis, complications of polio vaccination
3. OPV v/s IPV
142
GOVT MEDICAL COLLEGE, TRIVANDRUM
LUMINAIRE-2015

4. Manifestations of Coxsackie virus infection


5. Echovirus, Rhinovirus

Orthomyxoviruses
1. Diagram of influenza virus
2. Haemagglutination & Elution
3. Antigenic drift and shift
4. Pathogenesis of influenza, lab diagnosis
5. Prophylaxis & treatment of influenza
6. Hemoagglutination inhibitors

Paramyxoviruses
1. Diagram- Rubeolavirus
2. Mumps- Clinical features, lab diagnosis, prophylaxis, complication
3. Measles- Clinical features, pathogenesis, lab diagnosis, complications
4. Prophylaxis of measles
5. RSV

Arboviruses
1. Mosquito borne group
2. Tick borne group
3. Chikungunya
4. Japanese encephalitis
5. Yellow fever
6. Dengue fever
7. KFD

Rhabdoviruses (essay)
1. Rabies- Pathogenesis, lab diagnosis and prophylaxis of rabies
2. Negri bodies
3. Vaccination

Hepatitis Viruses (essay)


1. Hepatitis A
143
GOVT MEDICAL COLLEGE, TRIVANDRUM
LUMINAIRE-2015

2. Parenterally transmitted hepatitis viruses


3. Hepatitis B- pathogenesis, lab diagnosis, treatment and prophylaxis of
hepatitis B
4. Serological markers of HBV
5. Methods of detection HbsAg & antiHbs
6. Hepatitis B vaccine
7. Hepatitis C
8. Hepatitis D
9. Hepatitis E

Retroviruses- HIV (essay)


1. Diagram
2. HIV structural antigens & their role in diagnosis of HIV infections
3. Antigenic variation of HIV
4. Pathogenesis, clinical features, lab diagnosis
5. Oppurtunistic infections in AIDS
6. Window period- lab diagnosis in window period
7. Serological detection- applications
8. Epidemiology/prevention
9. Treatment of HIV
10.Post exposure prophylaxis

Miscellaneous Viruses
1. Papova virus
2. Ebola/Marburg disease
3. German measles /Rubella virus
4. Congenital rubella syndrome
5. Prophylaxis of Rubella
6. Rotavirus- Clinical features, pathogenesis, lab diagnosis, treatment
7. Diarrheagenic viruses (essay)
8. Viral gastroenteritis
9. Viral hemorrhagic fever
10.Slow viruses, Prion disease
11.SSPE
12.Viral URT infections
13.Aseptic meningitis 144
GOVT MEDICAL COLLEGE, TRIVANDRUM
LUMINAIRE-2015

Oncogenic Viruses
1. Mechanism of viral oncogenesis
2. Oncogenic Viruses

MYCOLOGY (very imp)


1. Mycological culture media
2. Fungal stains
3. Zygomycetes
4. Pityriasis versicolor, Piedra
5. Dermatophytoses (short essay)
6. Candidiasis- local & systemic
7. Systemic candidiasis
8. Reynolds-Braude phenomenon
9. Deep mycoses
10.Mycetoma
11.Chromoblastomycoses- sclerotic bodies
12.Sporotrichosis
13.Rhinosporidiosis
14.Histoplasmosis- tuberculate spores
15.Cryptococcus neoformans
16.Oppurtunistic mycoses
17.Aspergillosis
18.Mucormycosis
19.Otomycosis
20.Oculomycosis- Mycotic keratitis
21.Mycetism/mycotic poisoning
22.Aspergillum flavus
23.Dematiaceous fungi
24.Dimorphic fungi
PARASITOLOGY
Amoebae
1. Life cycle of Entamoeba histolytica
2. Amoebic ulcer; Amoebic hepatitis; Amoebic abscess; Amoeboma
3. Intestinal amoebiasis- pathogenesis, lab diagnosis, treatment (short
145
GOVT MEDICAL COLLEGE, TRIVANDRUM
LUMINAIRE-2015

essay)
4. Extraintestinal amoebiasis- pathogenesis, lab diagnosi and treatment
(short essay)
5. Diff b/w bacillary and amoebic dysentery
6. Primary amoebic meningoencephalitis (PAM)
7. Acanthamoeba
8. Naegleria
9. Acute amoebic dysentery

Flagellates
1. Giardia lambia
2. Trichomonas Vaginalis
3. Entero test
4. Morphological stages of haemoflagellates
5. Inoculative transmission. Give example
6. Stercorarian transmission. Give example
7. Blood Incubation Infectivity Test (BIIT)
8. Life cycle & lab diagnosis of T. cruzi
9. Romana’s sign, Chagoma
10.Life cycle of L. donovani, Leishman Donovan bodies
11.Kala azar- pathogenesis, lab diagnosis, treatment and prophylaxis
12.Post Kala azar Dermal Leishmaniasis (PKDL)
13.Formol gel test, Montenegro skin test
14.Cutaneous Leishmaniasis
15.Oriental sore
16.Espundia- mucocutaneous

Malaria parasites
1. Malaria- pathogenesis, lab diagnosis, treatment and prophylaxis
2. Life cycle of Plasmodium vivax
3. Exo-erythrocytic stage
4. Erythrocytic stage
5. Hypnozoites
6. Recrudescence
7. Schuffner’s dots
8. Malignant tertian malaria/ pernicious malaria 146
GOVT MEDICAL COLLEGE, TRIVANDRUM
LUMINAIRE-2015

9. Blackwater fever
10.Tropical splenomegaly syndrome
11.Cerebral malaria
12.Merozoite induced malaria
13.QBC test

Miscellaneous Sporozoa and Microspora


1. Endodyogeny
2. Tachyzoites & bradyzoites
3. Life cycle of Toxoplasma gondii
4. Toxoplasmosis
5. Babesia
6. Life cycle of Pneumocystis carini & lab diagnosis
7. Isospora belli
8. Cryptosporidium parvum

Ciliate protozoa
1. Balantidium coli

Trematodes: Flukes
1. Diecious blood flukes
2. Hermaphroditic flukes
3. Life cycle of Schistosoma hematobium
4. Clinical features of schistosomiasis
5. Katayama fever
6. Lab diagnosis of Schistosoma hematobium, Fairley’s test
7. Cercarian hullen reaction
8. Life cycle of Clonorchis sinensis
9. Life cycle of Fasciola hepatica
10.Halzoun
11.Life cycle Paragonimus westermani

Cestodes: Tapeworms
1. Life cycle of Diphyllobothrium latum
2. Sparganosis
3. Life cycle of Taenia saginata (beef tapeworm) 147
GOVT MEDICAL COLLEGE, TRIVANDRUM
LUMINAIRE-2015

4. Hexacanth embryo
5. Cysticercus bovis
6. Life cycle of Taenia solium (pork taperworm)
7. Cysticercus cellulosae
8. Life cycle of Echinococcus granulosus
9. Hydatid cyst
10.Osseous hydatid
11.Lab diagnosis of Echinococcus
12.Casoni’s intradermal test

Nematodes: General features, Whip worm & Strongyloides


1. Name some viviparous nematodes
2. Name some oviparous nematodes
3. Name an ovoviviparous nematode
4. Life cycle of Trichuris trichura
5. Life cycle of Strongyloides
6. Larva currens

Hookworm
1. Ancylostoma duodenale- Life cycle, clinical features, diagnosis,
treatment (Essay)
2. Diff b/w Ancylostoma & Necator
3. Ground itch

Pinworm
1. Enterobius- life cycle, clinical features
2. Lab diagnosis, NIH swab
3. Oxyuriasis

Roundworm
1. Life cycle, lab diagnosis, treatment
2. Ascaris eggs
3. Loeffler’s syndrome

Filarial worms
148
1. Filariasis- pathogenesis, lab diagnosis and treatment (essay)
GOVT MEDICAL COLLEGE, TRIVANDRUM
LUMINAIRE-2015

2. Worms causing lympatic filariasis


3. Worms causing subcutaneous filarisis
4. Worms causing serous cavity filariasis
5. Life cycle of W. bancrofti
6. Microfilaria
7. Occult filariasis- Tropical Pulmonary Eosinophilia
8. DEC provocation test
9. Distinguishing features of Mf. bancrofti & Mf. malayi
10.Loa Loa
11.Onchocerca volvulus, vector

Guinea worm
1. Life cycle of Dracunculus medinensis, Infective stage
2. Dracunuculosis- clinical features, diagnosis

Miscellaneous Nematodes
1. Larva migrans- Cutaneous and Visceral

Paper I
1. A 4 yr old boy was brought to the hospital with fever and sore throat and
toxemic appearance. O/E there was pseudomembrane over the tonsillar
area and a bull neck appearance. Cervical lymphadenopathy, pallor,
tachycardia and dyspnoea present.
a. What is your diagnosis?
b. Describe the morphology and staining characteristics of the
etiological agent.
c. What staining procedures are useful in the diagnosis?
d. Pathogenesis
e. What are the toxigenicity tests used?
f. How do you culture the organism?
g. Immunoprophylaxis
h. Lab diagnosis
i. List the other organisms causing URTI

149
GOVT MEDICAL COLLEGE, TRIVANDRUM
LUMINAIRE-2015

2. A 35 yr old female was admitted with vomiting and watery diarrohea.


O/E severe dehydration was noticed and motion was like rice water. No
mucous or blood in the stools.
a. What is your diagnosis? Name the etiological agents and describe
its morphology.
b. Media used to identify the organism.
c. Treatment
d. Mention the prophylactic measures to be followed to prevent the
spread of infection. (or) What advice will you give to the family
members and the people in the neighbourhood?
e. What is the pathogenesis?
f. How will you confirm the diagnosis?
g. What are the lab findings?
h. Explain the MOA of this toxin in this illness.

3. A 10 yr old boy presented with fever and pain with swelling of joints. He
gave a h/o sore throat 5 weeks earlier.
a. What is your diagnosis?
b. Name and draw the causative organism.
c. Name and describe the important serological tests for the diagnosis.
d. Name the important toxins.
e. Describe the immunological phenomenon in pathogenesis of this
d/s.

4. 3 young men in a canal work developed flu like symptoms. About a


week later one of them started to feel worse and was hospitalized. O/E
he was febrile and jaundiced.
a. What bacterial infection he might be having?
b. What is the significance of his condition?
c. How do you confirm your diagnosis?
d. What other serious complications may occur?
e. Prophylaxis

5. A 40 yr old man was brought to the hospital with c/o fever and vomiting.
O/E there was neck rigidity and signs of meningeal irritation. There is
petechial rash all over the body. 150
GOVT MEDICAL COLLEGE, TRIVANDRUM
LUMINAIRE-2015

a. What is the probable diagnosis?


b. What is the pathogenesis?
c. Mention the lab diagnosis.
d. What is the treatment?

6. A 10 yr old boy was brought to the hospital with acute respiratory


distress and convulsions, there was a h/o penetrating injury of the leg 3
weeks back. The unhealed contaminated wound was persisting.
a. What is the clinical diagnosis? Name the etiological agent
b. Describe briefly the pathogenetic mechanism of this organism and
typical features of the convulsion.
c. How will you manage this case?
d. Prevention of this d/s in a i) new born ii) a pregnant lady in early
pregnancy iii) a 25 yr old healthy man
e. How is the pathogenicity of the organism confirmed? (lab diagnosis)

7. A 33 yr old man complaining of evening rise of temp, weight loss and


cough. O/E axillary and cervical lymph nodes palpable.
a. What is your diagnosis?
b. Lab diagnosis
c. Concentration methods
d. Culture media used.
e. Organism causing the d/s in AIDS patients.
f. Pathogenesis.

8. A 25 yr old male present with sudden onset of diplopia, dry mouth,


weakness, dysarthria and dysphagia. Last night he ate some home
canned food.
a. What is the diagnosis and what is the organism responsible?
b. What is the virulence factor?
c. Lab diagnosis and treatment
d. Enumerate the other important bacteria causing food poisoning

9. An adult male patient complains of sudden onset of fever with rigor,


dysuria and pain in the loins and flanks. Midstream clean catch urine
yielded significant growth of E. coli more than 1 lakh colonies per ml of151
GOVT MEDICAL COLLEGE, TRIVANDRUM
LUMINAIRE-2015

urine.
a. What is your diagnosis?
b. What are the bacterial causes of this clinical infection?
c. How do you identify the urinary isolates?

10.A male patient aged 25 yrs was brought to the hospital with the
following complaints. Fever, ulcer on the shaft of the penis and h/o
exposure:
a. What is the diagnosis?
b. What are the serological tests to be done?
c. What is the procedure of routine serological test done for this
condition?
d. What is biological false positive?
e. What is the non-venereal mode of transmission of this d/s?

11.A 39 yr old male consulted a doctor complaining of fever and abdominal


discomfort for one week duration. O/E coated tongue, temp 103F, pulse
65 per min, spleen tip palpable.
a. What is your diagnosis?
b. Lab investigations
c. Pathogenesis of gastrointestinal lesions.
d. Complications
e. Enumerate the specific serological tests
f. Treatment and prophylaxis.

12.A 20 yr old male presented with dysuria and urgency of micturition. O/E
he was afebrile.
a. What is your diagnosis?
b. What are the bacteriological agents?
c. How do you collect the specimen and transport?
d. How do you diagnose the condition by lab methods?
e. What are the predisposing conditions?

13.A 25 yr old female complaints of fever, gripping abdominal pain,


passage of blood and mucous in the stool. Stool culture shows non 152
GOVT MEDICAL COLLEGE, TRIVANDRUM
LUMINAIRE-2015

motile gram negative bacilli.


a. What is your diagnosis?
b. What are the causes of dysentery?
c. How do you identify the organism?

14.A 50 yr old male with varicose ulcer on medial maleolus was


hospitalized and had secondary infection. The ulcer O/E had earthy
smell and bluish pus and a swab was taken and sent for investigation.
a. What is the infection due to?
b. Nosocomial infections and problems?
c. Common infection caused by the organism
d. Treatment and management
e. Prophylaxis

15.A 43 yr old man presents with c/c cough 2 months duration, evening
rise of temp, weight loss and occasional blood staining of sputum. X-ray
revealed a patchy opacity at the apex of the right lung.
a. What is the clinical diagnosis?
b. Name the best specimen for culture. How is it collected and
processed?
c. How will you culture and confirm the diagnosis?
d. What is the immunoprophylaxis available?

16.An 8 month old child presenting with paroxysmal cough for 2 weeks.
The cough is increasing in intensity and comes in distinctive bouts
which end in a whoop. O/E there is marked leucocytosis and relative
lymphocytosis.
a. What is your diagnosis?
b. Lab diagnosis
c. Pathogenesis
d. Culture media and appearance of colony
e. Stages of d/s? Which is the most infective stage?
f. Complications and prophylaxis

17.A 25 yr old female patient was brought to the op with complaints of 153
GOVT MEDICAL COLLEGE, TRIVANDRUM
LUMINAIRE-2015

fever, malaise and loss of appetite of 8 days duration. O/E patient was
febrile, pale, tongue was coated and mild dehydration present. There
was marked hepatosplenomegaly.
a. What is your diagnosis?
b. What are the lab findings?
c. What is the causative agent and name other agents included in third
group.
d. What is the pathogenesis of this condition and where is the
organism stored in carriers?
e. What is the treatment given? Also write the prophylactic measures.

18.A 25 year old male patient with burning urination, profuse urethral
discharge. 4 days ago he had unprotected sexual contact with a
prostitute. Smear of urethral discharge reveals intracellular gram
negative diplococci.
a. What is the diagnosis and what is the organism responsible?
b. How do you diagnose the condition by lab methods?
c. Treatment of this condition.
d. Enumerate other bacteria causing sexually transmitted d/s.

19.A 25 yr old male patient came to the STD clinic with fever, myalgia and
mucocutaneous rash. He gave past h/o sexual exposure and painless
ulcer over the genitals. O/E generalized lympadenopathy was present.
a. What is your diagnosis?
b. What is the pathogenesis?
c. What is the lab test for diagnosis?
d. What is the treatment? Serological test done? Various stages of this
infection?
e. Enumerate other STDs & causative organisms

20.A 50 yr old man came to the hospital with complaints of loss of appetite,
loss of weight, cough with blood stained sputum, evening rise of temp.
a. What is your diagnosis?
b. Name the etiological agents; describe its morphology and staining
characteristics.
c. How will you grow this organism in the lab? 154
GOVT MEDICAL COLLEGE, TRIVANDRUM
LUMINAIRE-2015

d. Name the drugs used in the treatment.


e. Immunoprophylaxis against this d/s

21.A 35 yr old agricultural worker was brought to the medical op with


irregular fever, severe muscle pain, malaise and yellow discolouration of
eye of 10 days duration. O/E he was febrile, mild hepatomegaly,
subconjuctival hemorrhage present
a. What is the diagnosis?
b. What is the pathogenesis of this condition?
c. Name the etiological agent and describe its morphology.
d. Explain the epidemiology and method of lab diagnosis.
e. Treatment and prophylaxis.
f. What is the source of infection?
g. Which are the other human pathogens morphologically resembling
h. What is the mode of transmission?

22.A 5 yr old boy complaining of a/c pain and discharge from right ear.
Doctor diagnosed as acute otitis media. Later he developed muscle
rigidity, spasm and convulsions.
a. What is your diagnosis?
b. Pathogenesis
c. Management of this condition.
d. Write the prophylaxis in this condition.
Answers
1- Diphtheria, 2- Cholera, 3- Strep. pyogenes, 4- Leptospirosis, 5-
Meningococcal meningitis, 6- Tetanus, 7- TB, 8- Botulism, 9- UTI, 10-
Syphilis, 11- Typhoid, 12- UTI, 13- Dysentery (Shigella), 14- Pseudomonas, 15-
TB, 16- Whooping cough, 17- Typhoid, 18- Gonococci, 19- Syphilis, 20- TB, 21-
Leptospirosis, 22- Tetanus

Paper II
1. A 10yr old boy was brought to casualty with anxiety, irritability and
difficulty in drinking and intense thirst. Attempts to drink brings painful
spasm of pharynx
a. What is the diagnosis
155
GOVT MEDICAL COLLEGE, TRIVANDRUM
LUMINAIRE-2015

b. What is mode of transmission


c. Describe pathogenesis, lab diagnosis & prophylaxis
d. Stages of this disease, different vaccines along with schedule of
administration

2. A 4 yr old child admitted with h/o fever, sore throat and weakness of
right lower limb, on examination child has neck rigidity and signs of
flaccid paralysis. No h/o immunization
a. Diagnosis
b. Pathogenesis
c. Lab diagnosis
d. How can u prevent this disease? Give schedule of immunization with
age.

3. A 40 yr old male presented with profound weight loss and persistent


diarrhoea. O/E generalized lymphadenopathy. H/o exposure to multiple
sexual partners
a. Diagnosis
b. Draw and label the causative organism
c. Various structural antigens of the virus
d. Name organisms causing diarrhoea in this disease
e. Lab diagnosis
f. Window period

4. A 20 yr old student presented with jaundice and dark yellow urine. O/E
there was hepatomegaly and no evidence of ascitis and splenomegaly.
He also complained of nausea, low grade fever and loss of appetite.
There was a h/o accidental needle stick injury 2 months back.
a. Diagnosis
b. Causative agent
c. Lab diagnosis
d. Serological markers of the virus
e. Prophylaxis

5. A 40 yr old man came to hospital with complaints of epigastric


tenderness. O/E there was extreme pallor of mucous membrane of 156
GOVT MEDICAL COLLEGE, TRIVANDRUM
LUMINAIRE-2015

eyes, lips, tongue, with puffiness of face, edema of lower eyelid and feet.
Patient had protuberant and lusterless hair
a. Diagnosis
b. Direct and indirect methods to confirm diagnosis
c. Infective form of nematode with mode of transmission
d. Pathogenic effects
e. Treatment

6. A 60 yr old male presented with periodic attacks of fever, associated


with painful swelling of scrotal contents, arms and legs due to
lymphadenitis and lymphangitis.
a. Diagnosis
b. Enumerate tissue nematodes
c. Lab diagnosis
d. What is DEC provocative test?
e. Draw and label larval stages of different species
f. Outline life cycle

7. A 22 yr old male presented with fever with chills and rigor, sweating
episodes. Fever is periodic and showed rise in temperature every 3rd
day. He gave h/o travel outside Kerala.
a. Diagnosis
b. Pathogenesis and life cycle
c. Lab diagnosis
d. Treatment

8. A 25 year old male presented with a visible swelling of the abdomen.


O/E, it was cystic in nature and there were associated pressure
symptoms. The patient gave a history of association with dogs for the
past few years
a. What is the probable diagnosis?
b. What is the mode of transmission?
c. Describe the pathogenesis.
d. Discuss the laboratory diagnosis.
e. What is the prophylaxis?
157
GOVT MEDICAL COLLEGE, TRIVANDRUM
LUMINAIRE-2015

9. A 5 year old boy presented with history of eating sand and mud and
failure to gain weight. On examination severe pallor++, no
hepatosplenomegaly, slight pedal edema; other systems within normal
limits. Lab findings: Hb-8 mg%, TC- nl, DC showed eosinophilia-30% and
ESR- 5 mm
a. Diagnosis
b. Describe the lifecycle of the pathogen with fig.
c. How will you confirm the diagnosis?
d. What are the concentration techniques for helminthic ova?
e. Describe the management and prophylactic measures to be taken to
prevent nematode infection.

10.A 3 month old baby was brought to the paediatrics out-patient


department with complaints of passing loose stool 8 times/day, fever
and vomiting. On examination, the stool was greenish in colour.
a. Which is the most common virus causing diarrhoea in infants?
b. Describe the morphology of the virus.
c. What are the other viruses causing diarrhoea?
d. Describe the pathogenesis of viral diarrhoea.
e. How will you prove the diagnosis and treat the case?

11.30 year old business man was brought to casualty with complaints of
fever, pain radiating from the back of epigastrium of 2 days duration. He
noticed multiple vesicular eruptions confined only to left side. He gives
h/o chicken pox when he was 8 years old.
a. What is the provisional diagnosis?
b. Draw and label the causative organism.
c. Classify the groups of microbes to which the organism belongs.
d. Briefly describe the pathogenesis.
e. Mention 2 complications.
f. Name the drug used for the treatment.

12.6 year old boy with fever, running nose and cough brought to pediatric
OP. Conjunctival injection present. Bluish white ulcerations are seen on
the buccal mucosa opposite the lower molars.
a. What is the provisional diagnosis? 158
GOVT MEDICAL COLLEGE, TRIVANDRUM
LUMINAIRE-2015

b. Name the causative organism.


c. Briefly describe the pathogenesis.
d. How will you confirm the diagnosis?
e. What are the complications?
f. What prophylaxis is advised?

13.A 40year old male presented with intermittent bouts of high fever with
rigor and painful swellings of inguinal region. He gave past history of a
few similar episodes within last 3 years. He belongs to Sherlallai of
Allepey district in Kerala. O/E inguinal lymph nodes were enlarged and
tender with associated lymphangitis.
a. What is the provisional diagnosis? Name 2 etiological agents.
b. Enumerate 3 other common clinical manifestations which can be
present in this patient.
Answers
1- Rabies, 2- Polio, 3- AIDS, 4- Hepatitis B, 5- Hookworm, 6- Filariasis, 7-
Malaria, 8- Hydatid cyst, 9- Hookworm, 10- Rota virus, 11- Herpes zoster, 12-
measles, 13- Filariasis

SPOTTING
1. Lowenstein-Jensen Medium with growth
• Use- isolation of Mycobacterium tuberculosis
• Ingredients- coagulated hens’ egg, mineral salt solution
(potassium dihydrogen phosphate, magnesium sulphate),
asparagine, malachite green, distilled water, glycerol
• Functions of ingredients- Malachite green- selective media;
inhibits other bacteria, gives colour to the media
• Sterilisation - By inspissation
• Nature of colonies- dry, rough, raised, irregular with wrinkled
surface, creamy white, buff coloured colonies. (rough, tough,
buff)
• Rapid growth (within 7days)- Atypical mycobacteria
• Other media for M. tuberculosis
▪ Solid media- With egg- LJ, Petragnini, Dorset; With blood-
Tarshis; With serum- Loeffler; With potato- Pavlovsky
159
GOVT MEDICAL COLLEGE, TRIVANDRUM
LUMINAIRE-2015

▪ Liquid media- Dubos’, Middlebrook’s, Sula’s and Sauton’


media
• Reason for incubating the inoculated medium at 25oC & 37oC- M.
tuberculosis grows only at 37oc, while atypical myco grows at
both temperatures.
• Guinea pig is commonly used for animal inoculation studies .
2. Blood culture bottle
• Contains brain heart infusion broth
• Method- 5 ml blood is collected by venepuncture & inoculated
into the bottle with 50 ml brain heart infusion broth. Ideally 3
sets of samples are to be collected from different sites at
1/2 hr interval.
• Indications-1st week of typhoid fever, Infective endocarditis,
Brucellosis, IMN, Leptospirosis, Pyrexia of unknown origin
• PUO- Incubated for 24 hrs.

3. Haemagglutination plate
• Identification- Numerous pits & large depressions.
• Use- To perform the following serological investigations
a. Haemagglutination test for
▪ Influenza
▪ Parainfluenza
▪ Rubella
▪ Mumps
b. Haemagglutinaton inhibition tests
c. Titration of viruses- influenza
4. VDRL slide
• Identification- slide is 75*56 mm & 3 mm thick, 12 polished
concavities, each 16mm diameter & 1.7 mm deep
• Use- diagnosis of syphilis
• Advantage - simple, rapid, sensitive, small amount of serum is
required, can be used to assess treatment, quantitative, CSF can
also be used.
• Method- Slide flocculation-0.05ml heat inactivated (56oC for
30mins) serum is taken in depressions and a drop of freshly
prepared Cardiolipin antigen is added, mixed with syringe, slide 160
is
GOVT MEDICAL COLLEGE, TRIVANDRUM
LUMINAIRE-2015

rotated at 180 rpm in a VDRL rotator for 4min. Examine under


microscope under low power- formation of floccules or clumps
indicate reactive serum.
• Modified test with carbon particles coated with cardiolipin is RPR
(Rapid Plasma Reagin) test.
• Biological false positive- Tropical eosinophilia, Malaria, Leprosy, R
elapsing fever, RA, IMN, Hepatitis, SLE, (Code- Tropical
MALARIAHS); Chronic(>6 months)- SLE and other collagen
diseases; Acute- Acute infections and inflammations
• False negative- Sera with high titres of antibody- prozone
phenomenon, dilute sera and repeat test
5. Robertson’s cooked meat medium
• Identification- Test tube containing fat free minced cooked meat of
ox heart in infusion broth with peptone and a layer of sterile liquid
paraffin over it.
• Uses - Suitable for growing anaerobes, preservation of stock
cultures (Bacterioides, Clostridium)
• Production of gas- bubbles, change in colour of meat, turbidity of
broth indicate growth of organism
• Saccharolytic- meat turns pink (Clostridium perfringens)
• Proteolytic- meat turns black (Clostridium tetani)
6. Widal test
• Tube agglutination test for the lab diagnosis of enteric fever
• Identification- many test tubes- To estimate serum agglutination
titres of O & H antigens of Salmonella typhi and H antigen of Sal.
paratyphi A & B. Agglutination appears by the end of 1st week.
• Narrow tube with conical bottom {Dreyer’s} for H agglutination,
round bottom {Felix} for O agglutination.
• H agglutination- loose, cotton woolly clumps, O agglutination- disc
like pattern
• False positive- TAB vaccine
• False negative- Treatment with chloramphenicol
7. MacConkey’s agar
• Use- Differential media for cultivation of enteric
• Ingredients- Peptone, Lactose, Agar, Neutral red, Sod. taurocholate
(Code- PLANT) 161
GOVT MEDICAL COLLEGE, TRIVANDRUM
LUMINAIRE-2015

• Functions of ingredients:
▪ Bile salt inhibits the growth of non enteric bacteria
▪ Neutral red is the indicator
• Lactose fermenters form pink colonies eg. E. coli, Klebsiella
• Non lactose fermenters form colourless colonies eg. Salmonella,
Proteus
• Late lactose fermenters eg. Vibrio cholera, Sh. sonnei
8. Agar Agar
• Use- For solidification of media
• Agar melts at 98oC and solidifies at 42oC, obtained from sea weeds,
contains long chain fatty acids
• Adv- Bacteriologically inert, free from growth promoting and
inhibiting substances, do not add to nutritive property of the media
• Concentration- 2% for solid media, 0.2-0.5% for semisolid, 6% for
prevention of swarming of Proteus and in Nagler reaction (Cl.
perfringens)
9. Membrane Filter
• Identification- Consists of two chambers with a cellulose acetate
membrane in b/w.
• Use- Water purification & analysis, Sterilisation, Sterility testing,
Preparation of solutions for parenteral use, to filter serum,
antibiotic solution, liquid media & exudates.
• Membrane is made up of cellulose nitrate & cellulose acetate.
• Total capacity-115 ml with markings at 20, 40, 60, 80, 100. Average
pore diameter of 0.22 micrometre is widely used.
10.Seitz filter
• Consists of a disc of asbestos composition through which the fluid
is passed.
• Composed of Magnesium silicate
• Use-
a. Sterilization of serum
b. To obtain bacteria free filtrates of toxins and bacteriophages
c. Sterilization of antibiotic solution, urease medium, hydatid fluid
(Casoni’s test)
• Disadvantage- Passing of viruses & mycoplasma, carcinogenic
potential of asbestos 162
GOVT MEDICAL COLLEGE, TRIVANDRUM
LUMINAIRE-2015

11.Mandlers candle filter


• Hollow open candles made of Kieselghur, asbestos, plaster of paris
• Use- For purification of water for industrial & drinking purposes
12.Sterile cotton swab
• Identification- A stick with cotton at its end in a closed test tube.
• Use- To take specimen from throat, anterior nares, conjunctiva,
uterus and cervix.
• Method of sterilization- Hot air oven, alpha radiation, autoclave
13.Mclntosh Fildes jar
• Identification- Metallic container with a lid having outlet & inlet
tubes. Blood agar plate is kept inside. Outer tubes connected to
vacuum pump while the inner to hydrogen source. Two terminals
are present for electric supply & catalyst- modified alumina pellets
coated with palladium
• Use- for anaerobic culture
• Disadv- Risk of explosion
14.Stokes antibiotic sensitivity test
• Disc diffusion technique for antibiotic sensitivity testing.
• Both test and control organisms are inoculated on same plate with
inoculums which give semiconfluent growth. Inhibition zone size
of test can be compared with that of control, so easier to do.
• Method- Standard sensitive strains of the bacteria to be tested is
inoculated into the middle 1/3rd of the plate eg. Bacillus culture
plate. Control bacterium is inoculated into the upper and lower
1/3rd. Antibiotic discs (small 6mm discs) are applied in between
the standard and test inocula such that the zones of inhibition are
around each disc
• Interpretation-
▪ If zones are equal, test bacteria are sensitive to antibiotic
▪ if zone around test is smaller than control, moderately
sensitive
▪ if there is no zone of inhibition then test bacteria are resistant
• MIC & MBC- from dilution tests, tube dilution and agar dilution tests
• Medium used- Mueller Hinton agar/ Nutrient agar
15.Kirby bauer antibiotic sensitivity method
• Disc diffusion technique used for antibiotic sensitivity testing. 163
GOVT MEDICAL COLLEGE, TRIVANDRUM
LUMINAIRE-2015

• Done in a lawn culture of the organism being tested


• Other methods
▪ Diffusion tests- Stoke’s method, E test
▪ Dilution tests- Tube dilution, Agar dilution
16.Western blot paper test
• Use: Confirmatory test for HIV antibody
• Method- Nitrocellulose strips with HIV proteins are reacted with test
& control- strong positive, weak positive, negative sera.
• In positive sera, bands will be seen with multiple proteins, atleast 1
core and 2 envelope proteins. (p18, p24, p55, gp120, gp160, gp41)
17.Candida microscopy & Candida germ tube
• Identification- Violet coloured oval budding cells
• They are normal inhabitants of skin and mucosa
• Diseases caused- Candidiasis- opportunistic fungal infection
(predisposing factors - DM, prolonged antibiotic therapy,
immunosuppression), otomycosis, keratomycosis
• Reynolds-Braude phenomenon- Ability to form germ tubes within 2
hrs in human serum at 37oC
18.Penicillium
• Identification- Brush like arrangement of conidiophores & chains of
spores extending from the ends of short branches of
conidiophores.
• Causes Penicilliosis in HIV, Otomycosis
19.Aspergillus
• Mycelium consists of septate hyphae
• Conidiophores arise from a foot cell & terminate in a vesicle from
which flask shaped sterigmata arises
• Microconidia arranged in chains on sterigmata
• Aspergillosis, Otomycosis, Oculomycosis
20.Sabouraud’s dextrose agar with colonies
• Greenish- Penicillium
• Blackish- Aspergillus
• Creamy white, smooth- Candida
• pH of medium- 5.4
21.Beta-Hemolytic Steptococci on blood agar
• Identification- Clear, complete zone of hemolysis around 164 the
GOVT MEDICAL COLLEGE, TRIVANDRUM
LUMINAIRE-2015

colonies.
• Infections produced- Suppurative, non suppurative (Rheumatic
fever, Acute glomerulonephritis)
22.Staphylococcus aureus on blood agar
• Identification- Hemolytic golden yellow colonies, zone of complete
hemolysis surrounds the colonies.
• Diseases caused- Abscesses, skin infections, tonsillitis,
pneumonia,osteomyelitis (Streptococci, E. coli, Psuedomonas,
Klebsiella)
23.Corynebacterium diphteriae on tellurite blood agar or microscopy
• Selective medium for the isolation of C. diphtheriae, reduces
potassium tellurite to metallic tellurium producing black colonies.
• Colonies are at first small, circular, white opaque discs but enlarge
on continued incubation & acquire a yellow tint.
• Cuneiform arrangement of bacteria (Chinese letter pattern)
• Special stains- Albert’s stain, Neisser’s, Ponder’s
•Other selective media for the same bacteria- Loeffler’s serum slope
•Morphological classification of the bacteria grown in this medium-
gravis, intermedius and mitis.
• Volutin granules
24.Pseudomonas on nutrient agar
• Identification- Green diffusible pigment production of Pseudomonas
aeruginosa on nutrient agar.
• Pigments produced- Pyocyanin, Pyoverdin, Pyomelanin, Pyorubin
• Diseases caused
25.Proteus- swarming growth on blood agar
• Culture media- Blood agar, MacConkey agar (no swarming)
• Characteristic odour- fishy odour
• Methods to inhibit swarming- (i) increase concentration of agar
(6%) (ii) Incorporate chloral hydrate (1:500), sodium azide (1:500)
[No swarming in MacConkey agar.]
26.Demonstration of capsule- Pneumococci
• Identification- Diploocci with clear halo around it
• Diseases caused- Pneumonia, otitis media, sinusitis, meningitis
• Organism used for isolation- Mice
• Other uses of negative staining- Demonstration of capsule of
165
GOVT MEDICAL COLLEGE, TRIVANDRUM
LUMINAIRE-2015

Cryptococci, Spirochaete demonstration (Negative staining for


spirochetes different from that used for capsule demonstration)
27.Microfilaria
• Identification- Larvae of filarial worm in peripheral smear.
• Differences between microfilaria of W. bancrofti and B. malayi
• Blood is collected between 10 pm and 4 am because of its nocturnal
periodicity
28.Aerobic spore bearer
• Identification- Gram positive bacilli arranged in long chains with a
central spore.
• Eg. Bacillus anthracis, Nocardia
• Destruction of spores by Duckering process, using 2%
formaldehyde.
29.Clostridium tetani
• Identification- Gram positive bacillus with rounded, bulging terminal
spores.
• Culture medium- Robertson’s cooked meat medium.
• Toxigenicity test in tail of mouse
• Toxins produced- Tetanospasmin, Tetanolysin (toxin responsible for
tetanus - tetanospasmin)
30.Gram negative bacilli
• Identification with examples (E. coli, Klebsiella, Salmonella)
• Enumerate diseases caused- UTI, diarrhoea, septicemia
• Medium used for culture- MacConkey’s agar
31.Satellitism (pic)
• For identification of H. influenza
• Blood agar deficient in factor V
• Staph. aureus streaked across a plate of blood agar inoculated with
H. influenza
• H. influenza colonies near the Staph. aureus colonies are bigger in
size
32.Castaneda’s method of blood culture (pic)
• Biphasic medium with both solid and liquid media
• Use- Brucella isolation and Salmonella culture
• Adv- Minimum material, manipulation, contamination, lab worker
infection, for slow growing organisms 166
GOVT MEDICAL COLLEGE, TRIVANDRUM
LUMINAIRE-2015

33.Elek’s gel precipitation test (pic)


• Virulence test for C. diphtheriae
• On 20% horse serum agar
• Animal used for inoculation studies- guinea pig, rabbit
34.HIV (pic)
• Genes in HIV- env, gag, pol (structural genes)
• HIV proteins- p24, gp120, gp41
• Screening test & Confirmatory test
35.Ascaris egg- fertilized
• Infective form of the parasite- egg containing rhabditiform larva
• Methods of diagnosis- demonstration of eggs in feces, larvae
and Charcot Leyden crystals in sputum.
• Chemotherapy- Pyrantel pamoate, Albendazole
• Methods to prevent infection- treatment of vegetables with iodine
200 ppm for 15 minutes kills egg and larvae.
36.Trichuris trichura (whip worm) egg
• Identifying features- barrel shaped, mucus plug on both sides,
bile stained
• Infective form- embryonated egg
• Part of the intestine where adult form of the parasite reside-
caecum
• No intermediate host
37.Cyclops
• Vector for
a. fish tape worm (1st stage larva- coracidium)
b. guinea worm (3rd stage larva)
38.Tapeworm
• Infective form form for humans- Cysticercus cellulosae or bovis
• Intermediate host- Pig or Cow
• Peculiarity of embryo- 3 pairs of hooklets
39.Pinworm
• Infective form- Egg containing larvae
• Egg- Planoconvex, double layered, colourless

STAINING PROCEDURES
167
GOVT MEDICAL COLLEGE, TRIVANDRUM
LUMINAIRE-2015

I. GRAM STAINING
1. Place the slide on the staining rack
2. Cover the slide with Gentian violet/Methyl violet/Crystal violet & keep
it for 1 min.
3. Wash with water, cover the slide with Gram’s iodine & keep it for 1
min.
4. Wash the slide with water
5. Decolorise with acetone, keep the slide in a slanting position and add
acetone drop by drop for 2-3 sec (until the draining fluid is colourless)
6. Immediately wash with water.
7. Cover the slide dil. carbol fuschin & allow to stand for 30 sec.
8. Wash thoroughly with water, blot & dry in air.
9. Examine under oil immersion.
10.Write down observations

II. ZIEHL-NEELSEN STAINING [ACID FAST STAINING]


1. Place the slide on the staining rack
2. Cover the smear with con. carbol fuschin. Heat until steam rise. Do
not boil. Allow the preparation to stain for 7 min, heat being applied at
intervals to keep the stain hot. The stain must not be allowed to
evaporate or dry. If necessary pour more con. carbol fuschin to cover
it.
3. Wash with water
4. Cover the slide with 20% Sulphuric acid. Keep it for 1 min.
Decolorisation is finished when the film is colorless or faintly pink.
5. Wash with water
6. Counterstain with Loeffler’s methylene blue for 3 min.
7. Wash with water, blot, dry.
8. Examine under oil immersion.
9. Write down observations as follows
a. Colour, Shape, Arrangement, Inference (The given smear
contains....)
b. Draw diagram ( don’t draw pus cells if not seen in field of
observation)
168
GOVT MEDICAL COLLEGE, TRIVANDRUM
LUMINAIRE-2015

▪ Acid fast staining- Mycobacterium tuberculosis (with 20% sulphuric


acid)
▪ Modified acid fast staining-
a. M. lepare (with 5% sulphuric acid)
b. Nocardia (with 1% sulphuric acid)
c. Spores (with 0.25-0.5% sulphuric acid)
d. Cryptosporidium parvum, Isospora belli (cold acid fast staining)

▪ ZN SMEAR EVALUATION AND AFB REPORT

No. of AFB Seen in oil immersion field (F) Report


0 300 F AFB not seen
1-9 100 F 1+
1-9 10 F 2+
1-9 1F 3+
10 or more 1F 4+

OSPE

  Case History Spotter kept

TB Cough for 2 weeks AFB staining or culture media


Staph. Aureus Breast abscess, Post operative infection Microscopy / culture
Streptococcus Rheumatic fever Microscopy / culture
UTI Dysuria, Fever with chills Lactose fermenters (MacConkey agar),
GNB
Typhoid Fever, coated tongue, hepatosplenomegaly Widal test results, NLF (MacConkey
agar)
Diphtheria Sore throat, Membrane over tonsil Microscopy / culture
Bacillary Fever, mucus and blood in stool NLF (MacConkey agar)
dysentery
Filaria Fever with lymphadenitis and lymphangitis Microfilaria
Syphilis Painless ulcer on genitalia VDRL slide
HIV Weight loss, Gen. Lymphadenopathy Picture of HIV
HIV + Candida Oral thrush Microscopy / culture
Dengue Fever, retroorbital pain Serology (IgM)
169
GOVT MEDICAL COLLEGE, TRIVANDRUM
LUMINAIRE-2015

PUO   Blood culture bottle

Sterilisation    

FORENSIC
MEDICINE
QUESTION PAPER PATTERN

ESSAY – 7 marks

SHORT NOTES – 5 x 3 = 15 marks

ANSWER BRIEFLY – 5 x 2 = 10 marks

DIFFERENTIATE BETWEEN – 2 x 2 = 4 marks


170
GOVT MEDICAL COLLEGE, TRIVANDRUM
LUMINAIRE-2015

DIAGRAM – 2 x 2 = 4 marks

DIAGRAM

1. Ossification centres of shoulder joint


2. Ossification centres of wrist joint
3. Ossification centres of hip joint
4. External appearance of wounds caused by a single edged weapon & rubber
tapper’s knife
5. External signs seen on head and neck in attempted hanging
6. LS of a shot gun, shotgun catridge
7. Parts of catridge of a rifled weapon
8. Types of hymen
9. Sex chromatin
10. LS of revolver catridge
11. Entry wound by a knife
12. Estimation of extent of body surface burns
13. Belly scales of snake
14. Shot gun wounds at varying distance
15. Sperm
16. Signs of strangulation
17. Human hair and animal hair

INTRODUCTION & LEGAL PROCEDURES


1. Forensic medicine, legal medicine
2. Medical Jurisprudence
3. Medical etiquette
4. Inquest & procedure
5. Situations where magistrate inquest is required
6. Courts of Law and their powers
7. Punishments
8. Cognisable offence
9. Subpoena/Summons/Litigation
10. Conduct Money
11. Dying Declaration, compos mentis
12. Dying Deposition
171
GOVT MEDICAL COLLEGE, TRIVANDRUM
LUMINAIRE-2015

13. Oral evidence and exceptions


14. Witness & types
15. Hostile witness
16. Perjury
17. Steps in record of evidence
18. Leading question
19. Summons case
20. Warrant case

MEDICAL LAW AND ETHICS


1. Function of Indian and state medical councils
2. Warning notice, Penal erasure
3. Infamous conduct
4. Dichotomy, Covering
5. Duties of medical practioner
6. Professional secrecy and Privileged communication
7. Professional negligence- types, difference
8. When does liability for negligence arise? (4 D’s)
9. Res Ipsa loquitur
10. Calculated risk
11. Novus actus intervenience
12. Difference between professional negligence & infamous conduct
13. Medical maloccurance
14. Corporate negligence
15. Contributory negligence
16. Defence against neglence
17. Therapeutic misadventure
18. Vicarious liability,
19. Products Liability
20. Euthanasia
21. Consent in medical practice- Types, MLI
22. Therapeutic privilege
23. Loco parentis
24. Malingering
25. Consumer protection act (COPRA 1986)

IDENTIFICATION
1. Corpus delicti
172
GOVT MEDICAL COLLEGE, TRIVANDRUM
LUMINAIRE-2015

2. Cephalic Index, types of skulls


3. Sex chromatin
4. Sex difference in bones
5. Medullary Index
6. Intersex, Concealed sex
7. Gustafson’s method
8. Difference between temporary & permanent teeth
9. Eruption of teeth (Age)
10. Diff b/w mandible in infants, adult and old age
11. MLI of Age (esp 10, 12, 14, 15, 16, 18, 21)
12. Age of fetus
13. Rule of Haase
14. Anthropometry
15. Dactylography
16. Types of fingerprints
17. Poroscopy, podogram
18. Cheiloscopy
19. Super imposition
20. Scars MLI
21. Tattoo marks MLI, Removal, Dyes
22. Difference between human & animal hair-Diagram, MLI
23. Forensic odontology,MLI
24. Bite marks
25. FDI two digit system, modified FDI system

MEDICO LEGAL AUTOPSY


1. Skin incisions
2. Objectives of medicolegal autopsy
3. Method of removal of organs
4. Post mortem clots
5. Unclotted blood- conditions
6. Undertaker’s fractures
7. Preservation of viscera
8. Preservatives
9. Conditions where preservation is not necessary
10. Ostometric board
11. Karl pearson formula
12. Exhumation & its authorization
173
GOVT MEDICAL COLLEGE, TRIVANDRUM
LUMINAIRE-2015

DEATH AND ITS CAUSES


1. Somatic death, Bishop’s tripod of life
2. Brain death, Moment of death
3. Harvard criteria of brain death.
4. Molecular death
5. Asphyxia and asphyxial stigmata
6. Tardieu spots
7. Classification of cause of death
8. Causes of sudden death
9. Negative autopsy
10. Obscure autopsy
11. Concealed trauma
12. Signs of death
13. Suspended animation - causes, MLI
14. Changes in eye- Tache noir, Kevorkian sign
15. Postmortem fluidity of the blood
16. Postmortem cooling of body
17. Postmortem caloricity
18. Postmortem hypostasis - synonyms, cause, fixation & nonfixation, MLI
19. Diff bw postmortem hypostasis and congestion
20. Primary flaccidity & secondary flaccidity
21. Rigor mortis - mechanism of production, time, order of appearence, MLI
22. Conditions simulating rigor mortis
23. Cadaveric spasm- MLI, difference from rigor mortis
24. Putrefaction – cause, bacteria associated with, enzymes, features of
putrefaction, site,factors affecting
25. Marbling
26. Glove - stocking appearance.
27. Casper’s dictium
28. Adipocere - morphology, cause, predisposing factor, MLI
29. Mummification — morphology, cause, predisposing factor, MLI
30. Maceration (asepctic autolysis, signs of dead born, refer chapter—infant death)
31. Embalming, embalming fluid

MECHANICAL INJURIES
1. Define injury
2. Classification of injuries
174
GOVT MEDICAL COLLEGE, TRIVANDRUM
LUMINAIRE-2015

3. Abrasion & types


4. Patterned abrasion
5. Brush burn (mech of production)
6. Pressure abrasion
7. Age of abrasion
8. Difference between AM & PM abrasion
9. MLI of abrasion
10. Contusion, MLI
11. Ectopic bruising, come-out bruise
12. Patterned bruising
13. Age of bruise
14. Difference between hypostasis & bruising
15. Define laceration, types
16. Differences between incised looking lacerated wound & incised wound
17. Incised wound & MLI, tailing of wound
18. Beveling cut, MLI
19. Difference between suicidal & homicidal cut throat
20. Self inflicted and fabricated wounds
21. Hesitation marks
22. Complications in a cut throat injury
23. Chop wound
24. Defence wounds
25. Stab wound, punctured wound, Complications
26. Shapes of wounds, Kinds of weapons used
27. Fishtailing
28. Penetrating and Perforating wounds
29. Concealed puncture wound
30. MLI of stab wound, Important dimensions
31. Difference between incised & lacerated wound

FORENSIC BALLISTICS
1. Rifling & its advantages.
2. Choking of a shotgun, Paradox guns
3. Calibre and gauge
4. Composition of priming mixture
5. Powder (1) black powder constituents (2) smokeless powder constituents
6. Dum dum bullet
7. Shot gun wounds at varying distance, Diagram
8. Features of contact wound for shot gun 175
GOVT MEDICAL COLLEGE, TRIVANDRUM
LUMINAIRE-2015

9. Wounds from rifled weapons- assessment of range


10. Backspatter, corona, abrasion collar, dirt collar
11. Contusion collar, point blank
12. Puppe’s rule
13. Dermal nitrate test, MLI
14. Difference between entrance & exit wound
15. Supported exit wound
16. Ricochet bullet, exhibit bullet, test bullet
17. Primary & secondary markings of a bullet
18. Neutron activation analysis
19. Blast lung
20. Tandem bullet, yawning bullet, tumbling bullet

REGIONAL INJURIES
1. Causes of black eye
2. Types of fracture of skull
3. Depressed fracture of skull, MLI
4. Signature fracture, Pond fracture, Gutter fracture
5. Contrecoup injury
6. Commotio cerebri, Commotio cordis
7. Lucid interval
8. Causes & features & MLI- Extadural, Subdural,Sub arachnoid, intra cerebral
haemorrhage
9. Apoplexy
10. Punch drunk syndrome
11. Whiplash injury
12. Railway spine
13. Concussion of brain
14. Injuries of pedestrains (Primary impact injuries)
15. Bumper injury
16. Under running / tail-gating
17. Motor cyclist’s fracture
18. Fracture of pontine haemorrhage
19. Buckling up of sterum
20. Steering wheel impact type of injury
21. Seat belt syndrome
22. Triage
23. Cardiac tamponade
24. Boxer fracture 176
GOVT MEDICAL COLLEGE, TRIVANDRUM
LUMINAIRE-2015

MEDICO LEGAL ASPECTS OF WOUNDS


1. Define injury (Sec 44 IPC)
2. Murder (Sec 300 IPC)
3. Dowry death (Sec 304-B IPC)
4. Homicide
5. Culpable homicide
6. Hurt ( Sec 319 IPC)
7. Grevious hurt (Sec 320 IPC)
8. Dangerous weapon
9. Assault
10. Medicolegal classification of injury
11. Classification of Fatal/ Dangerous Injuries
12. Fat embolism
13. Air embolism
14. Cause of death from wounds
15. Primary shock
16. Secondary shock
17. Shock lung
18. Remote causes of death from wounds
19. Crush syndrome
20. Difference between AM & PM. Wounds

THERMAL DEATHS
1. Trench foot, Immersion foot
2. Frost bite
3. Heat stroke
4. New classification of burns
5. Rule of nine
6. Pugilistic attitude
7. Heat haematoma
8. Scalds
9. Thermal fractures of skull
10. Difference between external haematoma due to burns & due to blunt force
11. Difference between antemortem & postmortem burns
12. Joule burns
13. Crocodile flash burn
177
GOVT MEDICAL COLLEGE, TRIVANDRUM
LUMINAIRE-2015

14. Current pearls


15. Bone pearls / wax drippings
16. Filigree burns
17. Cause of death in burns, Electrocution
18. Artifacts due to burns
19. Healing of burns.

MECHANICAL ASPHYXIA
1. Definition of hanging
2. Partial hanging , Typical hanging, Atypical hanging
3. Features of AM hanging
4. Causes of death in hanging & commonest cause
5. Features of Asphyxial death- cyanosis, congestion, petechial h’ge, edema,
fluidity of blood
6. Le-facie sympathique & its mechanism
7. Lynching
8. Strangulation –Types, cause of death, mechanisms
9. Diagram—signs of strangulation
10. Differentiating features between strangulation & hanging
11. Pseudo strangulation, Subpleural emphysematous bullae
12. Bansdola / garroting / mugging / throttling /smothering / overlying / burking/
Gagging
13. Define suffocation
14. Commando punch
15. Hyoid bone fractures — type & its MLI
16. Café coronary & mechanism of death
17. Traumatic asphyxia
18. Drowning & types
19. Immersion syndrome
20. Difference between drowning in fresh & sea water
21. Cutis anserine, Cadaveric spasm
22. Patho- physiology & mechanism of death in fresh & sea water drowning
23. Emphysema aquosum and odema aquosum
24. Paltauf’s hemorrhage
25. Dry drowning
26. Gettler’s test
27. Diatom test
28. Sexual asphyxia
178
GOVT MEDICAL COLLEGE, TRIVANDRUM
LUMINAIRE-2015

29. Wrinkling, bleaching, soddening- Times of occurrence

IMPOTENCE AND STERILITY


1. Impotence
2. Sterility
3. Frigidity
4. Sterilization
5. Surrogacy
6. Artificial Insemination
7. Causes (permanent & temporary) of impotence in male & female, MLI
8. MLI of impotence and sterility
9. Quoad hoc
10. Artificial Insemination- Indications, types, precautions, legal problems
11. Sterilization- Indications, types
12. Fecundation ab extra
13. Indications, legal problems of surrogate motherhood
14. Lerische ssyndrome

VIRGINITY, PREGNANCY AND DELIVERY


1. Define virgin, virginity, defloration, divorce, nullity of marriage
2. Hymen, types of hymen with figure
3. How to differentiate between tear of hymen produced by sexual offences &
notched hymen?
4. Cause of rupture of hymen
5. MLI of hymen, virginity
6. Features in genitalia of a virgin
7. MLI of pregnancy
8. Affiliation
9. Presumptive signs , probable signs & positive signs of pregnancy
10. Lochia and its types
11. What is (a) Chadwick’s sign (b) Hegar’s sign (c) Goodell’s sign (d) Osiander’s
sign & when do they appear?
12. Radiological signs of fetal death in utero
13. Pseudocyesis / phantom pregnancy
14. Pseudo virgin
15. Posthumous child
16. Superfecundation, superfoetation, supposititious child
17. Prenatal diagnostic act :-a) objectives (b)punishment
179
GOVT MEDICAL COLLEGE, TRIVANDRUM
LUMINAIRE-2015

18. Legitimacy, MLI


19. Estimation of paternity of a child , Disputed paternity
20. Difference between parous and nulliparous uterus

SEXUAL OFFENCES
1. Classify sexual offence
2. Define rape, punishment (Sec 375, 376 IPC)
3. Statutory rape
4. Features of a voluntary consent
5. Objectives of doing medical examination
6. Examination of a victim- features looked for, calculation of time of assault, s
pecimens collected
7. Examination of the accused
8. Hymenal tear & its healing
9. Corroborative signs of rape
10. Incest
11. Unnatural sexual offences
12. Sodomy ( & its other names), procedure of examination ,MLI
13. Buccal coitus
14. Sexual perversions.
15. Tribadism
16. Bestiality
17. Uranism,
18. Sadism
19. Lust murder
20. Necrophagia
21. Masochism
22. Fetichism
23. Transvestism
24. Exhibitionism
25. Voyerism
26. Troilism
27. Frotteuerism
28. Undinism
29. Indecent assault
30. Catamite
31. Chemical tests for semen
32. Microscopic examination, Motility of sperm
33. Proof of semen 180
GOVT MEDICAL COLLEGE, TRIVANDRUM
LUMINAIRE-2015

ABORTION
1. Abortion - classify
2. Justifiable &criminal abortion
3. Difference between natural and criminal abortion
4. Any 4 sections of IPC dealing with criminal abortion.
5. MTP act – indications, Who can do abortion & where
6. Duties of a doctor in case of therapeutic & criminal abortion
7. Enumerate the methods of therapeutic & criminal abortion
8. Abortifacient drugs
9. Abortion stick
10. MLI of placenta
11. Complications of criminal abortion
12. Causes of death in abortion
13. Punishment for criminal abortion (Sec 312 – 316 IPC)

INFANT DEATHS
1. Define infanticide, Stillbirth, live birth
2. Viability
3. Dead birth & its features
4. Spalding sign
5. Difference in lungs before & after respiration
6. Ploucquet’s test
7. Hydrostatic lung test (Raygat’s test) – conditions where it is unnecessary,
fallacies
8. Breslaus second life test, Wredin’s test
9. Caput succedaneum
10. Cephal hematoma
11. Precipitate labour, MLI
12. Difference between head injury due to labour and blunt force
13. Battered baby syndrome ( Cafey’s syndrome)
14. Concealment of birth
15. Abandoning of infants
16. Infantile whiplash syndrome
17. Munchhausen’s syndrome by Proxy
18. Sudden infant death syndrome (imp)
19. Ossification centres in sternum, sacrum
20. Umblicical cord changes after birth
181
GOVT MEDICAL COLLEGE, TRIVANDRUM
LUMINAIRE-2015

21. Non accidental trauma – Battered baby, Munchausen’s, SIDS

BLOOD STAINS
1. Screening tests for blood – benzidene, phenolphthalein, o-toluidine,
mucomalachite green tests
2. How will you differentiate human blood stain from other species by microscopic
examination of blood?
3. Examination of blood stains- Age of blood stains,sex and age of person,living or
dead body.
4. Benzidine test
5. Confirmatory test for blood
6. Teichmann’s test
7. Takayama test
8. Precipitin test
9. MLI of blood group
10. DNA finger printing
11. Difference between AM & PM blood stains
12. Microscopic appearance of RBC’s - diagram
13. PCR v/s RFLP

FORENSIC PSYCHIATRY
1. Delirium
2. Delusion & its types
3. Hallucination & its types
4. Illusion
5. Impulse & types
6. Phobia- types
7. Lucid interval
8. Diff b/w lucid interval in insanity & head injury
9. Diff b/w psychosis & neurosis
10.Diff b/w real and feigned insanity
11.Restraint of the insane
12.Delirium tremens
13.Civil & criminal responsibilities of an insane person
14.Testamentory capacity
15.Mc naughten’s rule - Section 84 IPC
16.Automatism
17.Post - epileptic insanity
182
GOVT MEDICAL COLLEGE, TRIVANDRUM
LUMINAIRE-2015

18.Drunkenness & law [Sec 85,86 IPC]

FORENSIC SCIENCE LABORATORY


1. Locard’s exchange principle
2. Difference between colostrum & milk
3. Microscopic appearance of blood
4. Confirmatory test for semen
5. Polygraph
6. Narco analysis- Truthserum drugs

ARTEFACTS
1. Resuscitation of artefacts
2. Artefacts related to rigor mortis and burns.

TOXICOLOGY
1. Toxicology and poison, classification of poisons
2. Duties of a doctor in suspected case of poisoning
3. Gastric lavage – method, C/I
4. Preservation for organs in poisoning
5. Antidotes – Types
6. Universal & physiological antidotes
7. Chelating agents
8. Name some poisoning substances used as abortificents
9. Contraindications in corrosive poisoning
10. Give examples for - stupefying agents, Cattle poisons, Household emetics
11. Corrossive poisons
12. Deliriants
13. Diaphoretics
14. Activated charcoal

AGRICULTURAL POISONS
1. Organo phosphorus poisoning – MOA, manifestation, causes of death,
diagnosis, management, PM findings
2. Cholinesterase reactivators
3. Plant penicillin
4. Zinc phosphide, Al phosphide

CORROSIVE POISONS 183


GOVT MEDICAL COLLEGE, TRIVANDRUM
LUMINAIRE-2015

1. Sulphuric acid poisoning – signs and symptoms, Causes of death , Rx, PM


features
2. Vitriolage and treatment
3. Nitric acid poisoning, Xanthoproteic reaction
4. HCl
5. Oxalic acid
6. Formic acid poisonings
7. Carboluria
8. Ochronosis

METALLIC POISONS
1. Arsenic poisoning - MOA, Rx, specific antidote
2. Viscera preserved in arsenic poisoning
3. Difference between arsenic poisoning & cholera
4. Hg poisoning –treatment
5. Signs & Symptoms of plumbism
6. Cu poisoning
7. Aldrich -mess lines
8. Rain drop pigmentation
9. Mercurial erithism
10. Hatter’s hakes
11. Mercuria lentis
12. Acrodynia
13. Punctuate basophilia
14. Bartonian lines

INORGANIC IRRITANT POISONS


1. Phossy jaw
2. Antidote in P poisoning
3. Difference in P poisoning and acute yellow atrophy
4. Difference between red and white phosphorus

ORGANIC IRRITANT POISONS


1. Toxalbumin / phytotoxin – examples
2. Active principles in (1)Ricinus communis (2)Abrus precatorius (3)semecarpus
anacardium (4) Calotropis
3. Suis
4. Ergotism
184
GOVT MEDICAL COLLEGE, TRIVANDRUM
LUMINAIRE-2015

5. Difference between poisonous & non-poisonous snakes, their bite marks


6. Belly scales
7. Difference between Cobra & Viper
8. Clinical features of Cobra bite & viper bite
9. Snake venom
10. Snake bite management
11. Anti- snake venom, dosage

CNS DEPRESSANTS
1. Factors affecting alcohol absorption
2. Metabolism of alcohol
3. Mc ewan’s sign , alchohol gaze nystagmus
4. Critical level of alchohol in blood
5. Define drunkenness
6. Collection of blood in alcoholics – precautions
7. Widmark’s formula, Henry’s law
8. Methods to determine blood alcohol
9. Alcoholic blackout
10.Delirium tremens
11.Korakoff’s psychosis
12.Rx of acute & chronic alchoholism
13.Rx of methanol poisoning
14.DD of alcoholism
15.Level of alchohol in blood above which driving is not allowed
16.DD of contracted pupil due to poisoning
17.DD of dilated pupil
18.Micke finn
19.Disulfiram reaction
20. Wernickers encephalopathy
21. Barbiturate blisters
22. Knock out drops

PSYCHOTROPIC DRUGS
1. Flashback phenomenon-LSD
2. Benzodiazepines

DELIRIANT POISONS
1. Toxins in Datrura, mechanism of action, symptoms ( 8D’s)
185
GOVT MEDICAL COLLEGE, TRIVANDRUM
LUMINAIRE-2015

2. Management of Datrura poisoning


3. Active principles in Cannabis
4. Run amok
5. Street poison
6. Cocaine bugs ( Magnan’s phenomenon)

DRUG DEPENDENCE AND DRUG ABUSE


1. Drug & drug dependence
2. Substance abuse
3. Substance dependence
4. Difference between drug addiction & drug habituation
5. Withdrawal symptoms
6. Body packer & Body stuffer

SPINAL POISONS
1. Active principles in Strychnine, mechanism of action
2. Signs & symptoms in Strychnine poisoning, Rx, Antidote
3. Difference between Strychnine poisoning & tetanus

CARDIAC POISONS
1. Active principles in nerium odorum, cerbera thevetia, cerbera odallam
2. Cerbera odallam – signs & symptoms, ECG changes, management
3. BHIST regimen
4. Aconite poisoning (Hippus)

ASPHYXIANTS
1. Critical level of CO in blood
2. Postmortem appearance & staining in CO poisoning, Reason
3. Tear gases
4. HCN poisoning – signs & symptoms, Mechanism of action, Fatal dose
5. Management of HCN poisoning – Antidote & its mechanism of action
6. Food poisoning
7. Active neurotoxic principles in Lathyrus sativus
8. Botulism
9. Ptomaines

1. An adult male was brought to the casuality dead with history of sudden loss of186
GOVT MEDICAL COLLEGE, TRIVANDRUM
LUMINAIRE-2015

consciousness .Postmortem examination revealed corrosion of stomach and


lower part of oesophagus with smell of bitter almond.Blood was bright red in
colour .
a)what is your diagnosis ?
b) mechanism of death ?
c)what are the important clinical features in this case?
d)briefly outline the treatment in this case
e)what is the antidote?
2. A 50 year old female was found dead inside a well at about 6am on 31-3-07 at
lokarnakavu temple premises .Recovery of the body from the well reaveled the
same in two pieces .Head and portion of neck amputed at the level of 3rd
cervical vertebra and rest of the body .A plastic rope with soft tissues and scalp
hairs entangled in it .postmortem examination revealed a lacerated wound
27.5cm all around the neck at the level of 3rd intervertebral .the amputed pieces
could be well approximated.
a)what is the mode of production of injury?
b)what is the cause of death?
c)what is the circumstance of death?

3. A 50 year old female was brought dead to hospital with history of poisoning
.during postmortem examination there were blackish discolouration and
thickening of mucosa of mouth,oesophagus ,stomach and small intestine.
Stomach contained brownish black fluid with pungent smell.
a)what is your diagnosis?
b)what are the poisons producing the above change?
c)what is the antidote?
d)enumerate the line of management
e)what is VITRIOLAGE?

4. A 45 year old female nurse was brought dead to the hospital with history of
excessive intake of some tablets .postmortem examination revealed multiple
white tablets and powdery particles in the stomach .Anoxic bullae were present
on front of both knees and back of elbows .Internal capsules of brain showed
softening.
a)wht is your diagnosis?
b)what is the line of management?
c)what is the fatal dose of the poison in this case?
d)what are the materials send for chemical analysis in this case?
e)what are the duties of the doctor attending this case at the casualty of187
GOVT MEDICAL COLLEGE, TRIVANDRUM
LUMINAIRE-2015

hospital?

5. A 30 year old female was brought to the hospital with history of alleged
consumption of the kernel of a mango like fruit .she complained of nausea
,vomiting and abdominal pain .on examination there were signs like
bradycardia,irregular pulse,and respiration.
a)what is your diagnosis?
b)mention the names of the toxic principles contained in this fruit
c)what is the line of management?
d)what could be the likely findings at the autopsy?
e)how will you preserve the viscera and tissues for chemical analysis?

6. A 45 year old man was brought to the hospital with watery diarrhoea
,tenesmus,pain and irritation around the anus .The stools were initially dark
coloured ,bloody and foul smelling,but later on became involuntary,colourless
and watery.The provisional diagnosis was food poisoning(bacterial)but the
patient suspected homicidal poisoning and gave a statement to that effect
before lapsing in to coma.
a)what is your diagnosis?
b)what are the skin manifestations in chronic poisoning?
c)what is the specific antidote ?
d)what is the line of management?
e)what are the specific tissues to be sent for analysis in this poisoni

7. A 20 year old young man was brought to the hospital with a history of a bite by a
reptile on the leg,ten minutes ago.on examination there were two small puncture
marks on the rt leg with pain and swelling around.He complained of
progressively increasing weakness of the muscles of limbs and face.
a)what is your diagnosis?
b)what is the line of management?
c)how will you test for the sensitivity of the antidote?
d)draw a diagram showing the peculiarity of the head scales of the
involved reptile.
e)if this patient dies,what special tissues will you collect for chemical
analysis and what presevatives will you use?
f) Diff b/w poisonous & nonpisonous snakes& their bite marks.

8. A person was brought to the casulty in an unconscious state with history of


188
consumption of poison.it is alleged that the poison was forcibly administered.on
GOVT MEDICAL COLLEGE, TRIVANDRUM
LUMINAIRE-2015

examination it was found that pupils were consticted and there was muscle
twitching with smell of kerosene in the breath.
a)mention the legal responsibility of doctor attending thus case
b)what is your diagnosis?
c)mechanism of action of this poison
d)what are the important clinical features?
e)what are the antidotes that can be given?

9. A twenty year old woman died of burns after 2 years of her marriage at her
home. The body was in flexed attitude, all fingers were hooked like claws. The
clothing and skin showed evidence of charring, in several places. The following
parts showed burns. Head and neck, both upper limbs, front of the trunk and
both lower limbs. The internal organs are congested. Brain showed a “cooked”
appearance, stomach empty, uterus contained a foetus of 16cm length.
a.Who should conduct the inquest and why?
b.What is the term used for the flexed attitude of the body and its MLI
c.Calculate the percentage of burns in this case.
d.What is the gestational age of the fetus?
e.What is the cause and manner of death?

10. The body of a male recoverd from a paddy field showing post mortem lividity of
the left side of the face, trunk, left upper and lower limbs( outer aspects ) and
fixed. Rigor present all over the body. Blood was oozing out of the left ear.
Contusion 5x2cm obliquely placed was present on the outer aspect of right
forearm at about its middle.
a.What type of inquest should be conducted
b.What steps have to be taken to identify the deadbody?
c.What was the position of the body when it was recovered?
d.What was the time since death?
e.What is the reason for bleeding from left ear?

11. The body of a 20 year old female is recovered from a river showing the clothes
wet, rigor was present in the lower half of the body. Right hand was clenched
tightly. Postmortem lividity seen on the face, front of the neck and the dependent
parts of upper and lower limbs. Froth is seen oozing out of the mouth and nose.
Uterus contained a fetus 9 cm long and 30gms in weight.
a.What is the time since death?
b.What is the probable cause of death?
c.What was the position in which the body was floating? 189
GOVT MEDICAL COLLEGE, TRIVANDRUM
LUMINAIRE-2015

d.Comment on the clenched right hand?


e.What is the age of the fetus? Describe the method of calculation?

12. A 3 year old child was brought to casualty with the history of swallowing some
seeds. The parents described them as soft spiny small seeds, blue-green or
rose-red in color. The child on examination showed increased salivation, weak
and rapid pulse and was drowsy. The symptoms included nausea vomiting and
bloody diarrhea with abdominal pain and cramps.
a.What is your diagnosis?
b.What is a toxalbumin?
c.What is the mechanism of action?
d.What is the line of management?
e.What are the post mortem appearances?
SPOTTERS
1. Zinc phosphide (Rat Poison)
a. FD: 5gm; F.P:24hours
b. M.0.A: on hydrolysis with dilute HC1 of stomach, phosphine is liberated
which acts as a powerful respiratory poison.
c. C/F: vomiting with retching, tremor, drowsiness, respiratory distress,
d. Cause of death: respiratory failure
e. Treatment: Stomach wash, demulscent, calcium disodium
versanate
f. post mortem appearence: Stomach mucosa shows tiny granular
appearence. Sticking on the crypts with garlicky order. All
other organs congested. Blood cherry red

2. Methylated spirit
a. F.D: 60-200 ml, F.P: 24-36 hrs
b. C/F: headache, dizziness, visual disturbances, metabolic acidosis- CNS
depressant
c. C.O.D: acidosis and respiratory failure
d, Treatment: stomach wash with sodium bicarbonate, correction of
metabolic acidosis, ethanol 50% 1 ml/kg body wt/2 hrs for 5
days.
e, PMA: congestion, h’age in the GIT, tubular degeneration in the kidney.

3. Endrin (plant penicillin)-dieldrin


a. Chemical nature : organochloro compound –broad spectrum
action 190
GOVT MEDICAL COLLEGE, TRIVANDRUM
LUMINAIRE-2015

b. FD: 3-6gm, FP- l/2-6hrs.


c. MOA: considered as a neuro toxin ;z
d. CF: dyspnoea, dilated pupil, tremor, ataxia, convulsions, failure of
respiratory centres.
e. Treatment: i.v Ca gluconate or oral Ca lactate. Cholestyramine-incr
excretn
f. PMA: kerosine like odour of stomach contents.

4. Tik-20- organophosphate
a. Chemical nature: diazinone
b. Organophoshorus compound.
c. FD: l gm oral, FP : within 24 hrs
d. MOA: inhibits the enzyme choline esterase, results in the
accumulation of Ach.
e. CF:
i) Muscarine like effect: Salivation, Lacrimation, Urination,
Defecation, G.I disturbance, emesis.[ code- SLUDGE]
ii) Nicotinic manifestation: muscle twitching, fasciculations,
weakness, paralysis.
iii) CNS effects: headache, giddiness, drowsiness. COD:
respiratory failure
f. Treatment: atropinisation, oximes(learn doses).
g. PMA: kerosine like smell in stomach, froth in air passages,
cyanosis

5. Oxalicacid (salt of sorrel)


a. FD 15-20 mg FP: 1-2 hrs
b. CF: nausea, vomiting, tenesmus, shock, hypocalcemia, hematuria
due to renal tubular necrosis, vomit-coffee colored
c. Treatment: Specific Ca preparation-
1. Calcium disodium versanate 2. Ca gluconate i.V.,
3. Saccharated soln of lime 4. I.v fluids, 5. Dialysis
d. PMA: corrosion of upper GIT, white shrivelled mucosa of GIT, dark
brown streaks of blood on stomach
e. MLA: accidental, suicidal poisoning

6. Gardenal sodium
a. FD 3-4 gm orally, FP: 1 -2 days.’
b. CF: slow respiration, anoxic blebs, coma, shock, drowsiness. 191
GOVT MEDICAL COLLEGE, TRIVANDRUM
LUMINAIRE-2015

c. MOA: CNS depressant


d. COD: respiratory failure, irreversible cereberal anoxia,
bronchopneumonia.
e. Treatment: stomach wash, forced alkaline diuresis, dialysis,
symptomatic treatment.
f. PMA: particles of tablet in stomach, general visceral congestion,
softening of basal ganglia.
g. MLI: suicidal, accidental, automatism, homicidal.

7. Charcoal powder
a. MO A: adsorbs within its pores poisons delaying systemic
absorption.
b. Universal antidote- animal charcoal : mgo : tannic acid -2:1:1. Two
table spoons of this is taken in a glass of water and glucose.
c. Animal charcoal adsorbs alkaloids, mgo-acids, tannic acid ppts
metals, alkaloids.

8. Stomach tube(Ewald/ Boa’s tube)


a. For gastric lavage. 1and half meters long.
b. Contraindications: Corrosive poisoning (except phenol),convulsant
poisoning, oesophageal varices, hypothermia, volatile poisons.

10. Oxime
a. Pralidoxime dephosphrylates the inactivated choline esterases
b, Dose 1 gm PAM is given i.v as 5% soln in water over 5 min. Repeat
4 times every 8 hrs.

11. Carbolic acid / phenol (black liquid with phenolic smell)


a. FD – l0 - 15gm, Fp: 3-4hrs
b. Locally acts a corrosive later anaesthetic effect, systemically act
as a narcotic.
c. CF: a/c poisoning is known as carbolism. Burning, numbness,
anaesthesia at the area of contact, smell of phenol in breath, pupils
constricted, shallow and rapid respiration – oochronosis(blackening)
Urine: hematuria, albuminuria, carboluria [urine turns to olive green due to
oxidation of metabolic products-Pyrrocatecholes and hydroquinones.
d. COD: respiratory failure, cardiac failure.
e. Treatment: stomach wash with mgs04, demulscents, liquid
paraffin, i.v fluids. 192
GOVT MEDICAL COLLEGE, TRIVANDRUM
LUMINAIRE-2015

f. PMA: stomach- ash grey in colour, hard, leathery, velvety contents


having smell of phenol.
g. MLI: suicidal, accidental poisoning, abortifacient.

12. Formic acid


a. Black corroded thickened rugae and mucosa
b. FD 50-200 ml
c. CF- burning pain, vomiting, altered blood, oliguria, hematuria
d. Treatment: intragastric milk drip, careful aspiration of stomach
contents, symptomatic treatment- Folinic Acid(1mg/kg)
e. Cause of death: acidosis, respiratory failure
f. PMA: corrosion of GIT, blatubular necrosis of kidney
g. MLI: suicidal, accidental.

13. Sulphuric acid


a. FD: 10 - 15ml, FP: 12-24hrs
b. CF: burning, vomiting, difficulty in breathing and swallowing
c. Treatment: morphine for pain, antidote is cao, mgo, i.v fluids, milk
of magnesia, Aluminium hydroxide gel
d. Contraindications: gastric lavage, bicarbonates by mouth
e. COD: circulatory collapse, spasm and oedema of glottis,
perforation of stomach, toxemia
f. PMA: Blackish corrosion of upper GIT, chalky white teeth,
perforation of stomach, corrosion of abdominal viscera.
g. MLI: vitriolage, grevious hurt, accidental or suicidal poisoning.

14. Nitric acid


a. Nature: corrosive mineral acid
b. FD: 10-15 ml, Fp: 12-24hrs
c. CF: burning pain, suffocation, vomiting [yellowish]
d. Treatment: antidote-Ca/mgo.
e. Contraindication: gastric lavage, bicarbonate by mouth.
f. COD: spasm and oedema of glottis, circulatory collapse, remote:
starvation due to stricture.
g. PMA: yellowish corrosion of GIT due to xanthoproteic reaction,
inflammation of respiratry tract, peeling of mucosa of GIT.
h. MLI: vitriolage, grevious hurt, accidental or suicidal poisoning.

15. HCL 193


GOVT MEDICAL COLLEGE, TRIVANDRUM
LUMINAIRE-2015

a. Nature of poison: corrosive mineral acid


b. FD: 15-20 ml, FP: 12-24 hrs.
c. CF: burning, pain, suffocation, vomiting
d. COD: suffocation, shock. Remote cause: bronchopneumonia,
stricture formation
e. Treatment: antidote: Ca/mgo, morphine, i.v fluids, demulscents
f. PMA: corrosion of mucosa with grey colour and blackish streaks of
blood vessels, oedema of glottis present.
g. MLI: vitriolage, accidental and suicidal poisoning.

16. Hammer
a. Dangerous weapon-any instrument for shooting, stabbing, cutting
or any instrument which is used as a weapon of offence and is likely to
cause death
b. Produces blunt injury- heavy weapon
c. MLI- weapon can be identified- imprint injury
d. Types of injuries produced: contusion, laceration, abraded
contusion, depressed fracture [fractured bone is driven into the skull
cavity. Also called as signature fracture because shape of fracture
resembles shape of weapon.

17. Lathi
a. Dangerous weapon
b. Tramline contusion
c. Types of injuries produced: contusion [a patterned contusion
/bruise is produced {2 lines of contusion seperatedby normal skin}],
laceration, abraded contusion, comminuted fracture or depressed
localized fracture.

18. Stone
a. Dangerous weapon, Produces blunt injury
b. Can produce depressed fracture in skull.
c. Types of injuries produced: contusion, laceration, abraded
contusion, depressed fracture in skull [may be irregular or triangular]

19. Cutting sharp weapon


a. Dangerous weapon
b. Injuries produced: abrasion, punctured wound, penetrating wound,
chop wound- depth of injury more, deeper sructures are involved. 194
GOVT MEDICAL COLLEGE, TRIVANDRUM
LUMINAIRE-2015

20. Rubber tappers knife- wound shape- boat shaped


a. Dangerous weapon
b. Square bracket shaped( [ ) wound is produced.

21. Rope
a. Dangerous weapon
b. Used for: hanging and strangulation

22. Single edged light cutting sharp weapon


a. Dangerous weapon
b. Injuries produced: abrasion, incised wound, stab
wound-perforating and penetrating.
Perforating:- through and through injury
Penetrating:- enters into the body cavity
c. Produces a triangular or wedge shaped surface wound. Blunt end
may show fish tailing of wounds (small splits in skin)

23. Double edged light weapon


a. Injuries produced: abrasion, incised wouind, stab wound [depth
greater than legth and breadth]
b. Shape of the wound elliptical or slit like and both angles are sharp
or pointed.

24. Extradural blood clot- least common(3%)


a. Cause: blow over the lateral convexity of head injury the middle
meningeal artery, vein and less commonly posterior meningeal artery
and anterior ethmoid aretery.
b. Blow on vertex- extradural heamorrhage from sagittal sinus.
c. Symptoms: immediate but temporary loss of memory/
consciousness, return of consciousness after a period of hours to
weeks- lucid interval. Death occurs due to respiratory failure due to
brain stem compression.
d. MLI- lucid interval.

25. Subdural clot


a. Causes rupture of bridging parasagittal veins, dural venous
sinuses, aneurysm, injury to cortical veins.
b. Drugs: dicoumarol, heparin, warfarin. 195
GOVT MEDICAL COLLEGE, TRIVANDRUM
LUMINAIRE-2015

c. Types: acute, sub acute, chronic


d. Death due to secondary pressure on brain stem.
e, MLI: mistaken for schizophrenism in young and senile dementia in
the elderly.

26. Subarachnoid h’age(diffuse hmg- ID)


a. Usually starts at the base of the brain. Never localized as easily
mixes up with the CDF, diffuses and spreads over the entire surface of
the brain.
b. Causes: rupture of aneurysm is circle of Willis; Traumatic usually
follows rupture of vertebral atery following a blow on the side, neck,
below the ear-Commando chop. BALA
B- Berry aneurysm rupture
A- Angioma, AV malformation
L- Leaukemia, blood dyscrasia
A-Asphyxia
c. Symptoms – headache, stiff neck, photophobia, deterioration of
consciousness.

27. Intracerebral / brainstem h’age


a. Causes:
1. Bleeding from lenticulostriate artery near basal ganglia.
2. Hypertension and vascular disease
3. Angioma
4. Malignant tum ors of brain.
b. Symptoms- Unconsciousness from the moment of injury.

28. Specimen of heart and lung


a. Rt ventricle and pulmonary artery seen opened with thrombus in
situ.
b. Causes:
1. Emboli from lower legs in bed ndden patients.
2. Varicosities of superficial veins of leg
3. Hypertension, diabetes mellitus, atherosclerosis
c. MLI: sudden natural death.

29. Cross section of kidney with Bilateral adrenal h’age


a. Causes : meningococcal septicemia, bleeding diathesis,
hypertension, shock. 196
GOVT MEDICAL COLLEGE, TRIVANDRUM
LUMINAIRE-2015

b. MLI: sudden natural death. Waterhouse-frederichsen syndrome

30. Sternum showing incised penetrating wound (cut fracture sternum)


a. One end of the wound is cleanly cut and the other end is split. The
weapon used is single edged.
b. COD: injury to the vital organs.

31. Piece of skin showing an injury with a pale base


a. Electric burn mark [joule burn]- diagnostic of contact with
electricity. -
Found at the point of entry of current an oval shallow crater 1 -3
cms in diameter with a ridge of skin around the circumference, there is
conversion of electricity into heat energy.
b. COD: paralysis of medullary respiratory centre, ventricular failure,
cardiac arrest.
c. MLI: accidental- done by convulsive therapy, rarely homicidal,
judicial execution in the U.S.

32. Crocodile skin


a. Piece of skin showing multiple burnt/punched out lesions- flash
burns,arc eye
b. Multiple burned out or punched out lesions, due to the electric arc
dancing over the body surface.
c. COD: paralysis of medullary centre, ventricular fibrillation

33. Depression fracture skull


a. Portion of skull and brain showing a depression of skull with
corresponding depression of brain.
b. MLI: signature fracture [resembles shape of weapon].

34. Stab injury head involving skull, dura and brain


a. Penetrating injury.

35. Abortion stick


a. Abortion stick in-situ with perforated fundus of the uterus and wall
of large intestine.
b. Inference: criminal abortion using abortion stick.

36. Uterus with appendage- 197


GOVT MEDICAL COLLEGE, TRIVANDRUM
LUMINAIRE-2015

a. wet specimen showing ruptured tubal pregnancy


Right ovary is covered with ampulla; portion of fallopian tube
where an embryo developed.
b. Causes: IUCD, tube pathology
c. MLI-sudden natural death.

36. Wet specimen of cervix showing cervical os


a. Multipara(transverse)
b. Nullipara(round)
c. Diff b/n multipara, nullipara

37. Wet specimen of uterus with coppert insitu


a. MTP act, abortion and delivery

38. Laceration of liver


a. Cause: blunt force like kicking, fall from height
b. COD: bleeding, shock

39. Specimen of oesphagus showing ruptured oesophageal varices


a. Causes:
1. Increased portal venous presssure
2. Portal vein thrombosis
3. Hepatic vein thrombosis
4. Pyelophlebitis
b. CF: hematemesis, anemia.
.c. MLI: sudden natural death.

40. Degloved skin- preserved in 10% formalin


a. Skin of hand is kept
b. Causes: burns, drowning, putrefaction.
c. MLI: fingerprints

41. Hyoid bone adduction fracture (inward displacement of broken bone)


a. Cause: manual stangulation [throttlirig]
b. PMA: fingernail marks, bruise and contusion, diffuse extravasation
of blood, thyroid and cricoid fracture, asphyxial signs.

42. Piece of skin showing patterned abrasion of a coir rope.


a. Features of pressure abrasion seen; typical hanging, ligature 198
GOVT MEDICAL COLLEGE, TRIVANDRUM
LUMINAIRE-2015

strangulation
b. COD: asphyxia, venous congestion, combined asphyxia and venous
congestion.

43. Piece of skin showing abraded contusion


a. MLI: 1. Evidence of aplication of blunt force.
2. Degree of violence may be determined
3. Age of injury from colour changes
44. Tatoo marks
a. wet specimen of forearm showing tattoo marks on ant aspect
b. Dyes used: indigo, cobalt, cinnabar, carbon, vermilion
c. Erasure
l. Surgical methods
2. Electrolysis
3. Caustic substances
4. Laser beam
d. MLI: identity, religion, God of worship, social status, drug addicts,
any sexual perversions.

45. Entrance and exit wounds

Entrance wound Exit wound

Size Smaller than diameter of the Diameter bigger than bullet


bullet

Edges Inverted Everted

Grease collar Present Absent

Burning / Tattooing May be seen around the wound Absent

Bleeding Less More

Tissues Cherry red Absent

46. Malunion fracture


a. MLI : Identification, to sue for damage –medical negligence,
grevious hurt.
199
GOVT MEDICAL COLLEGE, TRIVANDRUM
LUMINAIRE-2015

47. Foetal Sternum


a. Five osscification centres appear in the following order:
1 st- 6 months
2nd-6 months
3rd-7 months (viable)
4th-7 months
5th- 9 months.
Here 5th ossification centre appeared - So age of the foetus is 9 months

48. Infant sacrum


a. 5 centres appear in the following order:
1st-4 to 5 months
2nd - 4 to 5 months
3rd - 6 to 7 months
4th - 6 to 7 months
5th - 8 months.
If age is 8 months then MLI: viable foetus.

49. Scrotum of fetus with testis


a. Scrotum showing descended testis.
b. Descends to scrotum at 9th month of IUL
c. MLI: full term

Plants

50. Cannabis sativa


a. Toxic principle- tetrahydrocannabinol, cannabinol, cannabidiol,
cannabinic acid
b. FD- l-2 mgof THC
c. Preparation- bhang,majoon, ganja, charas.
d. C/F- Stage of inebriation- euphoria, increased appetite, loss of
perception of time & space.
e. Treatment- Stomach wash, symptomatic
f. MLI- Run amok, drug dependence, accidental poison, stupefying
agent

51. Abrus precatorius


a. Identification: small, round smooth bright scarlet colour seeds with
a large black spot at one end. 200
GOVT MEDICAL COLLEGE, TRIVANDRUM
LUMINAIRE-2015

b. Nature of poison: vegetable irritant, locally- inflammation,


vessication, ulceration, orally- GI irritation
c. Active principles: abrin [toxalbumin], abrine, abralin [glycoside]
d. MOD: Abrin inhibits protein synthesis & cause cell death. .
e. FD-90-120 mg by inj, 10mg orally
f. FP- 3-5 days
g. MLI- 1 .cattle poison in the form of suis
2. Accidental poison
3. Abortifacient
h. Specific antidote-antiabrin

52. Nerium odorum


a. MOA: cardiac poison
b. Toxic principle: nerin, oleandrin
c. FD-15-20 gm of the root,5-l 5 leaves
d. FP- 20-36hrs
e. C/F - slow & weak pulse, muscular twitchings, coma, difficulty in
swallowing, dilated pupils, signs of cardiac failure.
f. MLI- suicidal poison, abortifacient, cattle poison

53. Cerbera odallam


a. Poisonous part- kernel of fruit
b. Active principles- cerberin, cerberoside, odollin, odollotoxin,
thevetin, cerapain FD- kernel of one fruit
c. FP - 1-2 days
d. C/F-bradycardia, cardiac arrthymia, conduction defects,
hyperkalemia
e. Treatment- stomach wash, atropine 0.5 mg iv, correct hyperkalemia
f. PMA- petechial h’ age in the heart, congestion of internal organs
g. MLI-suicidal, accidental in children.

54. Cerbera thevetia


a. Active principles: thevetin, thevatoxin, cerberin, peruvoside,
nerifolin.
b. FD: 15-20gm root -
c. FP: 2-3hrs.
d. CF: rapid weak and irregular pulse, tingling and numbness of
tongue, loss of muscular power and death from peripheral
circulatory failure. 201
GOVT MEDICAL COLLEGE, TRIVANDRUM
LUMINAIRE-2015

e. Treatment: BHIST regime: sodium molar lactate transfusion with


glucose and 1 mg atropine, 2 ml adrena­line and 2 mg noradrenaline.
f. MLI: suicidal, abortifacient, cattle poison.

55. Calotropis
a. Nature of poison: vegetable irritant.
b. Locally produces inflammation, vesication, ulceration, orally
produces GI irritation and CNS irritation.
c. Active principles: gigantin, urcharin, calotoxin, calactin, calotropin.
d. MLI:abortifacient, cattlepoison, infanticide, fabricating injuries,
vitriolage.

56. Ricinus communis


a. Nature of poison: vegetable irritant.
b. Locally produces inflammation, vescication, ulceration, orally
produces GI irritation.
c. MOA- ricin blocks protein synth by inhibiting RNA polymerase
d. Active principle is a toxalbumen known as Ricin. Cake obtained
after oil extraction contains more of ricin
e. FD: 10 castor seeds
f. FP: 7 days
g. MLI: accidental poisoning in children. Large doses of castor oil
may prove fatal in children.

57. Semecarpus Anacardium [marking nut]


a. Nature of poison: vegetable irritant.
b. Locally produces inflammation, vescication, ulceration, orally
produces GI irritation.
c. Active principles: semecarpol, Bhilawanol.
d. FD: 5-10gm
e. FP: 24-48 hrs
f. MLI: 1. To fabricate an injury
2. Throwing juice to produce disfiguration of face[grevious hurt]
3. Abortifacient

58. Datura
a. Active principles- Hyoscine, hyoscyamine, Atropine
b. Action- first stim higher centres of brain, motor cntr, finally-depression,
paralysis 202
GOVT MEDICAL COLLEGE, TRIVANDRUM
LUMINAIRE-2015

c. C/F- corn picker’s pupil(unilat mydriasis), (8Ds-dryness of mouth,


dysphagia, dil pupil, dry hot skin,drunken gait, delirium, drowsiness,
death due to resp failure) -Mydriatic test
d. Circumstances – Road poison, abortifacient, aphrodisiac, accidnt cause
e. MLI person suff frm delirium due to pois- not criminal responsible

59. Strychnine- Nux vomica


a. Active principles- strychnine, brucine, loganin
b. Action- prevents the eff of glycine(inhib transmitter)- release excitation
Read Diff b/n strychn poisng and tetanus
c. C/F- convulsion, risus sardonicus, ophisthotonus

Snakes

60. Sea snake


a. Identifying features: small eyes, prominent nostrils on the top of
head, black in colour, compressed paddled tail, small tuberculated
dorsal scales, broad ventrals.
b. Nature of venom: myotoxin.
c. Signs and symptoms: muscle pain, stiffness, muscle weakness,
ptosis, visual disturbances, urine may contain albumin,
erythrocytes and myoglobin.
d. COD: acute renal failure due to myoglobinuria, respiratory failure.

61. Russel’s viper


a. Identifying features: flat heavy traingular head-with V shaped mark,
angle of the V pointing forwards.It has three rows of diamond
shaped black or brown spots along the back. Viviparous
b. Nature of venom: hemotoxic
c. Signs and symptoms:
d. Local symptoms: severe pain, ecchymosis, swelling, blood stained
discharge.
e. Systemic symptoms: blurring of vision, signs of collapse, h’age,
renal failure
f. COD: shock and h’age.
g. FD: 15 mg, FP: 1-2 days.
h. Treatment: RIGHT(Reassure, Immobilise, Get to Hospital, T
reatment)
1. First aid 203
GOVT MEDICAL COLLEGE, TRIVANDRUM
LUMINAIRE-2015

2. Specific: polyvalent antisnake venom serum 20 ml i.v

62. Cobra
a. Identification: dorsal side of head bears spectacle like mark, tail
shields are divided. Third labial touches the eye. Head scales large.
-oviparous
b. Nature of venom: neurotoxic
c. Affects respiratory and cardiac centres.
d. Treatment: Refer Russel’s viper treatment.

Microscopy

63. Human hair and animal hair

Human hair Animal hair

General fine and thin Coarse and


thick

Cortex Thick Thin

Medulla usually narrow Wider

MLI: 1. Differentiate b/w hair and fibre


2. Crime, investigations
A. RTA [hair root-blood group]
B. Rape, bestiality, sodomy
C. Stains-nature of assaults
3. Identification- age, sex
4. Texture of hair indicates from which part of body it is from
5. Chronic metal poisoning-arsenic

64. Intracerebral h’age, subarachanoid h’age[for questions refer wet


specimens]

65. Diatoms
a. Microscopic unicellular or colonial aquatic algae.
b. Appearence: spindle shaped, colourless, glassy/silicacious
capsule 204
GOVT MEDICAL COLLEGE, TRIVANDRUM
LUMINAIRE-2015

c. MLI: drowning

66. Unrespired lung( ID-glandular lobulated app)


a. Alveolar sacs are closed and lined with columnar cells
b. MLI: not live birth, indicates still birth or dead birth.

67. Respired lung (ID- alveoli with a lot of air spaces, cartilage can be seen)
a. Alveolar cells are dilated and lined with flat cells, vascularity
prominent
b. MLI: live birth

68. Chorionic villi


a. MLI: sure sign of pregnancy.

69. Alcohol content of beverages


Beer – 4-8%
Toddy – 2-5%
Wine – 10-15%
Sherry – 20%
Arrack/Brandy – 40-50%
Rum/Whisky/Gin – 50-60%
70. Ossification of bones
DATES FOLLOWED IN OUR STATE MEDICOLEGAL INSTITUTE
a. Eruption of permenant tooth
b. Ist molar 6-7yrs
c. Medial incisor 7-8yrs
d. Lateral incisor 8-9yrs
e. Canine 11-12yrs
f. I premolar 9-11 yrs
g. II premolar 10-12 yrs
h. II molar 12-14 yrs
i. III molar 17 or above or never

71. Appearance and Ossification of ossification centers

Bone Appearance Fusion

Acromion 14 18
205
GOVT MEDICAL COLLEGE, TRIVANDRUM
LUMINAIRE-2015

Coracoid process 14 18

Head of humerus 1

Lesser tuberosity 5 Composite 16-18


mass-6

Greater tuberosity 3

Medial epicondyle 8 14-17

Capitulum 2

Trochlea 11- compact mass- 14-17


14

Lateral epicondyle 13

Olecranon 10 14-17

Head of radius 6 14-17

Lower end of radius 2 14-18

Lower end of ulna 6 14-18

Base of 1st 2-4 14-17


metacarpal

Head of other MC’s “ “

Heaad of femur 1 15

Lesser trochanter 11 14-17

Greater trochanter 4 14-17

Iliac crest 14-16 18

Ischial tuberosity 16 19 (F) 21(M)

Ischiopubic ramus - 7-8

Triradiate cartilage - 11-14

72. Fetal foot


Find the time of appearance of ossification centres, diagram of bones 206
GOVT MEDICAL COLLEGE, TRIVANDRUM
LUMINAIRE-2015

forming fetal foot, calcaneum 5th month, talus-7, cuboid-9

73. Wet specimen of transection aorta- ulcerated and calcified


a. Atheromatous plaque- rupture, thrombosis.

74. Appearance of carpal bones


Capitate – 1yr
Hamate – 2yrs
Triquetral – 3 yrs
Lunate – 4 yrs
Trapezoid, scaphoid – 5 yrs
Trapezium – 6 yrs
Pisiform – 11 – to 12 yrs

75. Speccimen of stomach in various poisoning


CuSO4, Phenol, Fomic acid, Nitric acid, HCl
Analysis of postmortem certificate will be one of the compulsory exercise
for practical examination. One should know the objective of doing a medico- legal
autopsy for proper analysis of postmortem certificate
1.To find out the cause of death.
2.To find out the time since death
3.To get some clues regarding the manner of death
4.To establish the identity of unknown persons
5.In foetal autopsies to find out whether it was live born, still born or dead
born and to find out its intrauterine age.
An actual postmortem certificate will contain 4 paragraphs and an
opinion as to the cause of death. The first paragraph will be preamble. In the
certificate given for examination there will not be any preamble, instead you will be
given a history of the case. The history may contain relevant and irrelevant details.
Questions can be asked from history part also.
The second paragraph will be about the external appearances of the body
like height, weight, scale of body orifices, colour of nails, blood stains, salivary stains
etc.
The third paragraph will be about postmortem changes like rigor mortis,
postmortem staining, signs of decomposition if any, whether the body was refrigerated
etc.
Then there will be a list of injuries if any in the body.
The last paragraph will be about other positive findings and relevant
negative findings. The state of internal organs are described in this paragraph. There 207
GOVT MEDICAL COLLEGE, TRIVANDRUM
LUMINAIRE-2015

will not be any opinion as to the cause of death in the certificates given to you. Forming
and wording the opinion will be one of the questions.
Forming and wording the cause of death
While wording the opinion as to the cause of death you should remember
that the postmortem certificates are sent to the concerned magistrates and police
officers, who are laymen as far as we medical people are concerned. So wording
should be in such a way that they can understand it. In all cases laymen’s language
alone may not be sufficient to express the opinion. Here you can add an explanatory
sentence or word. We need not mention the exact mode of death (and it is not always
possible also) since the law enforcing authorities are not bothered about it. For
example in a case of hanging the mode of death may be asphyxia. Here we need
mention only whether the death was due to hanging or not.

Opnion in injury cases


Read all the antemortem injuries carefully and find out the fatal nature of
the injuries. The fatal injuries are classified into 3
(a) Necessarily fatal
(b)One which is sufficient in the ordinary course of nature to cause death
(c) One which is likely to cause death.
A necessarily fatal injury is one which is incompatible with life. Examples
are decapitation, transection of the trunk, avulsion of the heart, mangling of the body
etc. There is not even 1 % of chance of survival after sustaining such injuries
After sustaining an injury, if the victim is left to the ordinary course of
nature (without treatment), if death is most probable, it is called an injury which is
sufficient in the ordinary course of nature to cause death. All penetrating injuries are
considered as sufficient inthe ordinary course of nature to cause death. Ligature marks
the neck in hanging and strangulation are other samples. Fracture skull, with or without
brain damage is sufficient in the ordinary course of nature to cause death. Brain
damage alone also is sufficient. Multiple rib fracture is another example, Fracture of
femur, pelvis, spine etc. are sufficient in the ordinary cours of nature to cause death. In
any injury if a major blood vessel is involved it is sufficient in the ordinary course of
nature to cause death. Rapid loss of 1/3 of total blood volume is sufficient in the
ordinary course of nature to cause death. Burns affecting more than 1/3rd \ of the
body surface is another example
If death is the probable result of an injury (left untreated) it is an injury
which is likely to cause death. Examples are fracture of long bones other than femur,
burns affecting less than 1/3rd of the body surface etc.
While forming the opinion as to the cause of death if there is only fatal
injury, you can state that as the cause of death. If there is a penetrating injury to the 208
GOVT MEDICAL COLLEGE, TRIVANDRUM
LUMINAIRE-2015

chest, others being nonfatal you can word the opinion that “Death was due to the
penetrating injury sustained to the chest”. If there is a fatal head injury also the wording
shall be “Death was due to the injuries sustained to the the head and chest.. If there is a
fatal injury on the abdomen also then you may word your opinion as “Death was due to
the injuries sustained to the head, chest and abdomen. The terms bleeding, shock etc.
shall not be incorporated in the opinion. Where there are more than 3 fatal injuries you
can word the opinion as “Death was due to multiple injuries sustained”. In deciding
whether a culpable homicide is murder or not the fatal nature of the injury inflicted is
very important.
Forming Opnions in poisoning cases

In suspected poisoning, if the stomach contents has an unusual smell or


colour the opinion shall be “Postmortem findings are consistent with death due to
poisoning, Final opinion is reserved pending the report of chemical analysis.” If there is
no obvious cause of death the opinion will be “Reserved pending the report of chemical
analysis”
Cases of Burns
Presence of soot in the air passages, red line of demarcation between the
affected and nonaffected area, blisters containing serum rich in protein and choride
etc. are the antemortem features of burns. If the bums are antemortem the opinion as
to the cause of death shall be “Death was due to dermo-epidermal burns affecting it,
scale of about 70% of body surface” or “Death was due to infection following second
degree burns affecting about 30% of body surface” or “Death was due to the
complications of dermo-epidermal burns affecting about 20% of body surface”
depending upon the findings furnished in the P.M.C.
Hanging cases
Vertical salivary dribble mark and ‘Le facie sympathique; are the
conclusive evidence of antemortem hanging.If any one or both signs are present you
should word the opinion as “Death was due to hanging”. Never add asphyxia due to
hanging etc. If there is a noncontinuous oblique pressure abrasion on the neck and
underlying pale and dry subcutaneous tissue along with other signs of suspension, in
the absence of other causes of death, you can word the opinion as “Postmortem
findings are consistent with death due to hanging” even if conclusive signs are absent.
Drowning cases
Presence of sand, mud, leaves etc. in the air passages is a conclusive
evidence of drowning. If identical diatoms are detected in bone marrow of the
deceased and in the drowning medium it is another evidence of drowning. Some
experts consider fine white lathery tenacious consistent mushroom shaped froth at
nostrils together with emphysema aquosum as a conclusive evidence of drowning. So209
GOVT MEDICAL COLLEGE, TRIVANDRUM
LUMINAIRE-2015

if any of the above findings are present in the body the wording of the opinion shall be “
Death was due to drowning”. If the signs of submersion of the body in water like
washer woman’s hand, cutis anserina etc. are present and the conclusive proofs of
drowning are absent, in the absence of other causes of death you can furnish the
opinion as “Postmortem findings are consistent with death due to drowning”. In a case
of drowning if there is a fatal head injury also opinion shall be “Death was due to the
combined affects of drowning and head injury”. If a body is recovered from water in a
decomposed state and if diatom test is negative the opinion as to the cause of death
shall be reserved pending the report of chemical analysis though presence of sand etc.
in the air passages is indicative of drowning.
Electrocution
If there is a joule burn on the body in the absence of other causes of
death you can give an opinion “Death was due to electrocution”.
Foetal autopsy
In foetal autopsy one has to find out whether it was live born, dead born or
still born. If it was live cause of death is to be determined. Apart from these the
intrauterine age of the foetus also has to be found
Even without an autopsy we can certify live birth if there is physiological
jaundice, erythema around umbilicus etc. Presence of milk in the stomach, air in the
G.I.T. and the middle ear etc. are other conclusive evidence of live birth. In the absence
of the above Breslau’s 2nd Life test has to be done to ascertain livebirth. Microscopy of
lung is almost confirmatory.
In a live born foetus presence of Circumoral pallor is an evidence of
smothering. By applying Haase’s rule the age of a foetus can be determined from it
length. This is only an estimate. Conclusive proof is appearance of ossification centres
in sternum, ankle and sacrum. Some opinions given after foetal autopsy are:
The foetus was viable and live born
Death was due to smothering.
The foetus was not viable.
The foetus was dead -born (only when signs of maceration or
putrefaction is present).
The foetus was not live born (It is difficult to give an opinion that a foetus
was still born).By naked eye examination it is not possible to
determine the sex of a foetus if it is less than 4 months

Sudden deaths
Death occurring suddenly and unexpectedly in an apparently healthy
individual under suspicious circumstances due to natural causes are called sudden
deaths. The most common cause of sudden death is myocardial infarction. Some 210
GOVT MEDICAL COLLEGE, TRIVANDRUM
LUMINAIRE-2015

sample opinions in sudden death is given below.


• Death was due to coronary thrombosis - a disease of the heart (Natural
cause).
• Death was due to pneumonia - a disease of the lung (Natural cause).
• Death was due to subarachnoid bleeding - a disease of the brain (Natural
death).
• Death was due to advanced tuberculosis of the lungs.

Estimation of time since death


• Time since death is determined from postmortem changes like rigor
mortis, postmortem staining, of decomposition etc.
• Rigor mortis will appear in head and neck by 1 -2 hours, in the upper limbs
by 2-4 hours and in the lower limbs by 6 hours. The whole body will
become stiff due to rigor mortis by 6 hours. Rigor mortis will start
passing off by 18 hours. Complete disappearance is by 48 to 72
hours.
• Postmortem staining will get fixed by 6 to 12 hours. If postmortem
staining is fixed time since death is more than 6 hours. If not fixed
then less than 12 hours (exception being asphyxia).
• Greenish discolouration will appear in right iliac fossa by 18 - 24 hours.
Marbling will be seen by 36 .hours.
• If rigor mortis is fully present all over the body the minimum time since
death is 6 hours. If it has started passing off 18 hours elapsed after
death. Temperature, muscle mass exertion prior to death etc. are the
conditions which can modify the onset and duration of rigor mortis.
• If rigor mortis is present all over the body and postmortem staining is not
fixed, the time since death is more than 6 hours and less than 12
hours.
• If rigor mortis is retained all over and postmortem staining is fixed time
since death is more than 6 hours and less than 18 hours.
• If rigor mortis started passing off and greenish discolouration is present
in right iliac fossa time since death is more than at least 18 hours and less than
36 hours, (since marbling the next postmortem change has not appeared).
Time of death from stomach contents
• The gastric emptying time is 4-6 hours. If the stomach is empty the time
of death is at least 4 hours after the last solid meal. If the stomach
contains food time of death is less than 6 hours after the last solid
meal. Hence if the time of last meal is known, the time of death can be
determined. 211
GOVT MEDICAL COLLEGE, TRIVANDRUM
LUMINAIRE-2015

• Stomach emptying time maybe delayed in head injury, shock etc.


The rate of formation of urine in the bladder is 1 ml/mt. This will help to
fix the time of death to a certain extent only.
Manner of death
Manner of death means whether a death is accidental, suicidal or homicidal. It is
not always possible to state the manner of death. In certain cases from the history and
postmortem findings some clues regarding the manner of death can be obtained. A
chop wound on the top of head and a stab injury in an inaccessible area is strongly in
favour of homicide. A firearm injury from a distant range cannot be suicidal. In a case
of hanging if poison also is found in the stomach it is in favour of a suicide. A body
recovered from a well with “brush burns” on the palms is in favour of accidental fall.
Applicatiin of Locard’s principle of exchange may help in fixing the manner of death in
hanging. Examination of finger prints in weapons also will help to get some clues
regarding the manner of death.
Identity
• In unknown bodies, its identity has to be established.
• Sex determination will not be a problem unless the body is highly
decomposed. Prostate and nongravid uterus resist putrefaction for
a long time.
• Age can be determined from obliteration of skull sutures, fusion of
manubrium and xiphoid process with the body of sternum etc. Age
can be determined from a single tooth by Gustafsson technique.
• Pierced ear lobule in a male indicates that the victim is a Hindu.
Circumscised penis is in favour of the deceased being a Muslim.
• See whether there are callosities or other occupational mark.
• Dental formula of unknown bodies are expressed in modified F.D.I.
(Federation Dentaire International) Code

Conclusion
What is given above is not an exclusive note. To answer the questions given
along with the P.M.C. a thorough knowledge of the subject is necessary.

Time since death

Rigor mortis

Appearance on the face 1 to 2 hours

Upper limb 2 to 4 hours


212
GOVT MEDICAL COLLEGE, TRIVANDRUM
LUMINAIRE-2015

Whole body stiffness 4 to 6 hours

Starts disapperaing by 18 hours

Completing off 48-72 hours

Postmortem staining (PM Lividity)

Starts appearing 2 to 4 hours

Fixation 6 to 12 hours (if fixed > 6


hours)

Decomposition changes

Greenish discolouration- Rt iliac fossa 24 hours (18-24 hours)

Marbling 36 hours

Gaseous distention, PM blebs & peeling of 48 to 72 hours


cuticle

Loosening of teeth 72 hours

Liqefaction of brain 3 days

Skeletonisation

Exposed body 1 to 2 days

Buried body 2 to 6 days

Adipocere 3 days to 15 days

Mummification 3 weeks to 3 months

Gastric emptying time 4 to 6 hours

Examples of postmortem certificates


The history and Post mortem findings of the case are given below. Read it
carefully and answer the questions given which follow.

CASE NO 1
History: 213
GOVT MEDICAL COLLEGE, TRIVANDRUM
LUMINAIRE-2015

• Body of a female aged about.30 years was found in a rubber estate in a


semi -naked state. The body was identified as that of an agricultural
labourer. Her golden ear rings were found missing from the body.

POST-MORTEM FINDINGS: General


• Blood stained frothy discharge at mouth and nostrils. Multiple small
petechial haemorrhages were present under the conjuctiva, on the eye
lids and on the forehead. Nails bluish. Rigor months established all
over. Post - mortem staining on the back, fixed. Vulva and vagina were
normal, carunculae hymenalis present Injuries (Ante-mortem)
1. Contusion 2 x 1.5 cm on the right side of the front of neck over the thyroid,
infiltration of blood in the subcutaneous tissue underneath.
2. Four contusions each 1.5 x 1 cm one below the other, obliquely placed on the
left side of the front of neck with infiltration of blood in the subcutaneous tissue
underneath.
3. Fracture of the right ala of the thyroid cartilage with infiltration of blood around.
4. Multiple finger nail abrasions on the inner aspects of both thighs.

Otherfindings
• Lungs were congested and oedematous with multiple subpleural
petechial haemorrhages. Heart showed rnultiple sub-epicardial petechial
haemorrhages. Brain was normal. Stomach contained partly
digested food particles without any unusual smell, it’s mucosa was
normal. Liver, Spleen and Kidneys were congested.
• Vaginal smear showed the presence of Spermatazoa.
Questions:
• What is the cause of death ?
Hint: Death was due to throttling.
• What are the various mechanisms of death in such a case ?
Hint: (i) Asphyxia (ii) fracture of hyoid bone (Hi) fracture of thyroid bone.
• What is the manner of death ?
Hint: Homicide
• What is the approximate time since death ?
Hint:>6hrs&<18hrs
• She died within how many hours of her last meal ?
Hint:<6hrs
• In what position was the body lying after death ?
Hint: Supine Position
• What is the significance of injury no. 4 ? 214
GOVT MEDICAL COLLEGE, TRIVANDRUM
LUMINAIRE-2015

Hint: Rape attempt


• Comment on the sexual habit of the deceased.
Hint: Has undergone sexual intercourse - Carunculae hymenalis.
• Can you explain the absence of genital injuries in this case?
• What were the probable motives of the accused in this case ?
Hint: Rape & Theft
• What are the sections of the Indian Penal Code under which the accused
maybe charged ?
Hint: I PC 302 & 376
• What is Florence Reagent.?
Hint: I +KI + H2O

CASE NO .2
• The history and post mortem findings of the case are given below. Read it
carefully and answer and answer the questions given which follow.
History:
• Body of a female, aged about 28 years, was found hanging in a room of
her husband’s house. The doors off the room were found to be
locked from inside. She was staying with her husband and his
relatives after her marriage which took place 3 years ago. There is
history of harrasment by the husband and his mother.

POST MORTEM FINDINGS


Allgemein
• Rigor mortis established and retained all over the body. Post mortem
staining in the lower limbs and fixed
• Conjuctiva and nails were bluish extemal body orfices were nomal vulva
and vagina were nomal salivary dribble marks were found on the right
side of the mouth. A ligature mark made of a saree was found tied
around the neck with a slip knot on the left side of neck head was found
slightly titled to the right side
Injury ante-mortem
• Ligature marks were found on the fornt and sides of the neck above the
level of thyroid cartilage with a discontinuity on the left side of
the back of neck. The subcutaneous tissue undemeath was dry and
pale. Muscles bones and cartilages of the neck were normal and in tact.
No other injury was found on the body
Other findings
• Lungs were congested and oedematous stomach was empty liver spleen 215
GOVT MEDICAL COLLEGE, TRIVANDRUM
LUMINAIRE-2015

and kidneys were congested the uterus was normal it scavity was
empty vaginal smears showed the presence of motile spematazoa
Questions
1 What is the cause of death ?
Hint : death was due to hanging
2 What are the various mechanisms of death in such a case?
Hint : Asphyxia, vagal inhibition, venous congestion ,cervical vertebrae
#,cerebral anaemia
3. What is the manner of death ?
Hint: suicide
4. What is the approximate time since death?
Hint . >6hr & < 18hrs
5. She died within how many hours of her last meal
Hint > 4 hr
6 She died within how many how many hours [maximum after her last
intercourse ]
Hint : 24 <hours
7. There is an allegation that it is a case of post mortem suspension write
three points against this allegation hint
1) room locked in side
2) No external injuries
3) Salivary dribble marks
8. What is the time required for death to take place in a case of hanging?
Hint : 3-5 minutes
9. Hanging is considered as the most painless from of death, Why it is
painless?
Hint Loss of consciousness
10. under what section of IPC, the husband and his parents can be chargted
Hint S 304 B (Dowry Death)
Other Questions
1. Judicial hanging
COD : i) #- dislocation at level of 2nd /3rd or 3rd/ 4th cervical vertebrae
ii) rupture of brain strem (less commonly)
2. Lynching
Homicidal hanging of a suspect, enemy by a rope from a tree by the mob

CASE NO .3
History :
• Body of a 15 year old girl was recovered from a well near her house. Two 216
GOVT MEDICAL COLLEGE, TRIVANDRUM
LUMINAIRE-2015

days agop she had eloped with a man and stayed with him for a day.
When she returened home, she was severely scolded by her parents for
her grave misdeed.
POST-MORTEM FINDINGS:
External:
• Fine white lathery froth at mouth and nostrils. Conjunctiva normal. Nails
were bluish. Palms and soles showed soddening. Rigor mortis
established and retained all over. Post-mortem staining on the
front of chest, fixed. POSTMORTEM FINDINGS
• Hymen showed a recent tear. No other injury was found on the body.

Internal:
• Lungs were ballooned, pale and cut section exuded frothy fluid on
compression. Right lung weighed 35 gms and Left lung weighed 300
gms. Sand particles were present in the bronchioles. Stomach was
empty. Liver, Spleen and Kidneys were congeted. Uterus was
normal in size and it’s cavity was empty. Vaginal smear showed the presence of
spermatazoa.
Questions:
1. What is your opinion as to the cause of death?
Hint: Death was due to drowning.
2 Write 4 points in support of your opinion as to the cause of death.
Hint: i) Fine white leathery froth.
ii) Soddening.
iii) Sand in bronchioles.
iv) Ballooning of lungs
3. What is the manner of death?
Hint: Suicide
4. What is the approximate time since death?
Hint: >6hrs& 18hrs.
5. How long was the body lying submerged in water? (State the minimum
time).
Hint: 24hrs.
6. What is the normal chloride content of blood?
Hint:600 mg/100ml.
7. Which is more dangerous and why? Haemodilution / Hemoconcentration.
Hint: Hemodilution
8. Water may not enter the lungs yet a person may die. How?
Hint: COD: Laryngeal spasm (Dry drowning) 217
GOVT MEDICAL COLLEGE, TRIVANDRUM
LUMINAIRE-2015

9. Which finding show that the deceased was alive at the time of
submersion?
Hint: Sand in bronchioles.
10. What are the offences committed by the man with whom the deceased
eloped?
Hint: Rape, Abduction
11. Under what sections of the Indian Penal Code the man may be charged?
Hint: S.376.IPC, S.362 IPC
12. What is the maximum punishment which can be given to the man if
charges against him are proved?
Hint: Ten years
13. In order to teach the man a lesson, the parents of the deceased falsely
allege that the man had killed his daughter and put her body in the well.
Write 2 points against the allegation of the parents of the deceased.
Hint: i) No external injuries,
ii) post mortem findings of drowning are present
14. Fatal period is less in which of the following -a) Fresh water drowning or
b) Sea water drowning.

Other Questions:
1. Emphysema Aquosum
2. Gettler’sTest
3. Cadaveric Spasm
4. Paltaufs haemorrhages.
5. Washerwoman’s hand.

CASE NO. 4
History:
• A 20 years old unmarried female died in a hospital while under treatment
for poisoning. She was admitted in the hospital 2 days ago,
with a history of taking some poison.
POST-MORTEM FINDINGS:
External:
• Blood stained frothy discharge from mouth and nostrils. Conjunctiva
normal. Nails bluish. Rigor mortis passed off from
head and neck, but retained in the upper and lower limbs. Vulva
and Vagina were normal. Caranculae hymenalis present.
• Injection marks present on the front of right elbow and on the back of
right wrist. No other injury present on the body. 218
GOVT MEDICAL COLLEGE, TRIVANDRUM
LUMINAIRE-2015

Internal:
• Air passages contained blood stained mucous froth. Lungs were
congested and oedematous. Stomach contained 260ml
of greenish fluid with a kerosene like smell, it’s mucosa was
congested. Liver, Spleen and Kidneys were congested. Uterus was enlarged
in size and it’s cavity contained a male foetus. The crown - heal length of
the foetus was 16cms.
Questions:
1. What is your opinion as to the cause of death?
Hint: Death was due to poisoning.Final opinion reserved pending
chemical analysis report
2. What is the manner of death ?
Hint: Suicide
3. What is the time since death?
Hint: 18-24hrs.
4. What findings show that she has had frequent intercouse?
Hint: Caranculae hymenalis.
5. She has been pregnant for how many months?
Hint: 4 months.
6. State whether her pregnancy could have been medically terminated if it
was due to contraceptive failure?
Hint: Yes. (Refer MTF Act)
7. What was the poison consumed by the deceased?
Hint: organophosphates
8. If you are treating such a case, what specimens you will collect from the
patient, while under treatment, for chemical analysis?
Hint: i) Blood
ii) Urine
iii) Aspirated gastric contents.
9. What are the antidotes used for treating such a case? Mention the dose of
each ante-dote.
Hint: i) Atropine Sulphate 2-4 mg i.v.
ii) Oximes - l-2g i.v. as 5% solution
10. While conducting post-mortem examination in a case of poisoning, the
stomach and intestine is collected in one bottle and the liver
and kidney is collected in another bottle for chemical analysis. Why
in 2 separate bottles ?
Hint: To prevent contamination of solid viscera with GIT contents.
11. Formalin should not be used in preserving the viscera. Why ? 219
GOVT MEDICAL COLLEGE, TRIVANDRUM
LUMINAIRE-2015

Hint: Extraction of poison become difficult.


12. What preservative will you use for blood and urine?
Hint: NaF / KF
13. Who will conduct the inquest in such a case?

CASE NO. 5
History:
• A 30 year old male was found dead on a railway track (Meter gauge) with
injuries on the body. After the inquest the body was sent for
post-mortem.

POST-MORTEM APPEARANCES:
• Body was that of an adult male of height 173 cm and weight 60 kg and
having fair complexion. Conjunct were pale. Dry blood stains
were seen in and around the mouth and nostrils. Other body
orifices were normal Nails were blue. Post-mortem skinslip pages
were seen at several sites.
• Rigor mortis was retained uvtReTower limbs only. Post-mortem staining
was not clear. Greenish discoloratioi was noticed in the lower
abdomen. Marbling was seen on the arms. (Body was not
refigerated).
Ante-mortem injuries:
• Incised penetrating wound 5x1.5cm obliquely placed on the left side of
the front of chest with it’s lows border sharply cut and 5 cm
outer to the midline and 8cm below the collar bone. It’s outer end
was blunt. Left chest cavity was seen penetrated through the 6th
intercoastal space. Lower lobe of left lung was seen underfixed. The chamber of the left
ventricle was found to be pierced. The track of the wound
was directed backward downwards and to the right and the
total minimum depth was 11.5cm. Incised wound 5x5x 1.5cm
horizontally on the front of right fore arm 6cm below the elbow.

Post-mortem injuries:
1. Crushed decapitating wound on the neck at the level of 4th vertebrae.
2. Grazed abrasion 30 x 20cm on the back of middle of trunk, 6cm below root
of neck.
• Air passages contained blood. Lungs were pale. Stomach was empty.-
heart showed subendocardial bleeding. Spleen was found to
be shrunken. All internal organs were pale. 220
GOVT MEDICAL COLLEGE, TRIVANDRUM
LUMINAIRE-2015

Questions:
1. Who should hold the inquest in this case?
Hint: Police
2. Estimate the time since death from the post-mortem changes.
Hint: 36 - 48 hrs.
3. How can you account for the post-mortem skin slippages?
Hint: Hot surface contact of railway line.
4. What is the fatal nature of the Ante-mortem injury no. 1.
Hint: Sufficient in the ordinary course of nature to cause death.
5. What will you call me injury which was seen on the right fore arm?
Hint: Defence wound.
6. How much time might have elapsed after his last solid meal ?
Hint: >6hrs.
7. What is your conclusion from the sub-endocardial bleeding?
Hint: Head injury.
8. What is the reason why the spleen was found to be shrunken?
Hint: H’age
9. Is this a case of murder? State Yes / No / Cannot be stated?
Hint: Yes, incised penetrating wound.
10. What is your impression regarding the manner of death ? Give reasons.
Hint: Homicidal
11. What laboratory investigations will you do in such a case?
12. How will you word your opinion as to the cause of death?

CASENO.6
HISTORY
• A 26 year old Christian married woman was found hanging in a ceiling fan
in her own house. Her husband left her two years ago for
employment in Abu Dhabi. The Circle Inspector conducted the
inquest and sent the body for postmortem.
POSTMORTEM APPEARANCES
• Body was that of a well nourished and moderately built adult female of
height 15 7cm and weight 50kg A ligature made of a
synthetic sari was seen tied around the neck with a slip knot on
the right side. The noose measured 30cm, long free portion 147cm
and short free portion 10cm. Right eye was open and its pupil was dilated while the left
eye was closed and its pupil was constricted. Tip of tongue
protruded - not bitten. Vulva vagina and other body orifices 221
GOVT MEDICAL COLLEGE, TRIVANDRUM
LUMINAIRE-2015

were normal, hymen showed multiple old healed tears and vagina
admitted three fingers. Other body orifices were normal. Nails were bluish. Sali
vary dribble mark was not seen.
• Body was cold. Rigor mortis retained only in lower limbs. Postmortem
staining was seen at the lower part of limbs and was fixed. Both
iliac fossae showed greenish discolouration. (Body was not
refrigerated).

Injuries (antemortem)
1. Pressure abrasion 29cm long noncontinuous, obliquely placed on the
neck, above the level of thyroid cartilage being 6cm below the
chin (3cm broad), 8cm below occiput (2.5cm broad) and 7cm
below left ear lobule (3cm broad) with the discontinuity of 2cm
below left ear. On flap dissection of neck done under bloodless field the subcutaneous
tissue underneath the injury appeared pale and dry. Hyoid bone,
cartilages and other neck structures were found to be intact.
2. Abrasion 3x3cm overlying the left malar eminence.
3. Two skin deep incised wounds 3x0.2cm and 4x0.3cm on the front of left
wrist.
• The air passages were congested. Lungs were congested and
oedematous. The stomach contained 200ml of bile stained
fluid without any unusual smell its mucosa was normal. Uterus
measured 16x 10x5cm and its cavity contained a male foetus having a crown heel
length of 40cm within intact membranes. All other organs were
congested, otherwise normal, Urinary bladder was empty.

Questions
1. How will you word your opinion as to the cause of death ?
2. What is the fatal nature of injury No. 1?
3. How Locards’ principle of exchange can be applied in this case to
ascertain the manner of death ?
4. What is the age of the foetus ? Give reasons.
5. What could be the time since death ? Give reasons for your opinion ?
6. What could be the manner of death ? Substantiate your answer.

222
GOVT MEDICAL COLLEGE, TRIVANDRUM
LUMINAIRE-2015

WOUND CERTIFICATE
SI no: Date of :
Date and time of examination: Medical College :
l. Name
2. Age
3. Sex
4. Address
5. Occupation
6. Brought by whom
(as per requisition of CI of police, a person named………….., aged …………..
was brought for examination of injury involving Cr No………. He was
accompanied by PC No…...) 223
GOVT MEDICAL COLLEGE, TRIVANDRUM
LUMINAIRE-2015

7. Consent of subject
a) expressed
b) implied
8. Two Marks of identification
9. History and alleged cause of injury
10. Details of injury
• Age of injury, Name, Size, part, kind of weapon
• Simple/grevious
11. No: of additional sheets
12. Whether dying declaration was required
• if yes whether police or magistrate was informed
13. Investigation results, if any
14. .DOA IP no: DOD:
15. Condition on discharge
16. Opinion, with details including Cause of injury, Nature of injury, age of
injury, simple/grevious, Whether injury could be caused as alleged
by subject

Place: Name:
Date : Designation:
Issued to ————————— as per requisition no ———dated--------------------------

Signature of issuing authority

Case 1 : RTA
A patient in coma stage brought to the casualty with a head injury.(More details
will be given in the question paper)

Questions:
1. What is bumper injury?
2. Classify injuries in RTA in case of a pedestrian
3. What is brushburn abrasion?
4. What will be the cause of death if femur fracture occurs?
ans: 1. fat embolism
2. rapid loss of blood

Case 2 : RTA
An RTA brought to casualty with fracture on left tibia and fibula,CT scan showed
extradural hemorrhage. 224
GOVT MEDICAL COLLEGE, TRIVANDRUM
LUMINAIRE-2015

Questions:
1. If bumper height is >20cm, how will u explain the injury.
Hint: 20cm from above heel,so its a primary impact
injury-(bumper&acture).
2. Can the neurosurgeon do surgery for hematoma? substantiate
Hint: Yes, as per the law of emergency doctrine section 92 of IPC,
he can do for the benefit of the person even without the
consent& is not an offence.
3. If power failure during surgery and failure of surgery occured,then what is
the best defence plea?
Hint: Inevitable acident
4. What is underrunning / tailgating?
5. Seatbelt syndrome , whiplash injury?

Case 3 : RTA
An RTA, patient was first taken to thaluk hospital then reffered to medical
college. X ray revealed fracture of right tibia.

Questions
1. Who should issue the wound certificate?
2. Is it a Grevious hurt?

Case 4 : RTA
An 9 yr old student was hit by an auto while he was crossing the
road. The auto driver took him to the hospital. On examination the child had fracture in
his Lt upper arm. The auto driver gave the consent for treatment

Questions
1. Is the consent valid?
Questions of drunkenness
A man driving carelessly hit an old lady and she died, police arrested the man
and he was drunk.
Questions:
1. What was the crime committed and under which session?
Hint: Rash and negligent act-section 304-A of IPC.
2. What is sec 53 Cr PC?
Hint: a person under arrest can be examined without consent
3. Rate of metabolism of alcohol? 225
GOVT MEDICAL COLLEGE, TRIVANDRUM
LUMINAIRE-2015

Hint: Follows zero order kinetics


Other questions which can be asked:
1. DD of alcoholism.
2. Tests for muscle coordination.
3. Preservatives used
4. MLI of drunkenness
5. Delirium tremens.
6. Wernicke’s encephalopathy
7. Precautions taken for collection of blood.
8. The statutory limit of blood alcohol level in india is- 30mg%
9. Alcoholics anonymous

Probable questions
1. What is impotence,sterility?
2. The term for impotency in female
3. Test for evidence of recent sexual intercourse hint: penile wash
4. Name two civil and criminal cases each in which examination of potency
is asked for.
5. How can a person can have a child of his own if he is impotent
6. Quoad hoc
7. Vaginismus

Probable questions
1. A girl 15 year old raped with consent by fisherman Ratheesh and she was
brought for examination. (Details will be given in the question paper).
How will you differentiate between torn and fimbriated hymen?
2. What is the time taken for complete healing of torn hymen? Hint: 1 week
3. What is the absolute proof of presence of semen?
Hint: presence of atleast 1 unbroken spermatozoan or electrophoretic
LHD isoenzyme detection of sperms
4. How will you determine sexual intercourse from vaginal swab?
Hint: from motility of sperm
full => 3 hrs
50% => 8 hrs’
10% => 24 hrs
5. Write 4 causes of rupture of hymen other than sexual intercourse
Hint: accidental
surgical operation 226
GOVT MEDICAL COLLEGE, TRIVANDRUM
LUMINAIRE-2015

ulceration
masturbation
Other questions
1. Define adultery
2. Statutory rape,
3. Test for evidence of recent sexual intercourse in male
hint: penile washing-method: retract the foreskin of the penis and wash
glans penis- centrifuge the washing-take sediment and add KI. If vaginal
epithelial cells are present, glycogen in the cells wll take up Iodine
and stain brown.
4. Two tests for spermatozoa:
Hint: Florence test and Barberio test
5. Rape trauma syndrome

CONSENT -ENGLISH
The doctor has informed me about the nature of the body examination that I
have to undergo to ascertain
a) if sexual intercourse has taken place or not. ( Rape)
b) if I have consumed alcohol / whether I am under its influence
(Drunkenness)
c) the ability to perform the sexual act. ( Potency)
He warned me that the findings may go in my favour or against me when
reporting the matter to the concerned authorities. He also informed me that I have the
right to refuse such an examination. Having understood everything, I express my
willingness to undergo this examination and give the required samples.
Signature
Name, date
REPORT TO BE FORWARDED WITH THE MATERIAL OBJECTS SENT FOR CHEMICAL
EXAMINATION [VISCERA DISPATCH FORM]
Name of institution................................................................................................................
Post-mortem no:..............................................................Date...............................................
Name of the deceased...............................................,,,,,,,,,,,,,,aged about..............................
Crime no:........................................of.............................................................. police station
Material objects:
1. Stomach and parts of intestine with contents
2. Part of liver and one kidney
3. Blood
4. Urine.
Method of preservation : rectified spirit/saturated saline 227
GOVT MEDICAL COLLEGE, TRIVANDRUM
LUMINAIRE-2015

Mode of packing : collected in glass bottles, wrapped with paper, tied and
sealed
Impression seal used: affixed
Copy of label affixed to bottles/packages(enclosed)
Information furnished by police:
Clinical history,treatment,progress etc
Post-mortem appearance:
Examination required:
Signature:
Name:
Designation:

PM No:................................................... Dated:...............................
To
The Chief Chemical Examiner to Government
Thiruvananthapuram
Sir,
I am forwarding herewith the above mentioned material objects through
Sri................................PC No:.............................for chemical examination and certificate.
I request that 3 copies of your certificate may be sent to me at an early date.

Yours faithfully,
Medical officer
Copy to:
[Label]
AGE OF INJURIES
Abrasions
fresh - bright red
12 to 24 hours - bright scab
2 to 3 days - reddish brown scab
4 to 7 days - dark brown to brownish black scab
After 7 days - scab dries, shrinks and falls off
Bruise
At first - red
Few hours to 3 days- blue
4th day - bluish black to brown [hemosiderin]
5 to 6 days - greenish[hematoidin]
7 to 12 days - yellow[bilirubin]
2 weeks - normal 228
GOVT MEDICAL COLLEGE, TRIVANDRUM
LUMINAIRE-2015

Age of burns
1 hour - redness
6 hours - inflammatory reaction
12 to 24 hours - exudate begins to dry
2 to 3 days - dry brown crust
4 to 6 days - Sloughs falls off
After 2 to 3 weeks - scar formation

229
GOVT MEDICAL COLLEGE, TRIVANDRUM

You might also like